+ All Categories
Home > Documents > CANADIAN IMGS GUIDE TO OSCE AND PRACTICE · in an OSCE scenario You can use this study guide to...

CANADIAN IMGS GUIDE TO OSCE AND PRACTICE · in an OSCE scenario You can use this study guide to...

Date post: 14-Oct-2019
Category:
Upload: others
View: 79 times
Download: 5 times
Share this document with a friend
317
CANADIAN IMGS GUIDE TO OSCE AND PRACTICE A practical guide for International Medical Graduates who wish to pursue Medical Residency Training in Canada Hanan Ahmed
Transcript
Page 1: CANADIAN IMGS GUIDE TO OSCE AND PRACTICE · in an OSCE scenario You can use this study guide to prepare for the NAC OSCE and MCCQE2. The content was reviewed for the most recent updates;

1

CANADIAN IMGS GUIDE TO OSCE AND

PRACTICE

A practical guide for International Medical Graduates who wish to pursue

Medical Residency Training in Canada

Hanan Ahmed

Page 2: CANADIAN IMGS GUIDE TO OSCE AND PRACTICE · in an OSCE scenario You can use this study guide to prepare for the NAC OSCE and MCCQE2. The content was reviewed for the most recent updates;

2

Page 3: CANADIAN IMGS GUIDE TO OSCE AND PRACTICE · in an OSCE scenario You can use this study guide to prepare for the NAC OSCE and MCCQE2. The content was reviewed for the most recent updates;

3

Author and publisher; Hanan Ahmed, M.B.B.S

Internal Medicine resident, PGY3

University of Alberta

Canadian IMGs Guide to OSCE and Practice

ISBN 978-0-9947342-0-4

© 2015 Hanan Ahmed

All rights reserved. This book or any portion thereof

may not be reproduced or used in any manner

whatsoever without the express written permission of

the publisher except for the use of brief quotations for

scholarly purposes.

Page 4: CANADIAN IMGS GUIDE TO OSCE AND PRACTICE · in an OSCE scenario You can use this study guide to prepare for the NAC OSCE and MCCQE2. The content was reviewed for the most recent updates;

4

Disclaimer

This book is designed as a study guide for Immigrant and

Canadian International Medical Graduates to give them

an idea of the expectation of Canadian OSCE exams. It

was not written to provide advice on patient care or

management, and is not intended as a comprehensive

medical resource. Consultation with appropriate

resources is suggested when treating patients. The

author and editors are not responsible for any harm,

direct or indirect that results from the application of this

material to patient care.

Patient’s names used in this book are arbitrary. Any

similarity to reality is a complete coincidence.

Page 5: CANADIAN IMGS GUIDE TO OSCE AND PRACTICE · in an OSCE scenario You can use this study guide to prepare for the NAC OSCE and MCCQE2. The content was reviewed for the most recent updates;

5

Forward

To Doctors who move to Canada with a strong will

to succeed

Page 6: CANADIAN IMGS GUIDE TO OSCE AND PRACTICE · in an OSCE scenario You can use this study guide to prepare for the NAC OSCE and MCCQE2. The content was reviewed for the most recent updates;

6

Acknowledgment

My heartfelt gratitude to all people who supported this project. Many thanks to

my Program Director Dr. Darryl Rolfson for his encouragement, trust and support

throughout the whole process. Thanks also to Dr. Peter Hamilton for his help and

guidance. My appreciation to Dr. Liam Rourke for supporting the initial phases of

this work. Many thanks to the editors who volunteered their time and effort; Dr.

Sara Belga, Dr. David Ross, Dr. Shirley Schipper, Dr. Jay Shavadia, Dr. Subrata

Datta, Dr. Stephanie Keeling. Dr. Fraulin Morales, Dr. Anca Tapardel, Dr. Sadik

Salman, Dr. Abbeir Hussain, Dr. Erica Paras, Dr. Erin Toor, Dr. James Yeung, Dr.

Wasif Hussain, Dr. Abdullah Saleh.

My deepest thanks go as well to Cayti Beyer, who extended her passion to help

IMGs and volunteered to do the language editing.

Finally I would like to express my deepest appreciation to my dear husband,

Ayman, for his patience, support and help with website design.

Page 7: CANADIAN IMGS GUIDE TO OSCE AND PRACTICE · in an OSCE scenario You can use this study guide to prepare for the NAC OSCE and MCCQE2. The content was reviewed for the most recent updates;

7

Preface

When I moved to Canada, I was surprised to learn that there are very little

resources for International Medical Graduates (IMGs). There isn’t even a single

book to orient IMGs. Witnessing the struggle many IMGs face in preparing for

Canadian exams due to the lack of study guides targeting the special needs of this

group of physicians, I was motivated to execute this work. This book is an

overview of the Canadian Objective Structured Clinical Examinations (OSCEs). The

goal of this book is to introduce you to the special aspects and expectations of

Canadian exams and provide resources and links that will help you study while

preparing for these exams. While the book helps you prepare for exams step by

step, it is also a useful resource to understand what is expected from you when

you start your training.

Chapter one covers the type and content of OSCE exams IMGs commonly write. It

details the structure of the exams and pin points high yield scoring points.

Chapter two is history taking. History taking is the most common scenario in

Canadian OSCEs. This chapter details the various history taking situations based

on a general patient population and can be used as a question bank to choose

from for focused histories as needed on a case by case basis.

Chapter three is physical exam. It details most possible physical exam scenarios

and what is expected from examinees. The special aspects, expectations and

depth of physical exam pertinent to Canadian OSCEs are presented in this

chapter.

Chapter four is writing and counselling tips. An overview of counselling is

presented in this chapter along with any scenarios in which you may be asked to

document

Chapter five is clinical cases. Written in interactive format to facilitate practice,

this chapter covers 18 common clinical cases designed to reflect the exam format

Page 8: CANADIAN IMGS GUIDE TO OSCE AND PRACTICE · in an OSCE scenario You can use this study guide to prepare for the NAC OSCE and MCCQE2. The content was reviewed for the most recent updates;

8

and give you an idea of its flow. The cases presented are common clinical

scenarios. Each case has a variation of the case section at the end, to help you

practice a wide range of differentials when assessing common presentations. You

can use them as a guide as you design a study approach to more clinical cases.

Chapter six is ethical and legal principles. It goes over the basic ethical

foundations you need to observe during your practice and which could be tested

in an OSCE scenario

You can use this study guide to prepare for the NAC OSCE and MCCQE2.

The content was reviewed for the most recent updates; however, I strongly

encourage you to go over MCC website regularly for updates regarding the

exams.

The best way to use this book is as a study guide along with other resources. The

material presented here is meant to give you an idea of how things are done in

Canada and what is expected from you in an exam setting. It is not a

comprehensive medical resource. Every effort was made to ensure the accuracy

in this study guide. However, this book was not designed for direct patient care.

Consultation with appropriate resources is strongly encouraged when treating

patients.

Page 9: CANADIAN IMGS GUIDE TO OSCE AND PRACTICE · in an OSCE scenario You can use this study guide to prepare for the NAC OSCE and MCCQE2. The content was reviewed for the most recent updates;

9

Section Editors

Chapter 1

Sara Belga, MD

Third Year Internal Medicine resident

University of Alberta

Chapter 2

Shirley Schipper, MD, CCFP

Associate Professor

Department of Family Medicine

University of Alberta

David Ross, MD, CCFP

Associate Professor

Department of Family Medicine

University of Alberta

Chapter 3

Anca Tapardel, MD, FRCPC

Assistant Clinical Professor

Department of Internal Medicine

University of Alberta

Chapter 4

Cardiovascular Physical exam

Jay Shavadia, MD

Division of Cardiology, PGY6

University of Alberta

Gastrointestinal and Respiratory physical exam

Erin Toor, MD, FRCPC

Division of General Internal Medicine

Department of Medicine

University of Alberta

Page 10: CANADIAN IMGS GUIDE TO OSCE AND PRACTICE · in an OSCE scenario You can use this study guide to prepare for the NAC OSCE and MCCQE2. The content was reviewed for the most recent updates;

10

Neurology physical exam

M. Wasif Hussain, MBBS

Adult Neurology resident PGY4

Department of Neurology

University of Alberta

Musculoskeletal physical exam

Stephanie Keeling, MD, FRCPC

Associate Professor

Division of Rheumatology

Department of Medicine

University of Alberta

Breast physical exam and Fundoscopy

Sadik Salman, MD, ABIM, FRCPC

General Internist

Department of Internal Medicine

University of Alberta

Chapter 5

Case 1 and 2

Subrata Datta, MD,

Division of Internal Medicine, PGY4

University of Alberta

Cases 3 and 4

Jay Shavadia, MD

Division of Cardiology, PGY6

University of Alberta

Cases 5 and 6

James Yeung, MD, FRCPC

Rheumatologist

Division of Rheumatology

Department of Medicine

University of Alberta

Cases 7, 8, 10 and 15

Page 11: CANADIAN IMGS GUIDE TO OSCE AND PRACTICE · in an OSCE scenario You can use this study guide to prepare for the NAC OSCE and MCCQE2. The content was reviewed for the most recent updates;

11

Sadik Salman, MD, ABIM, FRCPC

General Internist

Department of Internal Medicine

University of Alberta

Case 9

Erin Toor, MD, FRCPC

Division of General Internal Medicine

Department of Medicine

University of Alberta

Cases 11, 12, 13 and 14

Erica Paras, MD

Obstetrics and Gynecology Resident, PGY4

University of Alberta

Cases 16 and 17

Abbeir Hussein, M.D

Pediatrics Resident, PGY3

University of Alberta

Case 18

Abdullah Saleh, MD

General Surgery Resident. PGY5

University of Alberta

Chapter 6

Fraulin Morales, MD, FRCPC

Associate Clinical Professor General Internal Medicine University of Alberta

English language editor of the whole book; Cayti Beyer IMG Program Developer/Coordinator & Career Coach Directions/Bow Valley College Calgary, Alberta

Page 12: CANADIAN IMGS GUIDE TO OSCE AND PRACTICE · in an OSCE scenario You can use this study guide to prepare for the NAC OSCE and MCCQE2. The content was reviewed for the most recent updates;

12

Page 13: CANADIAN IMGS GUIDE TO OSCE AND PRACTICE · in an OSCE scenario You can use this study guide to prepare for the NAC OSCE and MCCQE2. The content was reviewed for the most recent updates;

13

TABLE OF CONTENTS

ABOUT THE EXAM ..................................................................................................... 17

Overview of OSCE Exams ................................................................................................................................ 18

What Does the Exam Cover? ........................................................................................................................... 18

Interaction with the Standardized Patient (SP) ................................................................................................ 18

Interaction with the examiner ........................................................................................................................ 19

Interaction with the nurse/QE2 ...................................................................................................................... 19

Cases with extra family member ..................................................................................................................... 20

Structure of MCCQE2 ..................................................................................................................................... 20

Structure of NAC ............................................................................................................................................ 21

Scoring .......................................................................................................................................................... 21

HISTORY TAKING ....................................................................................................... 25

Introduction .................................................................................................................................................. 26

Basic Elements of History ............................................................................................................................... 28

Special Populations: History ........................................................................................................................... 36

PHYSICAL EXAMINATION ........................................................................................ 46

Introduction .................................................................................................................................................. 47

Cardiovascular System Examination (CVS P.E) ................................................................................................. 49

Respiratory System examination: ................................................................................................................... 64

Abdominal Examination ................................................................................................................................. 66

Neurological P.E ............................................................................................................................................. 72

Musculoskeletal System Examination (MSK) ................................................................................................... 84

Breast Physical Examination; ........................................................................................................................ 105

Page 14: CANADIAN IMGS GUIDE TO OSCE AND PRACTICE · in an OSCE scenario You can use this study guide to prepare for the NAC OSCE and MCCQE2. The content was reviewed for the most recent updates;

14

Fundoscopy; ................................................................................................................................................ 107

WRITING AND COUNSELING TIPS ....................................................................... 111

Introduction; ............................................................................................................................................... 112

Sample writing scenarios .............................................................................................................................. 112

Counseling; .................................................................................................................................................. 120

SAMPLE CLINICAL CASES ....................................................................................... 125

Case 1; Cough .............................................................................................................................................. 128

Case 2; Fatigue ............................................................................................................................................. 135

Case 3; Chest pain in the ER .......................................................................................................................... 142

Case 4; Chest pain in an out patient .............................................................................................................. 153

Case 5; Back pain ......................................................................................................................................... 161

Case 6; Knee pain ......................................................................................................................................... 171

Case 7; Diabetic Ketoacidosis (DKA) .............................................................................................................. 179

Case 8; DM history and counseling................................................................................................................ 191

Case 9; Diarrhea ........................................................................................................................................... 202

Case 10; Smoking counseling ........................................................................................................................ 210

Case 11; Preeclampsia (PET) ......................................................................................................................... 222

Case 12; Contraception ................................................................................................................................ 233

Case 13; Postmenopausal bleeding ............................................................................................................... 245

Case 14; Abortion......................................................................................................................................... 255

Case 15; Cancer patient refusing treatment ................................................................................................... 265

Case 18; Motor vehicle accident (MVA) ......................................................................................................... 291

ETHICAL AND LEGAL PRINCIPLES ...................................................................... 302

Introduction; ............................................................................................................................................... 303

Page 15: CANADIAN IMGS GUIDE TO OSCE AND PRACTICE · in an OSCE scenario You can use this study guide to prepare for the NAC OSCE and MCCQE2. The content was reviewed for the most recent updates;

15

Confidentiality; ............................................................................................................................................ 304

Autonomy, capacity, informed consent, substitute decision maker and power of attorney; ............................ 305

Beneficence and non-maleficence;................................................................................................................ 307

Justice; ........................................................................................................................................................ 308

Dealing with ones’ own and others errors; .................................................................................................... 309

Abbreviations .............................................................................................................................................. 312

INDEX ........................................................................................................................... 313

Page 16: CANADIAN IMGS GUIDE TO OSCE AND PRACTICE · in an OSCE scenario You can use this study guide to prepare for the NAC OSCE and MCCQE2. The content was reviewed for the most recent updates;

16

Page 17: CANADIAN IMGS GUIDE TO OSCE AND PRACTICE · in an OSCE scenario You can use this study guide to prepare for the NAC OSCE and MCCQE2. The content was reviewed for the most recent updates;

17

Chapter 1

About the exam Edited by; Dr. Sara Belga

Page 18: CANADIAN IMGS GUIDE TO OSCE AND PRACTICE · in an OSCE scenario You can use this study guide to prepare for the NAC OSCE and MCCQE2. The content was reviewed for the most recent updates;

18

Overview of OSCE Exams

OSCE stands for Objective structured Clinical Examination. Its purpose is to expose examinees to the

same set of clinical scenarios in order to guarantee fair evaluation of all participants. IMGs seeking

residency positions in Canada are required to pass one or more OSCEs. This book will familiarize you

with the structure, environment and medical topics encountered in these exams. I aim at providing a

complete guide to successful performance in OSCEs from A-Z, including many practice cases.

The exams may be made up of both clinical and written components. The clinical part corresponds to a

number of different cases. Each one of them starts with a clinical scenario posted on the door, asking

you to perform a specific task. When you enter the room you will find 2 people; the examiner and a

standardized patient (SP). You have a certain amount of time to perform your task (history and/or

physical exam and/or manage). Then you will move to the next station repeating through all 12 stations.

IMGs are required to do one or more Canadian OSCEs depending on the requirements of the province

they wish to practice in. These include: Medical Council of Canada Qualifying Exam 2 (MCCQE2) and

National Assessment Collaboration (NAC). Overview of the structure of each of them is presented in this

chapter. I encourage you to review the MCC website regularly as the exams’ structure may change.

What Does the Exam Cover?

You will be tested on common diseases in the major areas of medicine: Internal Medicine, Pediatrics,

Surgery, Obstetrics and Gynecology, Psychiatry, Preventive Medicine and Community Health.

You could be asked to do one or more of the following:

History taking

Physical Exam

Manage an emergency situation

Counsel a patient or address concerns about health issues

There may be a nurse in the room in the management stations. You will be scored on your interaction

with him or her.

In MCCQE2 you may be asked to fill in admission orders, write a progress (SOAP) note, a letter to the

employer or a prescription. There is no longer a written component to the NAC OSCE.

Interaction with the Standardized Patient (SP)

Page 19: CANADIAN IMGS GUIDE TO OSCE AND PRACTICE · in an OSCE scenario You can use this study guide to prepare for the NAC OSCE and MCCQE2. The content was reviewed for the most recent updates;

19

Standardized patients are actors and/or actresses who are paid to perform as patients in OSCE exams.

They will give you history and if required mimic physical findings. They are trained to give you a clear

history if you ask the right questions. Be aware that some of them have real findings like a scar from

previous surgery or a skin rash. They may also mimic real findings like tenderness or rebound tenderness

on abdominal examination. Be attentive and gentle during physical exam and report what you find.

Respect the SP as you would your own patients; you will be scored on how you interact with them.

Make sure you introduce yourself and address the SP by the name given on the door sign. Explain why

you need to ask sensitive or personal questions, wash your hands and respectfully drape SP during

physical examination. Examples on the variety of scenarios and challenges that you may encounter

when dealing with SPs are presented as cases in chapter 5. The SPs undergo intense training for these

high stakes OSCEs to ensure standardization across all exam centers.

Interaction with the examiner

You will be provided a note book with bar codes when you sign in. You will give each examiner one or

more bar code to stick on your scoring sheet. The examiner fills in the sheet based on his/her

observation of your performance. The number of bar codes you give to the examiner will be indicated in

the question stem. In general, you will always give the examiner one bar code except in the couplets of

MCCQE2 (see below for examination structure). Beyond that your interaction with the examiner is

limited to the following:

The examiner may tell you some findings on physical exam.

The examiner may stop you to ask you to re-read the question. This may happen if you are not doing the

specific task you are asked to do; for example, you may be taking history where the question is about

physical exam. Don’t panic if that happens, simply read the question again, and take few seconds if

needed to organize your thoughts.

The examiner may also stop you to protect the SP if you did something that made the SP uncomfortable.

Make sure you work in advance on your professional manners and treat SPs with utmost respect. If you

feel your interaction with the SP has gone off track, apologize to the SP and tell the examiner you didn’t

mean to make the SP uncomfortable and ask if you may continue.

The examiner may ask you questions, which are usually related to the clinical presentation you

just saw; questions may include differential diagnoses, most likely diagnosis,

management, investigations or response to ethical issues.

Interaction with the nurse/QE2

Introduce yourself and get to know the people around you (e.g. ask the nurse his or her name if he/she

doesn’t volunteer that information). The nurse will only do the tasks you ask him or her to do. In

Page 20: CANADIAN IMGS GUIDE TO OSCE AND PRACTICE · in an OSCE scenario You can use this study guide to prepare for the NAC OSCE and MCCQE2. The content was reviewed for the most recent updates;

20

emergency stations, he or she will not give oxygen and put the patient on the monitor unless you ask

him or her to do so. Provide clear and direct instructions. Avoid overwhelming the nurse by giving

multiple requests at the same time, wait until he or she completes them all before you tell him or her

what to do next. An example of an interaction with a nurse is presented in chapter 5.

Cases with extra family member

In some cases, especially when the patient has delirium/dementia or sometimes in pediatrics, you will

be asked to interact with one or more family members. Do the task you are asked to perform. If it is

history then stick to it, but make sure to apply basic ethical principles of autonomy and confidentiality

and state that you would like to speak to the patient when possible. If the case involves a teenager and

parents, respectfully ask the parents to leave the room when asking sensitive personal questions. If the

patient is a young child (<10 years) try to involve him/her in the conversation as much as you can. If the

case involves both partners always ask your patient if it’s ok for partner to stay.

Structure of MCCQE2

Consists of two sessions over the course of two days:

The first session is held on a Saturday and is comprised of 8 stations with each being 10 minute long +

two rest stations. In all of them you are interacting with a SP. You have 2 minutes to read the

instructions on the door and jog down some notes. Then you have 10 minutes to interact with the SP. At

9 minutes, a buzzer will sound to give you a warning. At 10 minutes, a second buzzer indicates the end

of the station and you have to exit the room. The examiner may ask you one or two quick questions.

Please note that this will be indicated in the question stem posted on the door and that the examiner

will ask the questions at the 9 minute buzzer; therefore, you need to make sure you are finished with

the SP at 9 minutes. The second session is held on a Sunday and is comprised of:

4 stations with a mix of 6 + 6 couplet stations. The sessions alternate between patient encounter and

written stations. You have 2 minutes to read the instructions and jog down some notes, and then you

have 6 minutes to interact with the patient. A buzzer will sound at 5 minutes indicating that you need to

wrap up. At the second buzzer after 6 minutes, the examiner will hand you the post encounter probe

(written component) and you must exit the room to complete it. Keep in mind that you may be asked

questions at the 5 minute buzzer and this will be indicated on the question stem. The written

component is similar except that you will be writing (Examples are given in chapter 4).

One 14 minute station; focused on assessing more complex cases which could involve family members

or members of a health team. You have 2 minutes to read the question and jot down your notes. A

buzzer will sound at 13 minutes, and at 14 minutes at which you must exit the room.

Page 21: CANADIAN IMGS GUIDE TO OSCE AND PRACTICE · in an OSCE scenario You can use this study guide to prepare for the NAC OSCE and MCCQE2. The content was reviewed for the most recent updates;

21

Keep in mind that the staff members are there to help you. Don’t hesitate to ask for help or guidance. A

tour on the exam day is posted on the MCC website. I strongly encourage you to go over it;

http://mcc.ca/examinations/mccqe-part-ii/exam-preparation-resources/

Structure of NAC

The NAC OSCE is a one day exam. You have 2 minutes to read the instructions and 11 minutes with the

patient. A warning buzzer will sound at 8 minutes, another buzzer at 11 minutes to indicate the end of

the station. At 8 minutes the examiner may ask you questions and as in MCCQE2, this will be indicated

in the question stem. In other cases you have the full 11 minutes with the patient. The types of clinical

scenarios are very similar to MCCQE2, and this will be covered in chapter 5

I strongly encourage you to review the comprehensive guide to NAC examination on the MCC website:

http://mcc.ca/examinations/nac-overview/osce-station-therapeutics-descriptions/

http://mcc.ca/examinations/nac-overview/exam-day/

Scoring

There are some differences in scoring between MCCQE2 and NAC. Only IMGs write NAC while both

Canadian Medical Graduates and IMGs write MCCQE2. For example, you will be rated on language

proficiency in NAC but not in MCCQE2. In both exams there is a space on the scoring sheet for the

examiner to write any concerns about your conduct. Below is an overview of what you need to know

about scoring to successfully prepare for these exams. A preset passing score is determined by the

examination committee, and you are not compared to other candidates. Refer to the MCC website for

further details.

MCCQE2 Scoring;

In the OSCE component, the examiner will observe your interaction and fill in a checklist of items. The

checklist covers clinical skills as well as communication, professional and ethical expectations. For

example, when you take a history or perform a physical exam you maybe scored on your organizational

skills, meaning that covering everything but in a disorganized way will cost you credit. It is not enough to

have solid knowledge to pass Canadian exams; you need to demonstrate professional behavior. I

suggest that you keep the following points in mind as you study and practice:

Introduce yourself

Page 22: CANADIAN IMGS GUIDE TO OSCE AND PRACTICE · in an OSCE scenario You can use this study guide to prepare for the NAC OSCE and MCCQE2. The content was reviewed for the most recent updates;

22

Appropriate eye contact; this may differ among cultures. Do not stare at the patient, and don’t ignore

them. Find a medium where you are comfortable, showing respect and acknowledgement of your

patients. You can do that by looking them in the eyes when you introduce yourself, and when you ask

questions. It is OK to write some notes while the patient is talking; you may attempt, however, to always

keep some eye contact.

Be aware of your body language; preparing for the exam is the best opportunity to identify any lapses in

your professional conduct. Listen attentively; nod your head to show understanding. Offer to help the

patient if you ask them to stand up, and watch for any signs of discomfort. Smile appropriately. Doctors

are genuinely caring, and you need to make sure that your caring nature is evident in your interaction

with the SP.

Address the SP by name as given on the door sign.

Wash your hands before physical examination. You may use hand sanitizer (it is alcohol-based gel that

you rub your hands with and will be available in the room)

Drape your patient appropriately; more on this will follow in chapter 3 (physical exam)

Avoid interrupting patients; in some cases the SP may be talkative and may even start telling life stories

not related to their medical complaints. In this case you may gently say: “Sorry Mr Smith, I would love to

listen to your story but I really need to stop you here and ask more questions about your headache”.

Alternatively you may say: “This sound like an interesting story, how about you tell me about it when we

finish talking about your back pain?”

Listen attentively; some examinees think that the OSCE is about asking all the possible questions

without listening to the patient’s answers. They often end up asking about information the SP had

already volunteered. Try to avoid repeating questions that the SP has already answered unless you are

seeking clarification in which case you should frame your question appropriately ie: “I know you already

mentioned that you take headache but can you clarify for me where exactly you feel it”

Be organized; now is your opportunity to prepare yourself to the variety of clinical scenarios that you

may encounter, work on your organizational skills as you practice, and use the sample cases in this book

as a guide.

Show empathy and understanding.

Give advice in a respectful manner. Avoid lecturing or judgemental statements, listen to their views and

ideas and address their concerns, and involve them in the management plan.

Keep in mind the basic ethical and legal principles as you interact with patients. This will be covered in

chapter 6.

Be specific, this is particularly important when asked questions and in the written component of the

exam. For example, if you think a surgical intervention is warranted specify the type of surgery; it is not

Page 23: CANADIAN IMGS GUIDE TO OSCE AND PRACTICE · in an OSCE scenario You can use this study guide to prepare for the NAC OSCE and MCCQE2. The content was reviewed for the most recent updates;

23

enough to just write or respond: surgery, for example; say; appendectomy if you think the patient has

appendicitis.

There are 5 sample cases and checklists on the MCC website. I strongly encourage you to review them;

http://mcc.ca/examinations/mccqe-part-ii/scoring/

In my personal experience and opinion, you can cover more points than what is listed in the MCC

checklist.

The interaction rating scale used by examiners to evaluate your communication with SPs can be found in

the following link;

http://mcc.ca/wp-content/uploads/Exams-interaction-rating-scale-items.pdf

Note that in each given station you will be scored on a number of clinical competencies and some of the

interaction rating scale areas as well.

NAC Scoring

You are required to perform at the level of a graduating Canadian Medical student. You may be

wondering; and what does that look like? I never practiced in Canada before! I totally get your point

since that is exactly how I felt when I first came here! I’m writing this book to tell you all about it.

Medical knowledge is so advanced that you may know too much or too little about any particular

disease, but what exactly do you need to do in a 5 or 10 min stations for your performance to be

considered satisfactory or beyond? You need to demonstrate that you have the basic knowledge to sort

out a patient’s medical complaint. You also need to demonstrate ethical and professional behavior.

Language proficiency and communication skills are also taken into account along with your

organizational skills. The above tips in MCCQE2 scoring are useful. You will become more familiar with

the exam expectations and how much to cover when you navigate through the cases. The NAC guideline

rating scale can be found here;

http://mcc.ca/wp-content/uploads/Exams-NAC-Guideline-rating-scale.pdf

In summary, you will be scored on about 9 competencies; history taking, physical examination,

organization, communication skills, language proficiency, diagnoses, data interpretation, investigations,

therapeutics and management.

I strongly encourage you to review NAC scoring system;

http://mcc.ca/examinations/nac-overview/scoring/

Page 24: CANADIAN IMGS GUIDE TO OSCE AND PRACTICE · in an OSCE scenario You can use this study guide to prepare for the NAC OSCE and MCCQE2. The content was reviewed for the most recent updates;

24

Important note: A pass score on the NAC OSCE is 65 (score range 0-100).

In order to be competitive for residency positions it is important to score

as high as possible in the NAC as this exam cannot be repeated unless

you fail. Therefore, it is advisable to ensure that your clinical

knowledge, communication skills and language fluency are assets not

detractors. Do not rush to take this exam until you are well prepared.

In general your goal is to achieve a score over 75 but the higher your

score, the more likely you will be considered for a CaRMS interview.

Programs such as Family Medicine may filter on your NAC score ie:

Ontario Family Medicine is transparent in their CaRMS description and

state that in general those invited for interview have a score of 81 and

higher.

References

1- Medical Council of Canada, accessed March 15/ 2015, www.mcc.ca

2- NAC Overview, accessed March 15/2015, http://mcc.ca/examinations/nac-

overview/exam-preparation-resources/

3- MCCQE2 preparation resources, accessed March 15/2015,

http://mcc.ca/examinations/mccqe-part-ii/exam-preparation-resources/

4- NAC scoring, accessed March 15/2015, http://mcc.ca/examinations/nac-

overview/scoring/

5- MCCQE2 scoring, accessed March 15/2015,

http://mcc.ca/examinations/mccqe-part-ii/scoring/

Page 25: CANADIAN IMGS GUIDE TO OSCE AND PRACTICE · in an OSCE scenario You can use this study guide to prepare for the NAC OSCE and MCCQE2. The content was reviewed for the most recent updates;

25

Chapter 2

History Taking Edited by: Dr. Shirley Schipper

Dr. David Ross

Page 26: CANADIAN IMGS GUIDE TO OSCE AND PRACTICE · in an OSCE scenario You can use this study guide to prepare for the NAC OSCE and MCCQE2. The content was reviewed for the most recent updates;

26

Introduction

History taking is probably the most important aspect of patient care. In addition to establishing

physician-patient relationship, asking the right questions in the right way determines the rest of

the management plan. One study estimated that the history alone led to the diagnoses in

78.58% of patients (1), while another study quotes the number 76% (2). It is the most common

element tested in OSCE exams, and is integral for patient care.

Stations in OSCE are 6-14 minutes long. During this limited time the examinee is trying to

achieve two goals; develop a trusting relationship with the patient, and get an accurate

comprehensive story. This may seem challenging, and some may feel overwhelmed. But it is

doable if you have a clear approach in mind, and approach the case from a solid clinical

reasoning foundation including a reasonable head to toe differential.

In most cases your history will be tailored to the case, and the job becomes easy if you have a

differential diagnoses. You need to analyze the chief complaint, make sure you are ruling out

fatal and incapacitating conditions, showing the examiner that you are thinking of more than

one possibility as a cause for the patient complaint. Then you need to go over the general

history including; past medical and surgical, family, medications, allergies and social histories.

All these sections are covered in details in this chapter. However, keep in mind that you don’t

need to ask every patient all the questions. You can think of this chapter as a questions bank

and use whatever is relevant to the case. This will be further illustrated as you go over specific

cases in chapter 5.

OSCEs are designed to measure competency of residents entering general practice, so it covers

a wide range of clinical encounters including; Internal Medicine, Pediatrics, Psychiatry,

Obstetrics and Gynecology and Surgery, Preventive Medicine and Community Health. Each

discipline has its unique essential history elements, which is covered in details in this chapter,

and expanded upon in case scenarios in chapter 5.

Practice is the key to master the science and art of history taking. A lot of IMGs are out of

practice for years before they write these exams, which makes their job a bit harder than

Canadian medical students and residents who take similar exams throughout medical school or

in the case of QE2, the same exam in year two of residency. This book is designed to help you

practice whether you choose to do it on your own or with a study group. I wrote my US Step 2

clinical skills exam at a different time than most of my colleagues. Left alone to practice I used a

Page 27: CANADIAN IMGS GUIDE TO OSCE AND PRACTICE · in an OSCE scenario You can use this study guide to prepare for the NAC OSCE and MCCQE2. The content was reviewed for the most recent updates;

27

pillow as my standardized patient (SP), and acted as if I was in a real exam; knocking the door,

and timing myself, etc... My sister made a lot of fun of me, but this was my way to succeed.

Bottom line; my advice to you; don’t just read this book, PRACTICE, PRACTICE, PRACTICE. It may

also help if you write scripts – word for word what you will say when presented with a common

case. You must practice for this exam by doing and saying not reading.

It is important to ask a mix of open and closed ended questions as you interview your patient.

Treat SPs as real patients; they are trained to respond to you, if you ask the right questions the

exam will move smoothly and you will be done on time. For example; in many cases just

starting with; how can I help you today? Will yield some information and save you some time.

Then you can ask direct questions to gather the rest of the story.

Key points;

1- Two Goals in mind when taking history; build a trusting relationship, and get accurate

comprehensive story

2- Be aware of time, always time yourself

3- Develop a clear differential diagnoses and your own organized approach to each

presentation

4- Use a mix of open and closed ended questions

5- Treat SP as you would treat your own patients in real life

6- Keep in mind the basic communication skills discussed in chapter 1

7- PRACTICE

Page 28: CANADIAN IMGS GUIDE TO OSCE AND PRACTICE · in an OSCE scenario You can use this study guide to prepare for the NAC OSCE and MCCQE2. The content was reviewed for the most recent updates;

28

Basic Elements of History

1. History of presenting illness (HPI)

Starts with analyzing the chief complaint. Common chief complaints include; pain, fever, cough,

fatigue, skin rash , trauma, etc…..

The first step in history is to have a differential diagnoses for all common chief complaints, then

analyze the chief complaint ruling in or out the most common, fatal and incapacitating

conditions. This is basically your HPI. Be precise and thorough; A systematic approach to data

collection leads to a logical differential diagnosis.

Apply your critical thinking skills to stay focused on the complaint so your history is relevant and

organized. Try to avoid low yield questions that are not relevant to ruling in/out from your DDX

as asking random questions will raise red flags in the examiner’s mind as to what you are

considering a DDX.

Common things to cover for each chief complaint include;

1- Onset; when did the problem start?

2- Frequency; How often do you get it?

3- Duration; How long does it last?

4- Timing; does it happen at a specific time of the day/night/month/year?

5- Progression; has two components depending on the nature of the problem. If it is a

one-time thing is it getting better or worse? If it comes in attacks for the last year or

so how is it progressing over time? And how does it start and progress for each

attack?

6- Characteristics; for pain it is the nature of pain (dull, sharp, stabbing, aching, etc…),

for other complaints it varies with the type of the complaint like the amount and

color of vaginal discharge. It will be illustrated for each complaint in detail in case

histories in chapter 5.

7- Aggravating and relieving factors: What makes it better? What makes it worse?

8- Pertinent positives and negatives; Appropriate review of systems (ROS) that rules in

or out differentials on your list. A comprehensive list of ROS is presented in this

chapter page 6.

Page 29: CANADIAN IMGS GUIDE TO OSCE AND PRACTICE · in an OSCE scenario You can use this study guide to prepare for the NAC OSCE and MCCQE2. The content was reviewed for the most recent updates;

29

9- Impact of the problem on patient’s life; How is it affecting your function, work?

Relationships? Sleep?

10- For pain remember to add; Location, radiation and severity. For the severity of pain

ask the patient; on a scale from 1-10, 10 being the worst pain ever, where do you

score your pain?

I will follow this layout for all common chief complaints detailed in chapter 5. Keep in mind that

some complaints; seizures for example, require that you develop a special HPI.

Common HPI questions for each chief complaint;

1- Onset

2- Frequency

3- Duration

4- Timing

5- Progression

6- Characteristics

7- Aggravating and relieving factors

8- Pertinent positives and negatives

9- Impact of the problem on patient’s life

10- For pain add; location, radiation and severity

2. Past Medical and Surgical History

In some cases it is enough to ask the patient if he/she has other medical problems, and any

surgeries or hospitalizations. In some cases you need to be more specific as patients may not

volunteer the information. For instance, in a patient presenting with chest pain you need to

specifically ask about the cardiovascular risk factors including; diabetes, hypertension, smoking,

dyslipidemia and previous heart attacks or heart disease. The patient will not understand what

dyslipidemia means so ask if he/she has high cholesterol. In some cases the patient may not

know and you will figure out what diseases they have from their medications history.

Page 30: CANADIAN IMGS GUIDE TO OSCE AND PRACTICE · in an OSCE scenario You can use this study guide to prepare for the NAC OSCE and MCCQE2. The content was reviewed for the most recent updates;

30

3. Family History

The same applies to family history. It may be enough in some cases to ask a generic question

like; what medical diseases run in the family? In other cases you need to be more specific; for

example; Is there a family history of inflammatory bowel disease in a patient with chronic

diarrhea. You may say; Does anyone in your family have chronic or bloody diarrhea? Has

anyone in your family ever been diagnosed with Crohn’s disease or Ulcerative Colitis?

4. Medication History

Includes;

1- Name of the medication

2- Dose

3- Frequency and time of ingestion

4- Route

5- Any recent change to medications

6- For how long has the patient been on the medication

7- In the elderly, you need to know who administers the medications and if they are

blister packed

8- Use of over the counter medications and herbal remedies

9- Compliance to medications

10- Significant side effects

In some cases the patient may have a list of his/her medications, or have the medication

bottles. Grab the list or bottles and quickly go over the medications with the patient to make

sure he/she is taking them. Don’t waste too much time trying to write all the details down.

Remember the sheet of paper they give you is for your own benefit and use, it will not be

scored.

5. Allergies and Medications Intolerance

Page 31: CANADIAN IMGS GUIDE TO OSCE AND PRACTICE · in an OSCE scenario You can use this study guide to prepare for the NAC OSCE and MCCQE2. The content was reviewed for the most recent updates;

31

Ask the patient if he/she is allergic to any medications, food and if they have environmental

allergies. If they say yes follow up what they are allergic to and what symptoms they get. In

some cases it is true allergy, in others it is medication intolerance like when the patient gets

headache from a drug. It is particularly important to document severe allergic reactions like

anaphylaxis.

6. Review of Systems

Next, I’m listing the most important questions to ask in each system. Note that you don’t need

to ask all the questions listed below when you go over the review of systems; ask questions that

are pertinent to the case to rule in/out your top differential diagnoses. You may choose to

include it in your HPI (which I recommend, as it shows you are organized and have a clear chain

of thoughts), or cover it as a separate section. If the patient has a positive symptom, you may

need to analyze it more depending on its significance.

Cardiovascular

1- Chest pain; Do you get chest pains?

2- Shortness of breath; including dyspnea at rest, exertional dyspnea, orthopnea and

paroxysmal nocturnal dyspnea; Do you feel short of breath? Do you feel out of

breath at rest or with activity? How far can you walk before you get short of breath?

On how many pillows do you sleep, or do you feel out of breath when you lie flat?

Does shortness of breath wake you up at night? Do you wake up at night gasping for

air?

3- Palpitations; Do you feel your heart is racing?

4- Dizziness; Do you feel dizzy or light headed?

5- Syncope and presyncope; have you ever lost consciousness or passed out? Do you

sometimes feel like passing out but you don’t?

6- Ankle edema; Do you have swelling of your ankles?

7- Fatigue; Do you feel tired more than usual? Or do you feel fatigued?

8- Intermittent claudication; Do you get pain in your calves when you walk?

9- Note that other complaints that are not primarily cardiac can be seen in some

cardiac diseases for example; cough and wheezing in heart failure.

Respiratory

1- Cough; Do you cough? What triggers your cough? Is your cough worse at a particular

time of the day/night?

2- Sputum; Do you bring up any phlegm? What’s the amount, color?

Page 32: CANADIAN IMGS GUIDE TO OSCE AND PRACTICE · in an OSCE scenario You can use this study guide to prepare for the NAC OSCE and MCCQE2. The content was reviewed for the most recent updates;

32

3- Hemoptysis; Do you cough up blood?

4- Shortness of breath; do you feel short of breath? Follow up with a functional

estimation, i.e.: how far the patient can walk?

5- Wheeze; do get noisy breathing or wheezing?

6- Chest pain

7- Snoring, night time apnea and excessive day time sleepiness to screen for

Obstructive sleep apnea (OSA); do you snore? Has your partner noticed that you

stop breathing at night? Do you feel sleepy during the day? Do you fall asleep behind

the wheel, reading a magazine, watching T.V?

Gastrointestinal

1- Abdominal pain; do you have tummy pain?

2- Nausea/Vomiting; Do you feel sick to your stomach? Do you throw up?

3- Hematemesis: Do you throw up blood? Do you throw up coffee-ground like vomit?

4- Diarrhea/Constipation; Do you have diarrhea? Are you constipated?

5- Acid reflux; Do you have a sour taste in your mouth? Do you have heart burn? Do

you have dry cough?

6- Jaundice; Have you noticed that your skin and eyes turned yellow?

7- Hematochezia; did you see any blood in your stool?

8- Tenesmus: do you feel the urge to poop and when you go to the washroom nothing

comes out?

Genitourinary

1- Dysuria; Does it hurt to pee?

2- Change in urine color or smell

3- Flank pain

4- Passing stones with urine; did you notice any small stones coming out with urine?

5- Vaginal or penile discharge

6- Groin masses, pain , itchiness

Neurological

1- Headache; Do you get headaches?

2- Loss of consciousness; Have you ever lost consciousness?

3- Seizures; Do you seize? Or do you have abnormal movements?

4- Visual changes

5- Hearing loss

6- Tinnitus; Do you feel ringing or any other noise in your ears?

7- Head injury

Page 33: CANADIAN IMGS GUIDE TO OSCE AND PRACTICE · in an OSCE scenario You can use this study guide to prepare for the NAC OSCE and MCCQE2. The content was reviewed for the most recent updates;

33

Musculoskeletal

1- Muscle pain; do you have pain in your muscles?

2- Joint pain or swelling; do you have pain or swelling of your joints?

3- Morning stiffness; do you feel stiff in the morning and need time to get going?

4- Muscle weakness (especially proximal muscles); do you find it hard to comb your

hair or get up from a chair?

5- Skin rash

6- Scleritis/episcleritis; do you have pain or redness of your eyes?

7- Jaw claudication; do you feel you get tired chewing and need to rest your jaw while

eating?

8- Headache

Constitutional

1- Fever; have you had a fever? Did you measure it?

2- Drenching night sweats; do you sweat at night so much that you have to change the

sheets?

3- Weight loss; have you lost weight recently? How much and over how long?

Endocrine

1- Heat or cold intolerance; do you feel hot/cold more than most people in the room?

2- Diarrhea/constipation

3- Fatigue and somnolence

4- Nervousness: do you feel more nervous or short tempered?

5- Palpitations

6- Sweating

7- Weight loss or gain; have you lost or gained weight recently?

8- Voice change; have you noticed any change to your voice?

9- Changes of vision; have you had loss of vision or double vision?

10- Skin/hair changes

11- Polydipsia/Polyuria; do you feel thirsty more than usual? Do you need to pee more

than usual?

Skin

1- Skin rash/itching; do you have skin rash or itching?

2- In diabetic patients, it is important to ask specifically about feet ulcers

3- In bedridden patients inquire about pressure ulcers

7. Social History

Page 34: CANADIAN IMGS GUIDE TO OSCE AND PRACTICE · in an OSCE scenario You can use this study guide to prepare for the NAC OSCE and MCCQE2. The content was reviewed for the most recent updates;

34

Social history is an important health determinant that can be modified. It includes the

following;

1- Occupation; which can be a risk factor for a disease, or affected by it, ask the patient;

what do you do for living? What did you do before?

2- Living conditions; with whom and where the patient lives, does he/she has easy

access to health care? One of the special things about the Canadian health care is

that you can consult a social worker and help patients out using means other than

drugs and surgery

3- Stress at work or relationships; do you have any stress in your life whether at work

or in your relationships?

4- Relationship status; do you have a partner?

5- Smoking; do you smoke? How much and for how long?

6- Alcohol intake; do you drink alcohol? What do you drink? And how often? I will

expand more on alcohol history next

7- Recreational drug use; do you use recreational drugs? What do you use? How often?

Will be further detailed next

8- Financial status, special cultural considerations and religion are appropriate in

certain cases

History of alcohol intake

Ask the patient what, how much and how often he/she drinks. If he/she drinks alcohol daily

over the guideline limits or binge drinks and if relevant to the case go over the CAGE

questionnaire;

1- C: Have tried to cut down on your drinking?

2- A: Do you feel annoyed by people criticizing your drinking?

3- G: Do you feel guilty about drinking?

4- E: Do you need alcohol first thing in the morning as an eye opener?

It is key that you gather this information without judgment. If the SP feels that your attitude is

judgmental they could resist giving you information.

According to JAMA; CAGE of 2 or more has a positive likelihood ratio of 6.9 for detecting alcohol

abuse or dependence in adults. (4)

If the CAGE is positive you can also inquire if the patient has had any legal problem because of

his/her drinking and if his/her work and relationship are affected by alcohol. This will help you

Page 35: CANADIAN IMGS GUIDE TO OSCE AND PRACTICE · in an OSCE scenario You can use this study guide to prepare for the NAC OSCE and MCCQE2. The content was reviewed for the most recent updates;

35

to identify any drinking problem and its extent. In cases where the patient has a drinking

problem, ask him/her if ready to change his/her behavior.

History of substance abuse

If positive for recreational drug use in addition to the general questions mentioned above, if

relevant to the case, the following should be covered;

1- Do you ever inject drugs?

2- Do you share needles?

3- In what circumstances do you use drugs? And with whom?

4- Were you tested for diseases that could be transmitted by injecting drugs?

5- Do you ever get admitted to the hospital because of intoxication/overdose?

6- Have you had any legal problems because of drug use?

7- Did drug use cause you any financial or relationship problems? Did it affect your

work?

8- How do you feel about your drug use?

9- Have you ever tried to quit? If yes; tell me more about it; when, for how long, if

medical help was provided and why did he/she fall off the wagon?

10- Are you ready to change your behavior regarding drug use?

8. Sexual history

If relevant to the case, a detailed non-judgmental sexual history is warranted. Assure the

patient that all the information he/she provides is completely confidential except if there was

an impending harm to the patient or others. Questions to cover;

1- Are you sexually active?

2- With men/women or both?

3- How many partners do you have now?

4- How many partners did you have in the last 3 months? 6 months? One year?

5- Do you have decreased libido?

6- Do you have problems with erection or ejaculation? ( for men obviously)

7- Are you satisfied in your sexual relationship with your partner?

8- Do you use contraception?

9- What type of contraception do you use? Do you use condoms? How often do you use

them?

Page 36: CANADIAN IMGS GUIDE TO OSCE AND PRACTICE · in an OSCE scenario You can use this study guide to prepare for the NAC OSCE and MCCQE2. The content was reviewed for the most recent updates;

36

10- Have you ever had any sexually transmitted disease? What, when and was it

treated?

11- Do you know if any of your partners ever had a sexually transmitted infection?

12- Do you have a vaginal/urethral discharge? Burning or itchiness?

13- History of sexual assault or abuse

The extent of questions to cover in history depends on the case. If, for example you are

interviewing a newly diagnosed HIV patient, you need to focus on questions related to sexually

transmitted infections. If the patient main issue was erectile dysfunction, then other aspects of

the sexual history in addition to detailed social and relationship history will be more relevant.

Cases detailing these points are covered in chapter 5.

Special Populations: History

When taking history of the following patient populations, you need to cover the general history

first, and then go over special areas pertaining to a particular subset of patients.

Obstetrics and Gynecology history

Gestational history

1- Total number of pregnancies

2- number of living children

3- number of abortions

4- number of term and premature deliveries

5- Any fertility problems and treatment.

History of current pregnancy

1- When was your last menstrual period?

2- Was it similar to your usual periods in terms of number of days and amount of

bleeding?

3- Were you on contraception when you got pregnant?

Page 37: CANADIAN IMGS GUIDE TO OSCE AND PRACTICE · in an OSCE scenario You can use this study guide to prepare for the NAC OSCE and MCCQE2. The content was reviewed for the most recent updates;

37

4- Do you have morning sickness? If vomiting ask how often and how much and if

she has lost weight also make sure she is keeping up her fluid intake

5- Do you have breast tenderness?

6- Do you have vaginal bleeding or discharge?

7- Do you have abdominal pain or dysuria?

8- Is there any change to your bowel habit?

9- Did you take folic acid before getting pregnant?

10- Are you taking prenatal vitamins?

11- Are you taking any medications or herbs?

12- Do you smoke, drink or do drugs?

13- What do you eat on a typical day? Do you exercise?

14- Do you have regular prenatal care?

15- Did you have any complications during this pregnancy such as diabetes,

hypertension or infection?

16- Is the baby moving (if GA is appropriate)?

17- What is your blood group? What’s the father’s blood group? (in many cases the

patient doesn’t know their blood group and you will order it anyway, but this is

to show the examiner that you are thinking about this very important point)

History of past pregnancies:

Including the date, mode of delivery and the outcome, gestational age at birth, birth

weight, need for resuscitation or neonatal intensive care admission and any

complications during past pregnancies or deliveries.

Bleeding history;

Includes history of menarche, menopause, regularity and heaviness of menstruation,

Painful cycles and severity of pain. Also includes abnormal bleeding including

prolonged heavy cycles, intermenstrual bleeding and post coital bleeding.

Premenstrual symptoms;

Including anxiety, nervousness, food cravings, bloating, headache, sleep

disturbances, breast tenderness and changes in libido.

Page 38: CANADIAN IMGS GUIDE TO OSCE AND PRACTICE · in an OSCE scenario You can use this study guide to prepare for the NAC OSCE and MCCQE2. The content was reviewed for the most recent updates;

38

Symptoms of menopause;

Includes hot flashes, sweating, sleep disturbances and vaginal dryness.

Previous use of contraception:

What was used, when and for how long? Were there any side effects?

Previous mammogram;

When and what was the result?

Previous Pap test:

When? What was the result? Any intervention needed?

History of infections

Previous sexually transmitted infection, urinary tract infections, vaginal infections

and their treatment.

Specific symptoms;

Vaginal dryness, itchiness, discharge, dyspareunia, urinary frequency, dysuria and

incontinence.

Sexual history

If relevant to the case obtain the sexual history (detailed in section 1.2.9)

Pediatrics history

In most pediatrics cases the history is obtained from the mother, father or another care giver.

Sometimes the history may be obtained over the phone (QE2). In some cases a child will

accompany his care giver, in this case make sure you manage time wisely and involve the child

as appropriate based on age and the complaint.

Page 39: CANADIAN IMGS GUIDE TO OSCE AND PRACTICE · in an OSCE scenario You can use this study guide to prepare for the NAC OSCE and MCCQE2. The content was reviewed for the most recent updates;

39

At the beginning of each SP encounter, establish who is telling the story and their relationship

with the child. Also ask who is the guardian, this is particularly important in cases of child

abuse.

Rourke record is a very good resource for age specific pediatric history, it is used by many

health care providers in Canada;

http://www.rourkebabyrecord.ca/

In addition to the general history, the following is a general outline of what you need to cover,

keep in mind your patient’s age and customize your history accordingly; ;

Perinatal history;

Especially for infants and young children. It is less significant in teenagers unless

related to the chief complaint. Includes;

1- Prenatal history;

Mothers obstetric history including the number of pregnancies and abortions

This pregnancy; was it planned? What was the gestational age at birth? Was it

single or multiple gestations? What is the age and blood group of the mother

and father? Was there a regular prenatal care? Was there any complication

during pregnancy (for example; infection, vaginal bleeding, diabetes,

hypertension, accidents and trauma)? Did the mother use any medications

during pregnancy including prescription and over the counter medications,

vitamins and herbs? Did one or both parents smoke, drink, or use recreational

drugs during pregnancy or after delivery? Was the mother exposed to X-ray

while pregnant?

2- Natal history (History of delivery);

At what gestational age was the baby delivered? Where did the delivery take

place? Was labor spontaneous or induced? What was the mode of delivery

(Normal vaginal, forceps, vacuum or cesarean)? What was the duration of

membranes rupture? Was the water clear or stained with meconium (baby’s

poop)? Did the mother have vaginal bleeding or fever? Was there fetal distress at

any point during labor?

3- Post natal history;

Page 40: CANADIAN IMGS GUIDE TO OSCE AND PRACTICE · in an OSCE scenario You can use this study guide to prepare for the NAC OSCE and MCCQE2. The content was reviewed for the most recent updates;

40

What was the birth weight? Did the baby cry immediately? What was the APGAR

score (some mothers would know, but frame your question carefully so the

mother doesn’t feel intimidated; there is a score that is calculated when the baby

is born called APGAR score, did your doctor tell you what was your child’s

score?)? Was the baby admitted to the neonatal intensive care unit? When was

the baby discharged from the hospital? Did he have breathing or feeding

difficulties? Did he have fever, jaundice or seizures?

Feeding history

Was the baby given breast or formula milk or both? If breast; how often and for how long?

Where there any problems with breast feeding? For formula; what is the type? How is it

prepared? How much and how often is the baby fed? Does the baby spit up or vomit often? If

yes what is the amount, color, frequency and content? When was solid food introduced? What

is the child currently eating? Is he/she given any vitamins or supplements? Also ask about the

child’s urine and stool frequency and amounts and if there are any problems (dysuria, bloody

stools or melena, bloody urine, etc...)

Vaccination History

What vaccines was the baby given? When? Did he/she have any reactions or side effects to

vaccines namely fever, rash, seizures or prolonged crying? If yes, then what was done?

Growth and development

Page 41: CANADIAN IMGS GUIDE TO OSCE AND PRACTICE · in an OSCE scenario You can use this study guide to prepare for the NAC OSCE and MCCQE2. The content was reviewed for the most recent updates;

41

What is the child’s weight and height? Then you need to go over the developmental milestones

by history, tailor your questions to the patient’s age, for example; inquire about sitting in a

seven months old, it’s too early to ask about walking.

Review RBR found on Bourke’s record for age appropriate questions;

http://www.rourkebabyrecord.ca/downloads.asp

Developmental milestones general inquiries;

Gross Motor; Holding the head up, rolling from side to side and from prone to supine and supine to

prone. Sitting with and without support, standing with and without support, walking, running,, jumping,

going up and down stairs and riding a tricycle or a bicycle.

Fine motor; Ability to hold the a crayon, scribble, draw a triangle, rectangle, circle and square

Speech and language; Does he point out his/her needs? How many words does he/she say? Can he/she

use full sentences? Can he/she tell a story?

Social; Does the baby make eye contact? Smile? Laugh? Recognize parents? Does he/she become overly

anxious around strangers? Can he/she wave bye bye? Play pee-ka-boo? Help with

buttoning/unbuttoning clothes, put on shoes, play with other children?

Family history

Ask about consanguinity and congenital anomalies in the family. Inquire about specific diseases

pertinent to the case

Social history

It differs in pediatrics from adults. Social history for adolescents will be detailed in the

adolescent history later in this chapter. Ask about;

1- Age of the building, space, occupants, pets and home environment.

2- Who cares for the child?

3- Any stress or violence at home?

4- Any major events like death, accidents or divorce?

5- What are the child’s interests and activities?

6- Does the child go to a day care?

Page 42: CANADIAN IMGS GUIDE TO OSCE AND PRACTICE · in an OSCE scenario You can use this study guide to prepare for the NAC OSCE and MCCQE2. The content was reviewed for the most recent updates;

42

7- How is the child’s school performance?

8- What is the parent’s occupation? Hours?

9- Are there any financial issues?

10- Does one or both parents smoke, drink or use recreational drugs?

Psychiatry history

There are usually one or two psychiatric cases in the exam. In some cases the patient maybe

depressed or angry which makes history taking challenging. The standardized patients are well

trained and the exam is designed with the purpose of testing how you would react in these

situations. Staying professional in these scenarios and asking questions in an organized non-

judgmental way will get you through.

Psychiatric history is comprised of the following;

History of the four basic psychiatric illnesses; depression, mania, psychosis and

anxiety

Assess suicidal and homicidal ideation (You may fail the station if you forget this

point). Ask a direct question: Have you ever thought about hurting yourself or

ending your life? If yes, then ask about details of previous attempts; when and what

did the patient do exactly, try to determine if it was a serious attempt or a cry for

help. The method used is useful in discerning seriousness; if the patient uses drugs

and sends a letter to someone at the same time; he/she is probably seeking

attention. If the patient tried to hang or shoot him/herself, this is more serious.

Attempting to commit suicide once is a predictor that the patient will do it again.

Next, ask the patient if he/she is suicidal now and if he/she has a plan. If yes, the

next step is to admit the patient to the hospital with or without his/her agreement

to make sure he/she is safe. The same goes for homicidal ideation, if the patient says

he/she wants to hurt a specific person, you have a duty to warn the victim if you can

, notify appropriate authorities and admit the patient to hospital (voluntarily or

involuntarily).

Assess insight; does the patient think his mood or hallucinations for example are a

problem?

Rule out secondary cause for the psychiatric problem like an organic disease or

drugs

Look for a precipitating factor

Take a proper social history exploring relationships/work problems, functional

decline , substance abuse

Determine if the patient needs admission via Form 1

Page 43: CANADIAN IMGS GUIDE TO OSCE AND PRACTICE · in an OSCE scenario You can use this study guide to prepare for the NAC OSCE and MCCQE2. The content was reviewed for the most recent updates;

43

You may need to obtain collateral history if the patient seems unreliable

Next are the specific questions you need to ask for each of the 4 basic psychiatric problems;

1- Depression;

Screen your patient by asking if he/she feels depressed or has suicidal thoughts, if yes

go into details. You can use the popular acronym;

M SIGE CAPS;

* Mood; Do you feel depressed? For how long?

*Sleep; Did your sleep patterns change?

*Interest: Have you lost interest in activities you used to enjoy?

*Guilt: Do you feel guilty or worthless?

*Energy: Do you feel you have less energy than usual?

*Concentration and memory: Are you able to concentrate? Do you feel more

forgetful?

*Psychomotor agitation or retardation: Do you feel you are moving more or less

than usual?

*Thoughts of death/suicide: Do you have thoughts of killing yourself? Do you have a

plan?

2- Mania;

*Do you feel you have more energy than what you can control?

* Did you engage in risky behaviors or investments lately?

* Do you need less sleep?

* Do you feel your self-esteem is inflated?

3- Anxiety:

*Do you feel you worry excessively?

*Is it general worry or do you have something specific in mind?

*Do you have fear or phobias?

* Are you obsessed about anything?

*Do you have any compulsive behavior? Ie:

4- Psychosis;

*Hallucinations: Do you see or hear things other people don’t see or hear?

* Delusions: Do you hold beliefs other people think are odd? If so, tell me about

them.

* Have you been taking care of yourself and look recently?

Page 44: CANADIAN IMGS GUIDE TO OSCE AND PRACTICE · in an OSCE scenario You can use this study guide to prepare for the NAC OSCE and MCCQE2. The content was reviewed for the most recent updates;

44

* Are you becoming more socially withdrawn?

Adolescent (teenager) history

Before even taking the history from a teenager, emphasize confidentiality. Tell them that

everything they share with you is strictly confidential, and that you are not going to tell their

parents or teachers. You need to let them know that the only time when you are going to tell

anybody is if someone else’s health or safety is affected as with planning homicide or in some

communicable diseases. This way you gain their trust and they are more likely to share

information with you. If the adolescent is accompanied by one or both parents, obtain some of

the history in the parents’ presence then ask them politely to leave the room.

Psychosocial history and identifying risky behaviors are the added areas of adolescent history.

There is a nice acronym commonly used in Canada to cover these areas;

HEEADSS

Home: With whom does he/she live? How is his/her relationship with parents/siblings?

Is there any stress or abuse at home? What does the home environment look like?

Education: which school does he/she attend? What grade? How is his/her school

performance? Is there any bullying/abuse at school?

Eating: here you screen for eating disorders which are more common in females. Ask first only

question number 1 to screen quickly. If negative – move on. If you hear a red flag in the

answer then continue with the other questions.

Eating disturbances can be a sign of depression as well. Ask about;

1- Number, content and frequency of meals

2- Do you snack?

3- Do you take any vitamins/supplements?

4- What is your current weight? Did you lose or gain weight? Was it intentional?

5- Are you happy with your weight and look?

6- Have you been on diet? How many diets over the last year?

7- Do you binge eating?

8- Do you try to make yourself sick and throw up to avoid gaining weight?

9- Do you take laxatives?

10- Do you exercise? How often and how much?

Page 45: CANADIAN IMGS GUIDE TO OSCE AND PRACTICE · in an OSCE scenario You can use this study guide to prepare for the NAC OSCE and MCCQE2. The content was reviewed for the most recent updates;

45

Activities: What activities do you do, including work, sports, exercise, art, etc…

Alcohol, Smoking and Drugs (if not previously asked in the social history) If positive

follow up with quantifying use. Then, was he/she ever in trouble because of substance abuse

Sex; Cover the sexual history and educate the teenager about safe sex practices.

Suicide and mood: Ask directly about any suicidal or homicidal thoughts and screen for

depression;

When obtaining a family history from an adolescent inquire about heart attacks, sudden death,

dyslipidemia, any genetic disorders and if there is a family history of substance or alcohol abuse

or a psychiatric illness.

References

1- Roshan M, Rao AP. "A study on relative contributions of the history, physical examination

and investigations in making medical diagnosis." J Assoc Physicians India. 48, no. 8

(2000): 771-5.

2- Peterson MC, Holbrook JH, Von Hales D, Smith NL, Staker LV. "Contributions of the history,

physical examination, and laboratory investigation in making medical diagnoses." West J Med.

156, no. 2 (1992): 163-5.

3- Simel, David L. The Rational Clinical Examination Evidence-based Clinical Diagnosis. New

York: McGraw-Hill Medical, 2009.

4- Macleod, John. Macleod's Clinical Examination. 12th ed. Edinburgh: Churchill

Livingstone/Elsevier, 2009.

5- Angela LaRossa, Developmental-behavioral surveillance and screening in primary care, In:

UpToDate, Topic 615 Version 20.0 , UpToDate, Waltham, MA, accessed August 20/2014,

http://www.uptodate.com/contents/developmental-behavioral-surveillance-and-screening-in-primary-

care?source=search_result&search=developmental+milestones&selectedTitle=1~92

Page 46: CANADIAN IMGS GUIDE TO OSCE AND PRACTICE · in an OSCE scenario You can use this study guide to prepare for the NAC OSCE and MCCQE2. The content was reviewed for the most recent updates;

46

Chapter 3

Physical Examination Edited by; Dr. Jay Shavadia Dr. Erin Toor Dr. M. Wasif Hussain Dr. Sadik Salman Dr. Stephanie Keeling

Page 47: CANADIAN IMGS GUIDE TO OSCE AND PRACTICE · in an OSCE scenario You can use this study guide to prepare for the NAC OSCE and MCCQE2. The content was reviewed for the most recent updates;

47

Introduction

IMGs represent a diverse group of physicians; some of them are specialists in their respective

fields and know way more than what is needed to pass the Canadian entry level exams. This

particular group of IMGs may wonder how far should they go when doing physical examination;

should they state that every step is evidence based and quote the likelihood ratio? Should they

do every single maneuver? Other IMGs may be wondering about what organ systems are tested

in OSCE. This chapter will go over physical examination (P.E) in detail.

As mentioned earlier, Canadian OSCEs that IMGs are required to pass to become eligible to

apply for residency positions in Canada evaluate the knowledge of the generalist not the

specialist. For example; you should be able to do general respiratory physical exam, but not

necessarily elicit all the specific findings in COPD. You don’t need to back up your maneuvers

with evidence based data either. Unlike in some other countries, the exam covers all organ

systems and each one of them is detailed next.

One of the special aspects of physical examination in Canadian OSCEs is that you need to

verbalize it, i.e; think out load. For example when you are listening to the heart you need to say

what you are looking for while listening; I’m now listening to S1, S2, etc… Which means you

can’t really focus on eliciting findings, instead if you say: looking for murmurs, the examiner will

tell you if there is a murmur. However, in some cases, especially with inspection the patient

may have real findings and in this case describe what you see. Now comes the question; how

much should I say during P.E? There is no right or wrong. Ideally you should say what you are

doing, what you are looking for and your findings. This chapter includes suggested verbalizing of

PE. Nevertheless feel free to use your own words and say less or more as the time permits.

Be professional when performing physical exam; Ask for permission at the start and only once

then proceed, explain to the patient what you are doing and why in an easy to understand

language, don’t use medical terms. One of the IMGs I studied with mentioned to me that his

friend used to ask patients; can I examine your cardiovascular system? Instead simply say; can I

examine your heart? Wash your hands before and after examining the patient (It is enough to

rub your hands with sanitizer). Use respectful draping keeping in mind the patient comfort and

privacy; don’t repeat painful maneuvers, and make sure the patient is warm and your hands are

warm!

As with history, reading this chapter is not enough. You need to PRACTICE. It is even more

important for physical examination, as you need to complete most exams in five minutes or

Page 48: CANADIAN IMGS GUIDE TO OSCE AND PRACTICE · in an OSCE scenario You can use this study guide to prepare for the NAC OSCE and MCCQE2. The content was reviewed for the most recent updates;

48

less. I can’t stress enough the importance of working on your own organized approach to

physical examination. Following the traditional sequence of; Inspection, palpation, percussion

and auscultation is a good way. However, this sequence may be altered when doing special

exams, for example; auscultation before palpation and percussion in abdominal exam. Or may

be modified as in musculoskeletal exam where you would do; Inspection, palpation, range of

movement and special tests. Both way be organized, and take into account your patient’s

comfort – i.e.: do not have the patient lying down, then sitting up, then lying down again. Make

sure you master all maneuvers, the examiner will be looking on how well you do each step, and

you will get zero if you verbalize correctly but do the procedure in a wrong way.

You should tailor your P.E exam to sort out the patient’s problem, this is the focused part of the

PE. In any case involving ER always make sure the patient is stable before starting your P.E. by

asking for vitals if not given on the door sign. Comment on the vitals given i.e.: patient’s vitals

are normal or this patient is tachycardic, etc. This is especially relevant in management cases

where you will not be asked to do a particular physical exam. Solid knowledge and practicing

the cases in this book will help you perform high yield P.E maneuvers.

The objective of this book is not to teach you how to do P.E or the differential of positive and

negative finding , but rather to point out what aspects of the physical exam are more relevant

in Canadian OSCEs, and how you are expected to perform.

This chapter details the physical exam steps then outlines the suggested verbalizations in boxes.

Conversation directed to the patient is in italic. I’m assuming that there are no findings, make

sure to practice and master the procedures so that you are able to elicit findings if any and

report them, in many cases the examiner will tell you if there are findings.

Key Points

- Verbalize your physical exam including what you are doing, what you are looking for

and your findings

- You don’t need to talk about the evidence behind your maneuvers

- Make sure you do the maneuvers correctly

- Expect to be tested on any organ system

- Develop your own organized approach following the traditional sequence of;

Inspection, palpation, percussion and auscultation as appropriate

- Always take permission, wash your hands and be aware of patient comfort and privacy

- Drape your patient in a respectful way; Keep body parts you are not examining

covered.

- Communicate with the patient in an easy to understand language; Avoid medical

jargon

Page 49: CANADIAN IMGS GUIDE TO OSCE AND PRACTICE · in an OSCE scenario You can use this study guide to prepare for the NAC OSCE and MCCQE2. The content was reviewed for the most recent updates;

49

- Listen to the examiner, they will give you pertinent findings or ask you to move on – if

they ask you to move on, do so.

- Manage your time and priorities wisely

- Practice Practice Practice

Cardiovascular System Examination (CVS P.E)

Edited by; Dr. Jay Shavadia

Includes:

1- Peripheral and central pulse, Blood pressure

2- Jugular Venous Pressure (JVP)

3- Precordium examination

4- General Examination related to CVS exam

5- Peripheral Vascular exam

In most cases you need to do 1, 2, 3 and 4. The question will usually be specific about

peripheral vascular exam. Always make sure the patient has stable vitals before starting CVS

physical exam; Take a quick look at the patient looking for pain, pallor, increased work of

breathing, and mention if any medications / oxygen is present at the bedside. Go over ABCs

and ask for the vital signs if not provided in the question stem.

Peripheral and central pulse

Ask permission, wash your hands and tell the patient that you are going to take their pulse.

Count the rate and note the regularity. An irregular pulse could suggest atrial fibrillation, or

premature atrial / ventricular complexes. With the patients arm resting on your right arm, feel

the brachial pulse with your right thumb and the radial pulse with your left index finger for a

brachio-radial delay. Next feel the left or right carotid pulse using your thumb, and note the

volume and upstroke.

Page 50: CANADIAN IMGS GUIDE TO OSCE AND PRACTICE · in an OSCE scenario You can use this study guide to prepare for the NAC OSCE and MCCQE2. The content was reviewed for the most recent updates;

50

I will examine the CVS by starting with ABC; the patient looks comfortable, breathing

spontaneously and speaking in full sentences. I’m feeling the radial pulse and it is regular 80

beats/min. There is no brachio-radial delay. The vital signs were just reported by the nurse and

are normal. I would like to get them checked again in X minutes (Depends on the case; In

emergency situations check the V.S every 10-15 minutes or put the patient on the monitor; if

you see a stable patient in your clinic you don’t need to repeat the V.S). The central pulse is of

normal volume and upstroke.

Precordium physical exam

Ask permission, wash your hands, and respectfully drape the patient by lowering his/her gown

to his/her waist. Keep the bra on in female patients, and drape the chest once the inspection of

the precordium is complete. Do the examination with the patient lying down. Speak up and

explain what you are doing, what you are looking for and your findings

- Inspection; Look at the shape of the chest for any deformities or asymmetrical

movement during respiration. Note any dilated veins, visible pulsations, scars

(sternotomy, lateral thoracotomy, pacemaker) or other skin changes. Inspect the

infraclavicular areas for pacemakers / defibrillators. Also inspect the epigastrium for

pulsations suggestive of abdominal aortic aneurysm or right ventricular enlargement.

Can I examine your heart? The SP will say yes. Wash your hands and respectfully drape the

patient. I start the precordial examination by looking for any deformities or asymmetrical chest

expansion with respiration and I don’t appreciate any. There are no dilated veins, visible

pulsations, scars or any other skin changes (you need to list findings if any)

Note; if you only say I’m inspecting the epigastrium for any abnormalities, the examiner will

consider this unsatisfactory.

- Palpation; you are palpating for four things; First: The Apex beat (defined as the most

lateral and inferior impulse NOT the point of maximal impulse), normally located in

Page 51: CANADIAN IMGS GUIDE TO OSCE AND PRACTICE · in an OSCE scenario You can use this study guide to prepare for the NAC OSCE and MCCQE2. The content was reviewed for the most recent updates;

51

the fifth intercostal space, midclavicular line. Feel for the character of the apical

impulse. It could be normal or sustained. Second; palpate for heaves (Forceful

contractions, feels as if your hand is being lifted). Use the palm of your hand and

palpate over the right and left parasternal borders and over the apex. Third; thrills

(palpable murmurs) and palpable heart sounds. Use the tips of your fingers to feel for

thrills over the 4 major valvular areas; Aortic, Pulmonic, Tricuspid, and Mitral. A thrill

feels like a vibration. A palpable second heart sound may be appreciated over the left

second interspace in pulmonary hypertension. Fourth; palpate the epigastrium for

pulsations suggestive of either right ventricular hypertrophy or abdominal aortic

aneurysm. With your hand in the epigastrium, the pulsation of a AAA ‘comes from

below’ while that of the right ventricle ‘comes from the top, under the sternum’.

I will move now to palpation, Do you have any pain?. If the patient says yes start palpation

away from the painful area. I’m feeling for the apex beat, I notice it is located as expected in the

fifth intercostal space midclavicular line. The apical impulse is normal. Next, I’m feeling for any

heaves, thrills and palpable heart sounds and don’t appreciate any. Finally I’m palpating the

epigastrium for pulsations, and there isn’t any

- Percussion; Skip in precordial exam

- Auscultation; Auscultate over all four valvular areas, using the bell then the diaphragm

of the stethoscope in three positions; lying flat, leaning to the left and sitting up. Feel

the carotid pulse while listening (S1 heard before the pulse, S2 after). Start with the

patient flat; listen for S1, S2, note if they are normal, loud or soft and if S2 is split, then

listen for any added sounds (mainly S3, S4, rubs) or any murmurs. Specify if the

murmur you are hearing is systolic or diastolic. Then get the patient to lean towards

his/her left side and listen for murmurs of the mitral valve in end expiration. After

that, get the patient to sit up lean forwards, take in a deep breath then exhale and

listen for murmurs of the aortic valve. If the examiner says that there is a murmur or if

you hear a murmur then you need to fully describe it; location, type (systolic vs.

diastolic), radiation and intensity.

It doesn’t matter which order you listen in, as long as you have an organized approach. I listen

to the base first, starting with the aortic then pulmonary then tricuspid then mitral. Some

people start at the apex and move upwards.

Page 52: CANADIAN IMGS GUIDE TO OSCE AND PRACTICE · in an OSCE scenario You can use this study guide to prepare for the NAC OSCE and MCCQE2. The content was reviewed for the most recent updates;

52

I’m now auscultating the heart; I’m listening over the aortic, pulmonic, tricuspid and mitral

areas. Listening for S1, S2, there is no S3, S4, no rubs and no murmurs Can you lean to the left

side please? I’m listening again over the mitral area. Can you sit up for me, lean forward, take a

deep breath in- out and hold your breath. I’m listening for aortic valve murmurs.

Helpful info;

- Mitral regurgitant murmurs have a ‘blowing quality’ radiate to the axilla, and

accentuate on end expiration

- Mitral stenosis murmurs are low pitch, ‘rumbling’; may be accentuated by asking the

patient to exercise (lean to- and fro- a few times)

- Aortic regurgitant murmurs are high pitched, caused by turbulence of blood through

the incompetent aortic valve and radiate to the left lower sternal border. Best heard

with the patient leaning forward.

- Aortic stenosis murmur is harsh, and radiates to the root of the neck, and carotids.

- Systolic murmurs; Aortic and pulmonic stenosis (ejection systolic / crescendo-

decrescendo) mitral and tricuspid regurgitation (holosystolic i.e begin with the first

heart sound, hence S1 will be soft or absent)

- Diastolic murmurs; Aortic and pulmonic regurgitation (high-pitched), mitral and

tricuspid stenosis (low pitched)

Intensity (grading) of murmurs;

“I – is a murmur barely audible with stethoscope. It is soft, heard intermittently, always with

concentration and never immediately.

II – is a murmur that is low, but usually audible. It is soft, audible immediately and with every

beat.

III – is a murmur of medium intensity without a thrill. It is easily audible, and relatively loud.

Page 53: CANADIAN IMGS GUIDE TO OSCE AND PRACTICE · in an OSCE scenario You can use this study guide to prepare for the NAC OSCE and MCCQE2. The content was reviewed for the most recent updates;

53

IV – is a murmur, which is of medium intensity with a thrill, is relatively loud, and is associated

with a palpable thrill.

V – is the loudest murmur heard with stethoscope on chest. It is loud enough to be heard by

placing edge of the diaphragm of stethoscope over the patient’s chest. It is associated with a

palpable thrill.

VI – is a murmur heard with stethoscope off the chest. It is so loud it can be heard even when

the stethoscope is not in contact with the chest and is held slightly above the surface of the

chest. It is associated with a palpable thrill.” (1)

Jugular venous Pulse (JVP) examination;

The JVP is a very important P.E skill. JVP height reflects the pressure in the right atrium. Refer to

figure 3-2 for JVP anatomy.

Start your P.E by asking permission, then wash your hands and position the patient supine with

the head of the bed elevated at 45 degrees. Place a pillow under the patients head to relax the

neck muscles. Appropriately drape the patient by lowering the gown a little so that the root of

the neck is visible. Do the examination by standing to the right side of the bed. If the top of the

JVP is not visible at this 45-degree position, the head of the bed should be altered up or down

and the JVP reexamined.

- Inspection; look for the JVP between the two heads of the sternocleidomastoid

muscle, notice the highest point and measure its distance from the sternal angle (use

the ruler to draw a perpendicular line from the sternal angle, then your pen to mark

the intersection between the highest point of JVP and the ruler). According to

traditional teaching the normal jugular venous pressure is 6-9cmH₂O (3-4 cm from the

sternal angle + 5cm the distance between the sternal angle and the right atrium).

Traditionally JVP of 4 cm from sternal angle or more is considered elevated. However,

according to the most recent JAMA evidence based clinical diagnoses article JVP of 3

cm or more in any patient position is considered elevated, as most recent data suggest

that physicians often underestimate the JVP.

- Recognize the wave form; Normal JVP is double impulse. Follow the link below for

description of the waves;

http://www.medinterestgroup.com/portfolio-items/jvp-normal-wave-form/

Abnormalities of the wave form will be mentioned below.

Page 54: CANADIAN IMGS GUIDE TO OSCE AND PRACTICE · in an OSCE scenario You can use this study guide to prepare for the NAC OSCE and MCCQE2. The content was reviewed for the most recent updates;

54

- Differentiate the JVP from Carotid pulse;

1- Change with position; JVP decreases when the patient sits up and increase when he

lies flat, the carotid pulse doesn’t change with position.

2- Change with respiration; if there is no pathology JVP decrease with inspiration as the

intrathoracic pressure decreases. The carotid doesn’t change with respiration.

3- Waveform; normal JVP has double waves. The carotid has single wave.

Abnormalities of the JVP waveform are listed below.

4- Occlusion of the blood vessel; the JVP disappears with pressure at the base of the

neck, while the carotid pulsation remains persistent.

5- Palpability; the carotid pulse is always palpable; the JVP is not palpable in most

cases.

6- Abdominojugular reflux; formerly known as the Hepatojugular reflux. Has 2

purposes; differentiating the Carotid from the JVP, and is one of the signs of

elevated left ventricular filling pressures (suggestive of left heart failure). Ask the

patient if he/she has abdominal pain and tell him that you will apply pressure to his

abdomen. Inflate the blood pressure cuff and place it on the abdomen (You can

place it anywhere on the abdomen and not necessarily over the liver; thus the term

abdominojugular; my personal preference is to put over the Right upper quadrant or

epigastrium) with your right hand apply pressure over the cuff till the blood pressure

monitor goes up to 30mmHg. Sustain the pressure for 10 seconds while looking for

elevation in the JVP. Normal JVP increase a bit and goes down quickly. A positive

sign in heart failure is sustained elevation of at least 3 cm for 10 seconds. The

Carotid pulse is not affected by pressure over the abdomen.

Can I examine your neck veins? Wash your hands, I will lift the head of the bed a little bit, let me

know if you are uncomfortable. Lower the gown so that the root of the neck and both clavicles

are visible and ask the patient to turn the head a little to the left. I’m looking for the JVP

between the 2 heads of the sternocleidomastoid, and I see a double impulse here (point with

your finger then measure it). To make sure this is the JVP and not the carotid I will do some

maneuvers. Can you slowly take in a breath for me? I notice that it is decreasing with

inspiration as expected. Apply some pressure at the vein origin at the root of the neck, and I

notice that unlike the carotid it is occludable. Feel it, and it is not palpable in his case unlike the

Page 55: CANADIAN IMGS GUIDE TO OSCE AND PRACTICE · in an OSCE scenario You can use this study guide to prepare for the NAC OSCE and MCCQE2. The content was reviewed for the most recent updates;

55

carotid which is always palpable. Warn the SP; I’m going to lower the bed a little bit? And lower

the bed, I notice it went up, unlike the carotid which doesn’t change with position. And as I

mentioned earlier it is double impulse while the carotid is single. I also notice that the

waveform is normal, no Cannon waves or CV waves. Lastly I will do the abdominojugular reflux;

do you have pain in your tummy? I’m going to press a bit on your tummy let me know if it hurts.

Inflate the blood pressure cuff, put it over he epigastrium or right upper quadrant. Press on the

blood pressure cuff till the BP monitor goes up to 30mmHg. I’m doing the abdominojugular

reflux by applying 30mmHg pressure over the abdomen and looking at the JVP, an elevation of

at least 4 centimeters that is sustained for 10 seconds is seen in left heart failure. In this case

the JVP went up briefly then immediately went down, so the abdominojugular reflux is

negative. Of note is that the Carotid pulse is not affected by pressure on the abdomen

Figure 3-1; JVP anatomy (2) Note that you are examining the internal not external jugular

Helpful info

Abnormalities of the JVP

Page 56: CANADIAN IMGS GUIDE TO OSCE AND PRACTICE · in an OSCE scenario You can use this study guide to prepare for the NAC OSCE and MCCQE2. The content was reviewed for the most recent updates;

56

1- Kussmauls sign: Is the paradoxical increase of JVP with inspiration seen in restrictive

pericarditis, constrictive and restrictive cardiomyopathy

2- Single impulse; seen in atrial fibrillation as the a wave of atrial contraction is lost

3- Giant (cannon a wave); seen in complete heart block where the right atrium

contracts against a closed tricuspid valve

4- CV wave: seen in tricuspid regurgitation

General Exam related to CVS P.E

- Auscultate the bases of the lungs looking for crackles and wheezing seen in Heart

Failure

- Check the lower limbs for edema, and scars of venous graft harvest site for CABG

surgery

- Examine the hands for; nicotine stains, peripheral cyanosis, splinter hemorrhage, Osler

nodes, Janeway lesions and palmar erythema (The last four are seen in Infective

Endocarditis).Note have already checked the radial pulse at the beginning of the

examination? If not do it now, comment on the rate and regularity, compare both

sides

- Examine the face for pallor, central cyanosis, malar flush, corneal arcus, Xanthelasma

- Finally check the liver span and examine the abdomen for ascites (Ascites exam is

covered under abdominal exam)

Can you sit up please? I’m auscultating the lungs for any wheezes, and listening to the bases for

crackles, I don’t appreciate any. I’m looking for lower limbs edema, it’s negative. I’m examining

the hands for nicotine stains, Jane way lesions, splinter hemorrhage palmar erythema,

peripheral cyanosis, all negative. I have already checked the pulse and noticed its 70b/min and

regular. I’m now examining the face for pallor, can you open your mouth please? Central

cyanosis, malar flush, corneal arcus, xanthelasma, and don’t appreciate any. Can you lie down

please? Do you have any pain? I percuss the abdomen checking the liver span. And looking for

ascites (refer to abdominal examination below for the technique)

Page 57: CANADIAN IMGS GUIDE TO OSCE AND PRACTICE · in an OSCE scenario You can use this study guide to prepare for the NAC OSCE and MCCQE2. The content was reviewed for the most recent updates;

57

- Finish the exam by covering the patient (if not already done earlier), and washing your hands.

Peripheral Vascular Exam

Includes P.E of the peripheral arteries and veins. I will detail each one separately. The exam will

be specific; possible questions are; examine the lower limbs for arterial insufficiency, or

examine the lower limbs for deep vein thrombosis (DVT), prepare yourself for more generic

questions like for venous insufficiency. If asked to perform peripheral vascular exam then

merge both by looking for findings of both while doing the systematic approach; inspection,

palpation, auscultation and special maneuvers.

1- Peripheral Arterial System Examination

Ask permission, wash your hands and respectfully drape the patient by exposing the neck, both

upper arms and the legs keeping the sheets in between the legs (keep the gown on and expose

the areas you are examining). In this examination you need to examine the abdomen for

abdominal aortic aneurysm (AAA) and bruits, keep it covered till you reach it. Always compare

both sides.

Make sure you cover the classical 6 Ps;

1- Polar (cold)

2- Pain

3- Pallor

4- Paresthesia

5- Paralysis

6- Pulselessness

- Inspection; Look for muscle atrophy, masses, skin changes mainly pallor, shiny hairless skin,

rash, necrosis, scars and ulcers (arterial ulcers are sharply demarcated and found on the tips of

Page 58: CANADIAN IMGS GUIDE TO OSCE AND PRACTICE · in an OSCE scenario You can use this study guide to prepare for the NAC OSCE and MCCQE2. The content was reviewed for the most recent updates;

58

fingers, toes, on the heel and at the head of the fifth metatarsal). Make sure you inspect the

hands and legs closely looking for nicotine stains. Look between the toes. Notice any redness of

the eyes or joints swelling or deformity, which could suggest vasculitis as the underlying

pathology.

I notice that the patient is not in pain, no skin rash or redness of the eyes. I’m looking into the

patient’s mouth, no central cyanosis. I inspect both upper and lower limbs comparing sides; the

muscles are symmetrical with no atrophy, no pallor, no cyanosis, no necrosis, no ulcers, no

nicotine stains, no joint swelling or deformities, and no loss of hair.

- Palpation; Start by feeling the temperature, compare both sides as well as different spots on

the same limb. Palpate all pulses specifying the exact anatomical location of each (refer to the

box below). Comment on pulse regularity (regular, regularly irregular, irregularly irregular) and

strength. Compare both sides; for radial and brachial pulses you can feel the right and left at

the same time. For carotid pulse listen first, if you don’t hear bruits palpate. Listen for bruits of

the femoral pulse as well. Squeeze the calves for tenderness suggestive of critical ischemia.

Notice that for the sake of time, you can compare the right and left radial pulses, and then

examine one side only, in this case let the examiner know what you are doing by saying; I will

compare both sides but for the sake of time in an exam setting I will focus on the right side for

now.

I’m feeling the temperature of the upper limbs comparing both sides as well as proximal and

distal areas of the arms, and do the same for the lower limbs. I’m going now to feel the pulses;

1- Starting with the radial pulse lateral to Flexor Carpi Radials tendon, I notice

the pulse is regular and strong, I feel the pulse in both arms at the same

time and it is symmetrical

2- I move now to the brachial pulse medial to biceps tendon, I compare both

sides no abnormalities

3- I listen to the Carotids first making sure there are no bruits then I feel each

side at the lateral border of Sternocleidomastoid at the level of the thyroid

cartilage, noting that the pulse is normal. (P.S; Never feel both carotids at

the same time)

Page 59: CANADIAN IMGS GUIDE TO OSCE AND PRACTICE · in an OSCE scenario You can use this study guide to prepare for the NAC OSCE and MCCQE2. The content was reviewed for the most recent updates;

59

4- I move to the femoral pulses; I need to feel the pulse in the upper part of

your leg, this may feel uncomfortable, let me know if you need me to stop. I

feel the femoral pulse at the mid-point of the inguinal ligament. I feel the

other side and compare. I listen for bruits.

5- Then I move to the popliteal pulse, can you bend your knees for me? I feel

the popliteal pulse in the popliteal fossa

6- I’m feeling the posterior tibial pulse 2cm behind and below the medial

malleolus.

7- And finally the dorsalis pedis pulse on the dorsum of the foot lateral to

extensor tendon of the big toe.

- Neural exam; Both sensory and motor;

1- Sensory; test all the lower and upper limbs dermatomes for simple touch using a

cotton ball (can be found on the table in the room). Name which dermatome you are

testing and compare both sides

Follow the link below for dermatomal distribution;

https://www.pinterest.com/pin/487162884665368556/

2- Motor; you don’t have to do it all; for the upper limbs do resisted shoulder abduction

and adduction, elbow flexion and extension. For the lower limbs resisted hip flexion,

knee flexion and extension, ankle dorsiflexion and plantar flexion.

I will now do motor and sensory screening; I will start with fine touch, touch the patient lightly

with the cotton ball on his hand and ask; do you feel that? I want you now to close your eyes

and say yes every time you feel it, and let me know if it doesn’t feel the same on both sides. I will

start with the arms; C4, C5, C6, C7, C8, T1. Now the lower limbs; L1, L2, L3, L4, L5, S1. There is

Page 60: CANADIAN IMGS GUIDE TO OSCE AND PRACTICE · in an OSCE scenario You can use this study guide to prepare for the NAC OSCE and MCCQE2. The content was reviewed for the most recent updates;

60

normal symmetrical fine touch sensation. I want you now to resist me, please push down with

your shoulders, that’s adduction, push up; abduction. Please bend your elbow; elbow flexion

and extension, I will do the other side and compare, the power is 5/5 and symmetrical. Now the

lower limbs; hip abduction, can you bend your knees for me and kick out, knee extension, now

in; knee flexion. Press on my hand as you press on the gas; ankle plantar flexion, now the

opposite direction; dorsiflexion.

- Special tests;

1- Capillary refill; Apply pressure with your thumb and index finger to the distal end of

the thumb and big toe till it becomes pale then release, normal color should

reappear in 2-3 seconds. More than 5 seconds is considered abnormal.

2- Pallor on elevation and rubor (redness) on dependency test; Raise the leg about 60⁰

or until pallor develops, some pallor is normal, marked pallor is seen in arterial

insufficiency. Now ask the patient to sit up and dangle his legs to the side and notice

the color of the legs. In patients with arterial insufficiency the pallor persists for

about 10 seconds and then the legs become very red.

3- Ankle Brachial Index; you will not be able to actually perform this test, because you

need Doppler Ultrasound (US). Nevertheless, it’s a good idea to mention it. The

examiner may ask how you would do it. Measure the blood pressure of the arm,

then measure the blood pressure at the ankle by placing the blood pressure cuff

around the calves. Use the Doppler US instead of feeling the pulse, you can test

either the posterior tibial or dorsalis pedis pulses. Then simply divide the pressure of

the leg over that of the arm; 1 is normal, less than 0.9 is abnormal, and values below

0.5 suggest critical limb threatening ischemia.

4- Allen test; occlude both the radial and ulnar arteries with your thumbs, ask the

patient to open and close his hands till the palm blanches, then release the pressure

Page 61: CANADIAN IMGS GUIDE TO OSCE AND PRACTICE · in an OSCE scenario You can use this study guide to prepare for the NAC OSCE and MCCQE2. The content was reviewed for the most recent updates;

61

over the radial and look for the return of normal skin color. Repeat the steps but this

time release pressure over the ulnar artery. The color normally returns in 10

seconds, if it takes longer then it indicates insufficient collaterals and it is better to

avoid puncturing the radial artery.

I’m checking for capillary refill, in the upper and lower limbs and comparing both sides. I will

now do pallor on elevation rubor on dependency test. I’m going to raise your legs let me know

if it’s painful. I notice that the legs became pale, but not white. Can you sit up and dangle your

legs over the edge of the bed? I notice that the normal color of the limb returned almost

immediately with no excessive redness. The test is negative for arterial insufficiency. I would as

well like to get an ankle brachial pressure index. At this point the examiner may say: move onor

may ask you to describe it. In the latter case go over the steps mentioned above. I will now do

an Allen test, I’m going to press over the arteries, let me know if it’s uncomfortable. Can you

open and close your hand for me? I notice the hand turned pale, I release pressure over the

radial and notice the normal return of the color. I repeat the same steps occluding both

arteries; can you open and close your hand for me? I notice the pallor and release the ulnar side

noting return of normal color.

- Related abdominal exam; you can now cover the limbs, and expose the abdomen by

lifting the gown up and covering the patient with the sheets (only exposing the

abdomen). Inspect the abdomen for visible pulsations then feel the abdominal Aorta

which is normally palpable in thin people. Place your hand on either side of the

abdominal aorta to estimate its width, more than 2.5cm warrants further evaluation

with Ultrasound. Now listen to the abdomen for;

Bruits of the abdominal aorta; listen in the epigastrium

Renal arteries bruits; 5cm above the umbilicus and 3 cm to each side of the

midline.

2- Peripheral Venous System Examination

Page 62: CANADIAN IMGS GUIDE TO OSCE AND PRACTICE · in an OSCE scenario You can use this study guide to prepare for the NAC OSCE and MCCQE2. The content was reviewed for the most recent updates;

62

This section covers P.E for Deep Vein Thrombosis (DVT). I will go over lower limbs DVT P.E

because it is more common. The same principles apply for upper limbs DVT, you need however

to look specifically for venous puncture sites.

Ask permission, wash your hands, and respectfully drape the patient by exposing only the lower

limbs keeping the sheets in between the legs.

- Inspection; Look for redness, swelling, thickened skin, venous ulcers on the medial

side of the leg, dilated superficial veins or discoloration of the skin. Compare both

sides, asymmetrical swelling is a particularly useful sign of DVT, as well as entire leg

swelling. Notice the general appearance and comfort of the patient, check the

respiratory rate and Oxygen saturation to show the examiner you are thinking about

Pulmonary Embolism as a fatal complication of DVT.

Comparing both legs there is no swelling, both sides are symmetrical, no redness, no thickening

of the skin, no ulcers, no dilated superficial veins. The patient appears comfortable breathing

normally. I would like to know the respiratory rate and oxygen saturation please. At this point

the examiner may tell you the values or ask you to go ahead and check the respiratory rate.

- Palpation; compare the temperature of both legs with the dorsum of your hand.

Measure the width of the calf, more than 3cm difference between the two sides is

significant according to JAMA, some other resources quote 2cm (McGee). The width is

measured at a certain distance from a bony prominence; I use the tibial tuberosity, go

10cm down the leg and measure the width. Squeeze the calf for tenderness. Do

Homans sign; Dorsiflex the foot and ask the patient if this causes calf pain.

Check the legs for pitting edema; using your thumb start at the distal end of the leg, press over

the shin, if there is edema walk your way up until the level of edema becomes clear. Feel for

lower limbs pulses.

Page 63: CANADIAN IMGS GUIDE TO OSCE AND PRACTICE · in an OSCE scenario You can use this study guide to prepare for the NAC OSCE and MCCQE2. The content was reviewed for the most recent updates;

63

I’m checking the temperature of both legs and there is no difference between the two. I’m

measuring the width of both legs 10 cm below the tibial tuberosity comparing both sides; there

is 1cm difference which is normal. Do you have pain in the back of your legs? I will feel it, let me

know if it hurts. I check for lower limbs edema and don’t appreciate any. Next, I’m doing

Homan’s sign, dorsiflexing the foot? Do you have any pain? Finally I will check lower limbs

pulses;

1- Dorsalis Pedis lateral to extensor halluces longus tendon.

2- Posterior Tibial 2cm behind and below the medial malleoulus

3- Popliteal pulse in the popliteal fossa

4- Femoral pulse in the mid inguinal point

Page 64: CANADIAN IMGS GUIDE TO OSCE AND PRACTICE · in an OSCE scenario You can use this study guide to prepare for the NAC OSCE and MCCQE2. The content was reviewed for the most recent updates;

64

Respiratory System examination:

Edited by Dr. Erin Toor

Introduce yourself to the patient, ask permission and wash your hands. Do the general

examination that includes examination of hands, head and neck. Then respectfully drape the

patient by exposing the chest; lower the gown to the waist, keep the bra on in female patients.

Speak up and explain what you are doing, what you are looking for and your findings.

- General: Note if there are signs of respiratory distress (Tachypnea, inability to speak in

full sentences, use of accessory muscles of respiration, tripoding, pursed lip breathing,

nasal flaring, paradoxical indrawing of intercostal muscles and central cyanosis).

Examine the hands: look for nicotine staining, peripheral cyanosis, clubbing, feel the

pulse and check for flapping tremor.

- Head and neck: Look at lips and tip of the tongue for central cyanosis. Examine the

throat for congestion and palpate the sinuses for tenderness. Examine for tracheal

deviation, and then ask the patient to take a deep breath and examine for tracheal tug

that is a sign of hyperinflation. Examine the laryngeal height during expiration. Palpate

the lymph nodes of the head and neck.

The patient is speaking full sentences, which suggests open airway and

spontaneous breathing. I would like to check the vital signs (the examiner says

stable, or may give you values). There is no pursing of the lips, no nasal flaring, no

audible wheezes, and no use of accessory muscles of respiration. The patient is not

tripoding

I am examining the hands for nicotine staining, peripheral cyanosis and clubbing

and I don’t appreciate any.

Can you please open your mouth? Looking at the lips and tongue, no central

cyanosis. There is no throat congestion. Please let me know if it is sore, I’m

Page 65: CANADIAN IMGS GUIDE TO OSCE AND PRACTICE · in an OSCE scenario You can use this study guide to prepare for the NAC OSCE and MCCQE2. The content was reviewed for the most recent updates;

65

palpating the sinuses for tenderness which may suggest sinusitis. There is no

tracheal deviation; can you please take a deep breath? No tracheal tug. Can you

please breathe in then out and hold your breath? The laryngeal height is 5 cm. I am

palpating the lymph nodes in the head and neck. Starting with the occipital, post

auricular, pre auricular, submandibular, submental, anterior and posterior groups

of cervical lymph nodes and supraclavicular, infraclavicular lymph nodes. I don’t

feel any enlarged lymph nodes.

1. Examine the chest:

Inspection: Look at the shape of the chest (kyphoscoliosis, pectus excavatum,

carinatum) and symmetry of chest expansion during respiration. Make a note of

scars, dilated veins, visible masses or skin changes.

Palpation: Check for chest expansion (done by tape measurement at the level of

the nipples), palpate the chest for tenderness, deformities, subcutaneous

emphysema.

Percussion: Percuss anteriorly and posteriorly, and over lungs apices. Compare

both sides and note any hyper resonance that may indicate pneumothorax.

Dullness on percussion may indicate consolidation, mass or effusion. Percuss for

diaphragmatic excursion.

Auscultation: Use the bell of the stethoscope and compare both sides. Listen for

symmetry of breath sounds and note if breath sounds are bronchial or vesicular.

Also, note any crackles or wheezes. Ask the patient to say “eee” and listen for

egophony over the same areas. You expect to hear “aaa” over areas of

consolidation.

I am going to examine the chest starting with inspection. Looking anteriorly, posteriorly

and from the sides, there is no barrel chest, no pectus excavatum, no pectus carinatum,

no exaggerated kyphosis and scoliosis. The chest moves symmetrically with respiration.

There are no scars, no dilated veins, no visible masses and no skin changes. On palpation

of the chest, I will demonstrate chest wall expansion. By placing the measuring tape

Page 66: CANADIAN IMGS GUIDE TO OSCE AND PRACTICE · in an OSCE scenario You can use this study guide to prepare for the NAC OSCE and MCCQE2. The content was reviewed for the most recent updates;

66

around the chest at the level of nipples, I will ask the patient:, Can you please take a deep

breath in , hold for few seconds and then out? Chest expansion is about 5cm and

symmetrical. Next, I will do percussion over all lung spaces starting with the apex and

comparing both sides. Then proceeding to the upper, middle and lower lung zones. I do

not appreciate any dullness or hyper resonance. I am percussing now for diaphragmatic

excursion locating the level of the diaphragm, Can you please take a deep breath in and

hold? I’m marking the lower end of the diaphragm, can you please exhale and hold, I am

marking the upper level of the diaphragm. Normal diaphragmatic excursion is between 4-

5cm. I will now compare both sides. (The examiner may ask you to move on, if not you

can say I would do the other side but for the sake of time I will move on unless u want

me to demonstrate it again) I will now auscultate the chest. Can you please breathe in

and out through your mouth each time I place the stethoscope on your chest? I am

listening for breath sounds over the upper, middle and lower lung zones and comparing

side to side. Breath sounds are vesicular and symmetrical on both sides. I do not

appreciate any crackles, wheezes and pleural rubs. Can you please say “eeee”? I’m

listening over the same areas, there is no egophony.

Abdominal Examination

Edited by; Dr. Erin Toor

This section covers general comprehensive abdominal examination. Note that you need to

tailor your physical examination to the suspected pathology, and be even more specific in

emergency situations. This will be fully detailed in respective cases.

Abdominal exam includes;

1- Examination of the abdomen itself

2- General examination related to gastrointestinal pathology

Introduce yourself to the patient and explain what you are going to do. Then, take permission

and wash your hands. Before starting the examination, respectfully drape the patient by

covering him/her with the sheet. Lift the gown up and expose the abdomen from the nipples to

the pubic symphysis ensuring that the pubic area is covered. Do the examination in the

following order; Inspection, auscultation, percussion, palpation.

Page 67: CANADIAN IMGS GUIDE TO OSCE AND PRACTICE · in an OSCE scenario You can use this study guide to prepare for the NAC OSCE and MCCQE2. The content was reviewed for the most recent updates;

67

Inspection: Note the patient’s position; is he/she comfortable or in pain? Then inspect the

abdomen. Look for abdominal symmetry, movement of the abdominal wall with respiration,

scars, striae, dilated veins, Cullen’s sign (ecchymosis around the umbilicus seen in hemorrhagic

pancreatitis), Grey Turner’s sign (flanks ecchymosis seen in hemorrhagic pancreatitis, look at

the back as well), Caput medusa, masses, distention.

I am inspecting the abdomen noting that it is not distended and moving symmetrically with

respiration. There are no obvious masses, striae, surgical scars or dilated veins. There is no

obvious abdominal distention, caput medusa, Cullen’s sign or grey turner’s sign

Auscultation: Auscultate all 4 quadrants for bowel sounds. Listen for bruits in the following

locations:

1- Epigastrium: for Abdominal Aorta bruits

2- 2 cm above the umbilicus and 2 cm to either side of the midline for renal arteries

bruits

3- Over the liver in the right upper quadrant (Hepatoma, hepatic hemangioma,

arteriovenous malformations may have bruits)

I am listening for bowel sounds over the 4 abdominal quadrants and notice they are present

and within normal limits. I’m now listening for aortic bruits over the epigastric area. For renal

bruits, auscultating 2 cm above and to either side of the midline. There are no abdominal aortic

or renal bruits. Listening over the liver, there are no bruits.

Percussion:

1- General percussion over all the 4 quadrants looking for tenderness and dullness.

Note that percussion tenderness is a sign of peritoneal irritation (it is important to

show the examiner you are looking for signs of peritoneal irritation), always look at

the patient’s face for discomfort when you percuss.

Page 68: CANADIAN IMGS GUIDE TO OSCE AND PRACTICE · in an OSCE scenario You can use this study guide to prepare for the NAC OSCE and MCCQE2. The content was reviewed for the most recent updates;

68

2- Measure the liver span by percussing in the midclavicular line starting from the right

iliac fossa and going up; mark the border between dull and tympanic that represents

the lower border of the liver. Then start from the second intercostal space and go

down; the border between dull and resonant is the upper border of the liver.

Measure the distance between the upper and lower borders; that is the liver span.

Normal liver span is less than 13cm.

3- Percuss the spleen; percussion is more sensitive than palpation for splenic

enlargement. You need to do 2 maneuvers:

Percussion over Traube’s space: marked by the 6th rib, mid-axillary line and

lower costal margin (on the left side of course). Percuss with the patient

breathing normally, dullness suggests splenomegaly. Other differential

includes food in the stomach, pleural effusion or lung consolidation.

Percussion over Castell’s spot: Percuss the lower intercostal space in the

anterior axillary line with the patient breathing in expiration and full

inspiration. Dullness suggests splenomegaly, with the same differential as

Traube’s space dullness.

4- Ascites exam:

Shifting dullness: Percuss parallel to the midline. Start from the level of the umbilicus and go

down till you reach the border between dullness and tympani, take the patients’ permission

then mark the border. Ask the patient to lean to the opposite side and percuss starting from the

mark you made. Shifting dullness is noted to be present when the area of dullness becomes

resonant, continue to percuss and measure the new area of shifting dullness.

Do you have any pain? If the patient is in pain, start from the furthest point. I’m percussing all 4

quadrants looking for dullness or percussion tenderness, both negative. I will now measure the

liver span at the midclavicular line starting from the right iliac fossa, reaching the dullness point

marking the lower border of the liver. I will percuss now for the upper border of the liver

starting from the second intercostal space. Now marking the distance between the two, the

liver span is……cm. I will percuss for the spleen in Traube’s space marked by the lower costal

margin, the sixth rib and the mid-axillary line while the patient is breathing normally. There is

no dullness. I will check now Castell’s sign. This is percussion on the lowermost intercostal

space in the anterior axillary line while the patient is breathing in full inspiration and expiration.

There is no dullness over the Castell’s spot. I will now do shifting dullness to check for ascites.

With the patient lying supine I am percussing parallel to the midline starting at the umbilicus,

Page 69: CANADIAN IMGS GUIDE TO OSCE AND PRACTICE · in an OSCE scenario You can use this study guide to prepare for the NAC OSCE and MCCQE2. The content was reviewed for the most recent updates;

69

reaching the dullness-tympanic border; can I mark the spot on your skin? Can you lean to the

left side please? I start percussion at the mark moving toward the umbilicus I notice that the

dullness-tympani border didn’t shift so the test is negative for ascites.

Palpation:

1- Superficial palpation: Start palpation at a site away from pain. Feel all the 4

quadrants. Look for guarding; a sign of peritoneal irritation.

2- Deep palpation: For deep masses and tenderness. Feel all 4 quadrants. Look for

rebound tenderness; a sign of peritoneal irritation.

3- Palpate for the liver. Ask the patient to breathe deeply in and out. Start from the

right iliac fossa in the mid-clavicular line, feel during inspiration, and move up 1cm

each time during expiration. Also palpate in the epigastrium. If you feel the liver

describe it:

How many cm below the costal margin does it extend?

Edges: sharp or round?

Surface: smooth or nodular

Any masses?

Is it soft, firm or hard?

Is the liver tender?

You have already measured the liver span when you did percussion.

4- Palpate for splenic enlargement: Start from the right iliac fossa and move obliquely

to the left upper quadrant. Then start from the left iliac fossa and move up. The

spleen may enlarge in either direction. Use the same technique you used for liver

palpation. If you don’t feel the spleen, ask the patient to lean to the right side and

try to feel for it. Percussion is more sensitive than palpation for splenic enlargement.

If it was negative it is unlikely that the spleen will be palpable. If you feel the spleen

then you can differentiate it from the kidney by the following:

Splenic notch is felt in splenic enlargement not kidney

The splenic surface feels smooth and regular unlike the kidney

Page 70: CANADIAN IMGS GUIDE TO OSCE AND PRACTICE · in an OSCE scenario You can use this study guide to prepare for the NAC OSCE and MCCQE2. The content was reviewed for the most recent updates;

70

The spleen is always unilateral, while kidneys can be bilaterally enlarged (as

seen in polycystic kidneys)

The spleen is always dull, the kidney can be dull or tympanic

The kidneys are ballotable, the spleen is not

The spleen moves diagonally (towards the left lower quadrant) with

respiration while the kidneys move vertically

5- Palpate for the kidneys: Place your left hand on the patient’s back between the

costal margin and iliac crest. Feel for the kidney with your right hand and use your

left hand to lift the kidney and see if it’s ballotable. Check both sides. Examine for

costophrenic angle tenderness with the patient sitting up (You may defer it to the

end to avoid moving the patient unnecessarily)

6- Palpate for Abdominal aortic aneurysm; described in CVS P.E (Cardiovascular

physical examination) above

7- Elicit signs of Appendicitis, include;

Pin point tenderness at McBurney’s point: 1/3 the distance from the anterior

superior iliac spine to the umbilicus

Rebound tenderness at McBurney’s point

Rouvsing’s sign: Palpation of the right lower quadrant produces pain in the

left lower quadrant

Psoas sign: Abdominal pain on active flexion of the hip

Obturator sign: Abdominal pain on internal rotation of the flexed hip

8- Palpate for gallbladder pathology:

Murphy’s sign: Ask the patient to breathe out, then place your hand on the

patient’s abdomen in the right midclavicular line below the costal margin.

Ask the patient to take a deep breath in. If the patient has cholecystitis,

he/she will have pain with inspiration (positive sign) due to tender

gallbladderand will hold his/her breath.

Page 71: CANADIAN IMGS GUIDE TO OSCE AND PRACTICE · in an OSCE scenario You can use this study guide to prepare for the NAC OSCE and MCCQE2. The content was reviewed for the most recent updates;

71

Courvoisier’s sign: Is a palpable gallbladder. This can be seen in cases that

cause acute distension of the gallbladder. For example: pancreatic

malignancy

Do you have pain in your tummy right now? I’m starting with superficial palpation, feeling for

any masses or tenderness in all 4 quadrants. I notice there is no guarding. Looking at the

patients face, there is no tenderness. I will do deep palpation looking for deep masses or

tenderness. Both are negative. I will palpate now for the liver in the midclavicular line, can you

breathe in and out for me please? Feeling on inspiration and advancing with expiration, I don’t

feel the liver edge. I will use the same technique now to feel for the spleen, starting first from

the right iliac fossa and moving towards the left upper quadrant, I do not feel the splenic edge. I

will do the same to feel for the spleen starting from the left iliac fossa this time. I will now

palpate the kidneys, both are not palpable. I will check now for signs of appendicitis; looking for

pin point tenderness and rebound tenderness at McBurney’s point. Does it hurt when I let go?

Can you bring your right knee to your chest? Does this cause pain in your abdomen? The Psoas

sign is negative. I’m going now to move your leg, let me know if it hurts. Internally rotating the

right hip looking for obturator sign and it is negative. I will now demonstrate Murphy’s sign. I’m

pressing at the midclavicular line just below the right costal margin. Can you take a deep breath

and hold your breath? The patient didn’t catch their breath, so the Murphy’s sign is negative. I

don’t feel an enlarged gallbladder; hence Courvoisier’s sign is negative.

Page 72: CANADIAN IMGS GUIDE TO OSCE AND PRACTICE · in an OSCE scenario You can use this study guide to prepare for the NAC OSCE and MCCQE2. The content was reviewed for the most recent updates;

72

Neurological P.E

Edited by; Dr. M. Wasif Hussain

Includes six main examinations; mental status, cranial nerves, motor system, sensory system,

cerebellar exam and examining the gait. Mental status if directly asked should be done through

a Mini Mental status exam (MMSE) or Montreal cognitive assessment (MOCA) exam, but

otherwise can be conducted through a Glasgow coma scale, which will be discussed at the end

of this section. However, MMSE should be done in cases of delirium.

Cranial nerves

Include:

1- CN 1 (Olfactory); Test each nostril by asking the patient to close their eyes and one

nostril then try to identify a known smell like coffee. Repeat with the other side.

During the exam, look at the examination table, if there was coffee or a special smell

on it go ahead and examine the olfactory nerve. However, in most cases there will

be nothing in the room to test CN1 with. In this case just describe what would you

do.

2- CN2 (Optic); involves looking for five things;

Visual acuity; using Snellen chart or near card examine each eye at a time. If

there isn’t a chart or card in the room, describe what you would do.

Color vision; done with Ishihara chart. In most cases you would just mention

it and the examiner will ask you to move on

Visual fields by confrontation; stand or sit at eye level with the patient. Test

each eye at a time. Ask the patient to cover one eye, and cover or close your

corresponding eye to compare your vision to that of the patient’s (i.e. cover

your right eye to examine the patient’s left eye). Use your pen or finger at a

distance that is approximately at the midpoint in between you and the

patient and ask the patient to let you know when he/she can see it,

alternatively you may do the counting method by asking the patient to count

how many fingers they see; test 4 quadrants for each eye; superior and

Page 73: CANADIAN IMGS GUIDE TO OSCE AND PRACTICE · in an OSCE scenario You can use this study guide to prepare for the NAC OSCE and MCCQE2. The content was reviewed for the most recent updates;

73

inferior nasal, and superior and inferior temporal. Provided you have normal

visual fields the normal patient can see the object or fingers when you can.

Test for extinction with the patient’s both eyes open, hold both of your

hands up and while the patient is staring at your nose, ask them to point to

the hand that is moving. Start by moving each hand individually and then

both simultaneously. A patient with extinction will neglect the contralateral

side.

Pupillary reflex; shine a light into the patient’s eye (You should bring your

own to the exam), and notice the pupil’s direct (the eye to which you shine

the light at) and consensual reflex (constriction of the other pupil). Do the

swinging light test by moving the light in a swinging motion from one eye to

the other, holding for approximately 1-2 seconds on each eye to test for

RAPD (relative afferent pupillary defect). Note that the pupillary reflex

afferent limb is the optic nerve; the efferent is the Oculomotor nerve.

Fundoscopy; Is detailed under Fundoscopy P.E

3- CN3 (Oculomotor), CN4 (Trochlear), CN6 (Abducens); Inspect the eyes for pupils

shape, size, position. Note if both sides are symmetrical and if the patient has

nystagmus at primary gaze. Then test the extra ocular movements; Ask the patient

to keep his/her head still, and follow your finger; draw an H and notice both eyes

moving in all direction. Look for any nystagmus and ask the patient to report any

diplopia.

4- CN5 (Trigeminal); Has two components;

Motor; CN5 innervates the muscles of mastication. Ask the patient to clench

his/her teeth together and feel for the masseter muscle on both sides. Also

feel for temporalis muscles. Ask the patient to open his/her mouth against

resistance testing the power of Masseter and Temporalis muscles. Then

check the power of Ptyregoid muscle by asking the patient to move his/her

jaw right and left resisting your hand pushing in the opposite direction.

Sensory; CN5 supplies the skin of the face. Use a pin to test pinprick over the

3 branches of CN5; frontal (V1), maxillary (V2) and mandibular (V3). Compare

both sides. Do the test with the patient’s eyes closed.

5- CN7 (Facial); Inspect the patient face for symmetry and ask the patient to make a

variety of facial expressions; Wrinkle his/her forehead or raise his/her eye brows to

differentiate an UMN facial palsy from a lower (UMN spares the frontalis) Close

Page 74: CANADIAN IMGS GUIDE TO OSCE AND PRACTICE · in an OSCE scenario You can use this study guide to prepare for the NAC OSCE and MCCQE2. The content was reviewed for the most recent updates;

74

his/her eyes tight and resist your try to open them, Smile; note symmetry of both

lower lips and Nasolabial folds, Close his/her mouth and resist you trying to open it.

Puff his/her cheeks and resist your try to pop them

6- CN8 (Vestibulocochlear); Test to cochlear component. Whisper into the patient’s ear

while distracting the other ear by rubbing your fingers together. Ask the patient to

identify what you have just whispered into his/her ear, repeat with the other ear.

You are not required to bring a 512 tuning fork to the examination. However, if

there was one in the room, do Weber and Rinne test; Weber; Place the tuning fork

on the patient’s forehead and ask him/her if it’s different between the two ears.

Rinne test; Put the tuning fork on the mastoid process and ask the patient to let you

know when the sound stops. Immediately place the fork in front of the patient ear.

Air conduction is normally stronger than bone conduction. You don’t need to test

the Vestibular component.

7- CN9 (Glossopharyngeal) and CN10 (Vagus); test them together; look into the

patient’s mouth and notice any deviation of the palate or any asymmetry of palatal

movement when the patient says: Ahh. Mention that you could examine the gag

reflex to be thorough, but this will rarely be asked for the sake of the actor’s

comfort. Note, a palatal deviation is due to weakness on the contralateral side. Test

articulation by asking the patient to say; Pa Ta Ka.

8- CN11 (Accessory); Ask the patient to shrug his/her shoulders up and resist your

hands pushing down. Test the power of Sternocleidomastoid muscle by asking the

patient to turn his/her head to either side and resist your hand pushing on the

lateral side of the chin.

9- CN12 (Hypoglossal); Ask the patient to open their mouth and notice any

fasciculations or atrophy. Then ask the patient to stick his/her tongue straight out

and notice any deviation. Ask the patient to move his/her tongue from side to side.

If there is a question of weakness, you can ask the patient to push their tongue into

their cheek and resist you pushing against it. Note, a tongue deviation is due to

weakness on the ipsilateral side.

The first cranial nerve is the Olfactory, which I test by asking the patient to identify a known

smell using one nostril at a time with his/her eyes closed. I examine the 5 components of CN2

starting with visual acuity using a Snellen chart, and color vision using Ishihara chart (the

examiner will most likely say move on). Then I test the visual fields by confrontation test; can

Page 75: CANADIAN IMGS GUIDE TO OSCE AND PRACTICE · in an OSCE scenario You can use this study guide to prepare for the NAC OSCE and MCCQE2. The content was reviewed for the most recent updates;

75

you please close your right eye and tell me how many fingers you see. Superior Temporal,

Superior Nasal, Inferior Temporal, Inferior Nasal. I will do the same on the right side; can you

please close your left eye? I will now test for extinction with the patient’s both eyes open; how

many fingers do you see? (point to the hand that is moving) I will shine a light into your eyes.

I’m doing the pupillary reflex noting the direct and consensual reflexes. Then I will do the

swinging light test looking for RAPD. All tests are normal so far. The final component of the

optic nerve exam is Fundoscopy; can you please focus on that point on the wall, I will dim the

light and look into patient’s eyes looking for papilledema. Next I will examine 3 cranial nerves

together; 3rd, 4th and 6th ; I’m inspecting the pupils I notice that both sides are symmetrical

2mm in diameter with normal shape and position and no nystagmus. Can you please keep your

head still and follow my finger? Please let me know if you have double vision at any point. The

patient has normal extraocular movements and no nystagmus. Now I will examine CN5 starting

with its sensory component; can you close your eyes and nod your head each time I touch your

face with the pin? Does it feel the same on both sides? I’m testing V1, V2 and V3. Now moving

to the motor component; can you please clench your teeth together? I’m palpating the

masseter muscle and it feels normal. I’m also palpating the Temporalis. Can you please open

your mouth and don’t let me close it? Testing the same two muscles. Can you move your jaw to

the left and resist me? Great now to the right. I’m testing Pterygoid. The trigeminal nerve is

intact. Next is the facial nerve, I notice that the face is symmetrical, Can you raise your

eyebrows? Forehead wrinkles are symmetrical. Can you close your eyes and don’t let me open

them? Can you smile please? No mouth drop and the Nasolabial folds are symmetrical. Now

close your mouth and don’t let me open it. Can you puff your cheeks and resist me trying to pop

them? The facial nerve is intact. Now cranial nerve 8, the Vestibulocochlear, I will test the

hearing component; I’m going to whisper in your right ear? Say anything like table, chair or a

number. What did I say? Will do the same on the left ear, what did I say? I will do Weber test to

see if there is any lateralization, placing the tuning fork on the patient’s forehead, do you hear it

in both ears? Does it sound the same? Next I will do Rinne test placing the tuning fork on the

mastoid process, let me know when the sound disappears? Putting the fork in front of the ear;

do you hear it now? Air conduction is normally stronger than bone conduction, CN8 is intact. I

will test now the Glossopharyngeal and Vagus nerves. Can you open your mouth and say Ahh?

There is no palatal deviation, the palatal movement is symmetrical. Can you say Pa Ta KA?

Articulation is intact. I will do the gag reflex. The examiner may say pass as the test is not

comfortable. Both nerves are intact. Next is the accessory nerve; Can you shrug your shoulders

up? Don’t let me bring them down. Move your face to the right and resist me. Now to the left.

The accessory nerve is intact. The last nerve is the hypoglossal; can you open your mouth

please? No tongue fasciculation, atrophy, or asymmetry. Can you stick your tongue straight

out? There is no deviation. Can you move you move your tongue from side to side? The

hypoglossal nerve is intact.

Page 76: CANADIAN IMGS GUIDE TO OSCE AND PRACTICE · in an OSCE scenario You can use this study guide to prepare for the NAC OSCE and MCCQE2. The content was reviewed for the most recent updates;

76

Motor examination;

Includes;

1- Inspection; Look at the patient posture, note muscle’s bulk and symmetry.

Observe for fasciculations or abnormal movements.

2- Tone examination; you need to examine the tone of the upper and lower limbs.

Ask the patient to relax and passively move each limb initially slowly throughout

looking for rigidity and then having a fast phase on extension/supination in the

upper extremity and on flexion in the lower extremity looking for spasticity.

Spasticity is velocity dependent while rigidity is not. If the patient is not fully

relaxed try distracting him/her by talking to them.

For the upper limbs do the following; flexion and extension of the elbow,

pronation and supination, rotation at the wrist to look for cogwheel

rigidity seen in Parkinson’s.

For the lower limbs do; internal and external rotation at the hip, flexion

and extension at the knee (with the patient lying supine and the legs fully

extended; briefly left the knee- illustrated in the video). Flexion and

extension of the ankle. Test for clonus as well.

3- Power; you need to rate each muscle group you test. On a scale from 0-5; 0

complete paralysis, 1 flicker of movement, 2; movement with gravity eliminated,

3; movement against gravity but no resistance, 4; movement against gravity and

some resistance (often graded as 4-, 4, or 4+), 5; full power. You don’t need to

mention the exact nerve or nerve roots you are testing; it is enough to name the

movements. Give the patient clear instructions; mimic the movement you need

them to do if you have to. For all muscle groups except deltoids and hip flexors

always support proximally and test distally and compare the two sides. Start with

pronator drift followed by select muscle groups. If there is weakness in a focal

area, you will need to examine the muscles in more detail (these will be written

in italics).

Page 77: CANADIAN IMGS GUIDE TO OSCE AND PRACTICE · in an OSCE scenario You can use this study guide to prepare for the NAC OSCE and MCCQE2. The content was reviewed for the most recent updates;

77

Test for pronation drift; very sensitive for UMN weakness in the upper

extremity; ask the patient to hold his/her arms straight forward with the

palms up, fingers extended and eyes closed and notice if any of the arms

drop, pronate and fingers flex (not all need to be present for a positive

drift).

Shoulders abduction, Shoulder adduction

Elbows flexion and extension

Wrists flexion and extension

Fingers abduction, extension and flexion

Thumb Abduction

Hips flexion and extension and abduction and adduction

Knees Flexion and extension

Ankle dorsiflexion and plantar flexion and inversion and eversion.

4- Reflexes; do deep tendon reflexes (DTR). You need to mention the name of the

reflex you are doing and the nerve root. Ask the patient to relax, and warn

him/her that you are going to tap with a hummer to test their nerves. Watch and

feel for muscle contraction. Grade the reflexes on a scale from 0-4: 0; no

reflexes, 1; decreased, 2; normal, 3; increased (brisk, with spread), 4; increased

with clonus. Test the following (note an easy way to remember the roots is listed

below as 12345678);

Ankle jerk; S1,S2

Knee jerk; L3, L4

Brachioradialis; C5.6

Biceps jerk; C5.C6

Triceps;C7, C8

If you can’t elicit a reflex, try asking the patient to clench his/her teeth for upper limbs reflexes,

and lock fingers and pull hands apart for lower limbs reflexes.

Page 78: CANADIAN IMGS GUIDE TO OSCE AND PRACTICE · in an OSCE scenario You can use this study guide to prepare for the NAC OSCE and MCCQE2. The content was reviewed for the most recent updates;

78

Examine the plantar reflex after doing the ankle jerk. Warn the patient that you are going to

stroke their foot and that it may be uncomfortable, and move the sharp part of your hammer

along the lateral side of the plantar aspect of the foot turning in just before the toes forming an

arc. Watch if the big toe goes up or down. (a Babinski sign is an up going plantar)

As part of the motor exam you need to be able to differentiate upper motor neuron (UMN)

from lower motor neuron (LMN) lesions. Refer to table 3-1 for explanation. Note that mixed

UMN and LMN signs are seen in motor neurone disease such as ALS.

UMN LMN

Muscle weakness, NO fasciculations Muscle atrophy and weakness WITH

fasciculations

Hypertonia with spasticity Hypotonia

Hyper-reflexia Hypo-reflexia

Up-going Plantar reflex Down going Plantar reflex

Table 3-1; Difference between UMN and LMN lesions.

I’m examining the motor system starting with inspection; no abnormal posturing, movements,

or fasciculations. The muscles are symmetrical with no atrophy. I will start by testing the tone.

Can you make your arm floppy for me? Flexing and extending the elbow slowly then rapidly,

now pronating and supinating the hand, rotating the wrist looking for cogwheel rigidity. I will do

the same with the other arm. The tone of the upper limbs is normal. Can you make your leg

floppy for me? Internal and external rotation of the hip, flexion and extension of the knee,

ankle dorsiflexion and plantar flexion. I will test for clonus as well. Now I will compare both

sides. Lower limbs tone is normal with no clonus. Next I will test power. I show patients the

movement I need them to do. Can you put your arms out straight in front of you with your

palms up and fingers straight and close your eyes? There is no pronator drift. Can you go like

this and resist me? Shoulder abduction and adduction. Now can you go like this and resist me;

Elbow flexion and extension. Move your wrist up and don’t let me break you, now down, wrist

flexion and extension. Spread your fingers and don’t let me bring them together; fingers

abduction. Keep your fingers straight and don’t let me bend them. Curl your fingers and don’t

let me straighten them (alternatively you can check grip strength). Finger extension and flexion.

Comparing both sides, upper limbs power is 5/5. I will now do the lower limbs with the patient

Page 79: CANADIAN IMGS GUIDE TO OSCE AND PRACTICE · in an OSCE scenario You can use this study guide to prepare for the NAC OSCE and MCCQE2. The content was reviewed for the most recent updates;

79

lying supine; Lift your leg up off the table with your knee straight and resist me pushing you

down; Hip flexion (Note ; test hip extension at the end of the exam by asking the patient to lie

prone). Bring your hips apart and don’t let me bring them in, now the opposite. Hip abduction

and adduction. Bend your knee and kick out, resist me, now in. Knee flexion and extension.

Press down with your foot as if you are pressing the gas, ankle plantar flexion now bring your

foot up, ankle dorsiflexion , now out; eversion, and in; inversion. Lower limbs power is 5/5. Next

I will do the DTR. I’m going to test your nerves by taping with a hummer, this shouldn’t hurt at

all. Starting with the ankle jerk S1, S2, comparing both sides, now the knee jerk L3,L4. Normal

lower limbs reflexes. I’m testing Brachioradialis C5.C6, then Biceps C5. C6, and finally Triceps

jerk C7. Normal upper limbs reflexes. I will now do the plantar reflex. I’m going to stroke your

foot with a sharp object, it may be a little uncomfortable, is that ok? plantars down going

bilaterally.

Sensory examination;

The sensory examination involves testing a wide variety of sensations and can take

a tremendous amount of time. Make sure you know your dermatomes very well (figure 3-4)

Practice and master the following for the sake of the examination. Always start by eliciting a

history of any sensory complaint and focus on the areas mentioned.

1- Pinprick; using a pin, which you will find on the table in the examination room, ask the

patient if he can feel it with his/her eyes open at a single location so that the sensation

is recognized. Then test the upper extremities with the patient’s eyes closed comparing

both sides in such a way that you are testing proximal sensation, distal sensation

(looking for a stocking glove pattern of peripheral neuropathy) and also dermatomes

and nerve distributions. Note that in most cases it is sufficient to test the upper and

lower limbs. Nevertheless, if spinal injury is suspected you need to test the neck and

trunk as well. The same principle apply for testing other sensory modalities like fine

touch, and temperature, but it’s unlikely to be asked to do them especially with the time

restraint of OSCE stations. Follow the link below for dermatomal distribution;

https://www.pinterest.com/pin/487162884665368556/

Page 80: CANADIAN IMGS GUIDE TO OSCE AND PRACTICE · in an OSCE scenario You can use this study guide to prepare for the NAC OSCE and MCCQE2. The content was reviewed for the most recent updates;

80

2- Vibration sense; use a 128 tuning fork , place it on the patient sternum at first to

identify the sensation. Then test over bony prominence, ask the patient to let you know

when it stops. Start with the interphalyngeal joint of the fifth metatarsals of the lower

limbs bilaterally. If vibration is preserved distally you don’t need to test more proximal

locations. If it is lost you need to test proximally until the level becomes evident. To test

vibration, you will measure the duration of sensation felt by the patient until they do

not feel it anymore and compare it to the other side as well as your thumb (presumed

normal control)

3- Proprioception; test the thumb and big toe. First show the patient what you will be

doing with eyes open before doing the test with the eyes closed. You will move a single

joint (interphalyngeal) up or down relative to the previous position. Start with large

amplitude movements followed by more subtle movements.

4- Romberg test; this is classically tested after gait assessment. Ask the patient to stand

with his/her feet together with arms either extended forward or straight by their side

and with eyes open. Observe balance. Then ask the patient to close their eyes for 5

seconds. A positive sign is a fall not swaying .Make sure to stand behind the patient to

catch him/her if needed. Balance is lost with eyes closed in cases of proprioception loss

as vision compensates for loss of joints position feeling in space. While 50% of patients

with cerebellar disease maintain their balance at 60 seconds.

Next is Sensory exam; I will start with pinprick. Do you feel a sharp poke on your hand? I need

you to close your eyes and say yes each time you feel the pin. Let me know if it doesn’t feel the

same on both sides. C4 left and right, C5,C6,C7,C8,T1. Now the lower limbs; L1, L2, L3, L4, L5,

S1. The same principle applies for testing fine touch, cold and hot sensations I will now examine

proprioception holding the patient thumb, that’s up and that’s down, can you close your eyes

and let me know if it’s up or down? I will compare both sides, then I will do the same with the

big toe; this is up and this is down, can you tell me if it’s up or down? Comparing both sides is it

up or down? (Note that you need to move each finger/toe multiple times,for example; up up

down up). Next I will test your vibration sense with a tuning fork; that’s how it feels.. Can you

close your eyes? Do you feel the vibration? Let me know when it stops. Next I will do the

Romberg test; Can you stand up and bring your feet together, extend your arms in front of you.

Now close your eyes. The patient is steady and the test is negative.

Page 81: CANADIAN IMGS GUIDE TO OSCE AND PRACTICE · in an OSCE scenario You can use this study guide to prepare for the NAC OSCE and MCCQE2. The content was reviewed for the most recent updates;

81

Cerebellar exam

Be sure to note the handedness of the patient as that may explain mild

asymmetries of coordination. Ataxia should always be out of keeping with any weakness or

sensory loss. Exam Includes;

1- Speech; note if speech is normal.

2- Extraocular movements looking for nystagmus and saccadic movements.

3- Finger to Nose test; ask the patient to touch your finger then his/her nose as fast and

accurate as he/she can. Change the position of your finger, and try to get the patient to

fully extend his/her arm as he/she tries to reach for your finger.

4- Rapid alternating movements; ask the patient to keep the palm of one hand up and tap

on it with the other hand alternating palmar and dorsal sides as quickly as possible (i.e.;

pronating and supinating the arm) Switch hands to test the contralateral side.

5- Heel-shin test; ask the patient to slide his/her heel on the shin moving up-down –up in a

straight line. You may want to show the patient how to do it. Test both sides. This test

should ideally be done while lying supine but can be done while sitting.

6- Gait; detailed next, as gait is an important part of general neurological exam as well. In

cerebellar exam ask the patient to walk heel to toes in addition to general gait

examination. Make sure to walk along with the patient to catch him/her in case he/she

is unstable.

I start my cerebellar exam by noticing that the patient’s speech is normal. Follow my finger with

your eyes, try to keep your head still and let me know if you see double. Can you touch your

nose then my finger as quickly and accurately as you can? Can you go like this for me? (Show

the patient how to do rapid alternating movement). Can you slide your heel on your shin going

from your foot up to your knees and back to the foot? (Good idea to demonstrate to the

patient how it’s done), Can you walk for me? Turn around and come back? Can you walk on

your tiptoes? Can you walk in a straight line touching your heel with your foot? (Also show the

patient how to do it) Now walk on your heels? The gait is smooth with normal base and arm

swing, normal turning, no imbalance, no difficulty with tiptoes or heels. (Always walk with the

patient and be ready to catch him/her if they lose balance)

Page 82: CANADIAN IMGS GUIDE TO OSCE AND PRACTICE · in an OSCE scenario You can use this study guide to prepare for the NAC OSCE and MCCQE2. The content was reviewed for the most recent updates;

82

Gait

Obviously you will ask the patient to walk! Observe the stance, base, smoothness of the gait,

arm swing, balance. Comment on patient’s balance. It is important to walk along with the

patient so that you can catch him/her if balance is lost. Ask the patient to turn around and ask

the patient to walk on the tip toes (S1) and heels (L4). Make sure to support the patient if

needed. If Parkinsons disease is suspected, look for festination of gait, shuffling, turning en bloc

and perform the pull test while standing behind the patient; warn the patient that you are

going to pull him/her to the back. A fall or festination (taking multiple steps to the back-one

step is normal) is a positive sign.

Refer to the cerebellar exam above for suggested verbalization of gait examination.

Glasgow Coma Scale

Do it in all trauma and coma cases if asked to assess, manage or perform a physical exam.

Score range is 3-15 covering 3 major functions;

1- Eye opening (4 points in total) ; 1; none, 2; to pain, 3; to voice, 4; spontaneous

2- Best motor response (6 points) ; 1; none, 2; Extension to pain (Decerebrate

posturing), 3; Flexion to pain (Decorticate posturing), 4; withdrawal from pain,

5; localizes pain, 6; Obeys commands

3- Best Verbal response (5 points) ; 1; none, 2; incomprehensible sounds, 3;

inappropriate words, 4; confused, 5; oriented

I will calculate Glasgow Coma Scale to assess patient’s consciousness; starting with the first

component eye opening, I notice the patient opens his/her eyes spontaneously that’s 4 for eye

opening. Then I will assess the best motor response can you left your right arm up for me

please? The patient obeys a command that’s a 6 for best motor response. Then the best verbal

Page 83: CANADIAN IMGS GUIDE TO OSCE AND PRACTICE · in an OSCE scenario You can use this study guide to prepare for the NAC OSCE and MCCQE2. The content was reviewed for the most recent updates;

83

response; what is your name? Where are you? What is todays date? Who am I? The patient is

oriented and speech is normal, that’s a 5 for best verbal response. Total Glasgow Coma Scale is

15.

Page 84: CANADIAN IMGS GUIDE TO OSCE AND PRACTICE · in an OSCE scenario You can use this study guide to prepare for the NAC OSCE and MCCQE2. The content was reviewed for the most recent updates;

84

Musculoskeletal System Examination (MSK)

Edited by Dr. Stephanie Keeling

Family doctors in Canada are the first gate to all sorts of medical problems. Unlike other

countries where orthopedic surgeons and rheumatologists may be the first physicians to assess

a patient with joint pain, family doctors in Canada often see these patients first, determine the

pathology and then refer as appropriate. Thus, a good screening musculoskeletal exam is

common in Canadian OSCEs.

The sequence of the MSK examination differs somewhat from a general examination. Do the

following;

1- Inspection: Look for SEADS; swelling, erythema (redness), atrophy of muscles,

deformity and skin changes. Inspect how the joint looks when it is moved.

2- Palpation: Feel for temperature difference first, then palpate for tenderness

naming the muscle, ligament or tendon you are palpating.

3- Range of motion: test both active (performed by the patient) and passive

(performed by you) giving the patient clear instructions and naming each

movement as you go. Look for pain, listen and feel for clicks or crepitus and

observe limitations of movement.

4- Special tests: will be specific for each joint.

5- For completeness: say you will examine the other joints including the joint above

and below, and the neurovascular compartment of the limb. In most cases, you

will run out of time after doing the special tests. It is important however to

mention that you would do all of the above. One exception to keep in mind is that

you must include neurovascular examination of the lower limbs when you

examine the back.

In this section I will cover the P.E of the hands and wrists, elbows, shoulders, back, hips, knees

and ankles.

The three most common MSK complaints include back, knee and shoulder pain. Practice them

well.

Hands and Wrists P.E

Page 85: CANADIAN IMGS GUIDE TO OSCE AND PRACTICE · in an OSCE scenario You can use this study guide to prepare for the NAC OSCE and MCCQE2. The content was reviewed for the most recent updates;

85

- Start by stating that you will examine and compare both sides, examining the elbows

and hands, and the neurovascular compartments of the arms. Focus on the affected

wrist.

- Inspection: compare both sides, looking at the palmar (volar) and dorsal aspects of the

joint. Look for SEADS (see above)

- Palpation: feel for temperature difference with the dorsum of your hands. Then feel

the affected joint with both hands looking for any tenderness or swelling.

- ROM: Includes:

Active: Ask the patient to do the following movements and demonstrate to the

patient what you need him/her to do:

1- Flexion; by bringing the dorsum of the hands together with the fingers

pointing downwards

2- Extension; by doing the prayer sign.

3- Ulnar deviation

4- Radial deviation

5- Arm pronation

6- Arm supination

Passive movements; Ask the patient to make his/her wrist floppy for you and

move the wrist in all 6 directions covered in active movements.

- Special tests: Include:

Testing for Carpal Tunnel Syndrome (CTS):

1-Katz diagram; ask the patient to draw for you where exactly he/she feels the pain/tingling;

pain felt at the fingers, wrist or proximal to the wrist is classic for CTS, pain felt in the palmar

aspect and the first, second and third fingers indicates probable CTS. Pain felt in the radial

nerve distribution makes CTS unlikely.

2-Hypoalgesia: reduction of sensation to painful stimuli over the palmar side of the index (2nd)

finger compared to the little (5th) finger.

Page 86: CANADIAN IMGS GUIDE TO OSCE AND PRACTICE · in an OSCE scenario You can use this study guide to prepare for the NAC OSCE and MCCQE2. The content was reviewed for the most recent updates;

86

3- Weak thumb abduction

4-If you have more time and can complete the power and sensation testing described below,

consider asking them to do Tinel’s (tapping over the distal crease on the median nerve

produces paresthesia the median nerve distribution) and Phalen’s (flexing the wrist at 90 for 60

seconds produces paresthesias in the median nerve distribution) tests although they have low

likelihood ratios, because they are so classic

Allen’s test

Particularly important in trauma cases, and prior to performing a radial artery puncture for

arterial blood gases. Refer to page 14 of this chapter for full description. Feel the radial pulse

then proceed with the test. Only proceed with this if you have completed everything else

including power and sensation testing described below.

- Test the power of the wrist asking the patient to resist all the above movements. Then

test sensation over the radial, ulnar and median nerves. Do upper arm DTR and feel

the pulses. If you run out of time, inform the examiner that you would intend on

performing power and sensation testing in a clinical setting. In most cases you will not

have time to examine the joint above and below but it is always a good idea to

indicate you would do it.

I will examine both wrists, starting with the right (do the affected first), then I will examine the

elbow and the hand and the neurovascular compartment of the upper limbs. I start with

inspection comparing both sides looking at the dorsal and palmar aspects of the wrists, no

swelling, erythema, no thenar or hypothenar atrophy, no deformities, and no skin changes. Do

you have pain right now? I’m feeling for temperature difference, and now palpating the wrist

for any tenderness or swelling, also feeling over the snuff box for scaphoid fractures. I will now

check active ROM; can you copy my movements please? Extension, flexion, ulnar deviation and

radial deviation, arm pronation and supination. Next I will do the passive range of movement

looking for any limitation, pain, or clicks/crepitus, I’m going to move your wrist, can you make it

floppy/relax for me? Flexion, extension, ulnar deviation, radial deviation, pronation and

supination. No abnormalities. I will now do special tests for CTS: Do you get pain in your hands

or wrists? Can you draw for me exactly where you feel the pain (Comment if Katz diagram is

classic, probable or unlikely). Then I will test for hypoalgesia: This will feel sharp. Please close

your eyes and tell me if you feel it? Is the feeling the same over the index (2nd) and little (5th)

fingers? Then I will test the power of thumb abduction. Can you resist me pushing your thumb

please? Next I will tap on your wrist let me know if it hurts or feels tingly. Tinel’s test is negative.

Page 87: CANADIAN IMGS GUIDE TO OSCE AND PRACTICE · in an OSCE scenario You can use this study guide to prepare for the NAC OSCE and MCCQE2. The content was reviewed for the most recent updates;

87

Can you bring both your hands together with the fingers facing down and hold for 60 seconds?

Phalen’s test is negative as the patient didn’t experience numbness, pain or paresthesia

Elbows P.E

- State that you will examine both sides but will start with the affected one first and you

will examine the shoulders, wrists and neurovascular compartments of the upper

limbs

- Inspection: Expose both upper limbs; look for SEADS comparing both sides

- Palpation: feel for temperature first, and then feel the joint for any swelling, nodules

or tenderness paying particular attention to the olecranon process, medial and lateral

epicondyles.

- ROM: includes flexion, extension, pronation and supination. Test both active and

passive ROM looking/feeling for any limitations, hyperextension, pain, or crepitus. A

flexion deformity (missing full extension) will suggest an elbow effusion and should be

palpated.

- Special tests:

Lateral epicondylitis: palpate the lateral epicondyle with the elbow extended,

forearm pronated and wrist flexed, feel for any tenderness (seen in tennis

elbow; think about the movement you make when you play tennis); pain at the

lateral epicondyle with resisted extension of the elbow suggests tennis elbow.

Medial epicondylitis: palpate the medial epicondyle with the elbow extended,

forearm supinated and wrist extended looking for pain (seen in golfer’s elbow);

pain at the medial epicondyle with resisted flexion of the elbow suggests

golfer’s elbow.

Page 88: CANADIAN IMGS GUIDE TO OSCE AND PRACTICE · in an OSCE scenario You can use this study guide to prepare for the NAC OSCE and MCCQE2. The content was reviewed for the most recent updates;

88

I am going to examine both elbows and compare both sides but focusing on the right. Then I

will examine the shoulders and wrists and the neurovascular compartment of the upper limbs. I

start with inspection, no swelling, erythema, no muscle atrophy, no deformities, and no skin

changes. I will feel your elbow; please let me know if it hurts. I am looking for temperature

difference, feeling for effusions and palpating the dorsal aspect of the forearm, the medial and

lateral epicondyles and the olecranon process. No swelling or deformities. Next I will do the

Active ROM. Can you copy my movements please? Flexion, extension, pronation, supination.

Can you relax your arm for me and I will move it for you, doing the same movements you just

did. No limitation of movement, no crepitus, no pain and no hyperextension. Now I will test for

tennis elbow noting any pain as I palpate the lateral epicondyle and with resisted extension.

Next I’m testing for golfer’s elbow, feeling for any pain over the medial epicondyle and with

resisted flexion.

Shoulder P.E

The shoulder is probably the most complicated joint. There are so many special tests. I will

cover basic knowledge about shoulder anatomy and P.E in this section to help you as a

generalist determine if the pain is arising from the shoulder, surrounding tissues or referred

from another site.

Rotator cuff tears and tendinitis are very common so knowing the muscles that make up the

rotator cuff and their specific actions will help you conduct a meaningful P.E. Other common

pathologies include: bicipital tendinitis, impingement, frozen shoulder and anterior dislocation.

Four muscles make up the rotator cuff (SITS):

1- Supraspinatus Abduction

2- Infraspinatus External rotation

3- Teres Minor External rotation

Page 89: CANADIAN IMGS GUIDE TO OSCE AND PRACTICE · in an OSCE scenario You can use this study guide to prepare for the NAC OSCE and MCCQE2. The content was reviewed for the most recent updates;

89

4- Subcapularis Internal rotation

Follow the link below for shoulder joint anatomy;

http://www.webmd.com/pain-management/picture-of-the-shoulder

- Start shoulder P.E by stating you will examine both sides but will focus on the affected

side, and that you will also examine the joint above (cervical spine) and below (elbow)

and the neurovascular compartment of the upper limbs.

- After asking permission to examine the patient, wash your hands (or use hand

sanitizer as done in some OSCEs) and respectfully drape the patient by lowering the

gown exposing both shoulders, keep the bra on in female patients. Ask the patient to

report any pain or discomfort during the P.E.

- Inspection: compare both sides looking at the shoulders from front, back and sides.

Noticing any SEADS; swelling, erythema, muscle atrophy, deformities and skin

changes. You can ask the patient to complete a shoulder arc test at this point and

inspect the shoulder as they elevate the arm in the scapular plane then bring it down

again. You are looking for any abnormalities including difficulty completing this

motion. Alternatively, you can complete this later in the ROM section.

- Palpation: check for temperature difference. Feel for tenderness and swelling starting

from the sternoclavicular joint, palpate along the clavicle, acromion, acromioclavicular

joint, humerus head, bicipital tendon in the bicipital groove, spine of scapula,

supraspinatus and infraspinatus muscles and medial border of the scapula.

- ROM: Includes

Active: Ask the patient to complete the following movements:

1- Flexion: Move the arm forward

2- Extension: Moving the arms back, often completed with the elbow bent

Page 90: CANADIAN IMGS GUIDE TO OSCE AND PRACTICE · in an OSCE scenario You can use this study guide to prepare for the NAC OSCE and MCCQE2. The content was reviewed for the most recent updates;

90

3- Abduction: Bring the arm away from the body in the scapular plane until

the hand is above the head, and the shoulder/upper arm is as close as it

can get to the ear without using the trapezius muscles to lift up the

shoulder girdle.

4- Adduction: Bring the abducted arm back to the side.

5- Internal rotation: Ask the patient to flex the elbow to 90 degrees, and then

medially rotate the arm at the shoulder, bringing the hand to rest on the

abdomen. The flexed elbow remains in the same position resting at the

patient’s side throughout this movement.

6- External rotation: Same position as in internal rotation but ask the patient

to rotate the arm at the flexed elbow laterally, away from the abdomen as

far as it can go. The flexed elbow remains in the same position resting at

the patient’s side throughout this movement.

7- Adduction and internal rotation: The dorsum of the hand is touching the

back (note that you have already tested each of these movements

separately, you have the option to ask the patient to do this movement as

you test more than one muscle at the same time)

8- Abduction and external rotation: The patient puts his hands behind his

head with the palms touching the head.

Passive: Ask the patient to relax, and gently move the shoulder in all the above

directions noting any limitation, pain, clicks or crepitus.

- Special tests: NOTE: It is key to do the general physicial exam well and know a few

vital special tests. If you focus on special tests but miss the general examine

maneuvers you will score low.

As I mentioned, you have a long list to choose from. Don’t panic! The most important tests

(which are also the easiest to remember) include those that test the four rotator cuff muscles.

If you can complete the tests for the 4 SITS muscles, you will know where the pathology is for

the majority of patients. For the other tests listed, you may need to do them depending on

what the patient has, but they are less critical in the general clinic setting.

Painful arch (Shoulder Arc Test): Pain between 70 and 100⁰ during shoulder

abduction. To save time you can mention this test when you ask the patient to

Page 91: CANADIAN IMGS GUIDE TO OSCE AND PRACTICE · in an OSCE scenario You can use this study guide to prepare for the NAC OSCE and MCCQE2. The content was reviewed for the most recent updates;

91

abduct the shoulders when testing active movements. May suggest

subacromial impingement.

Testing the power of Supraspinatus (abductor): Ask the patient to bend the

arms (like the chicken dance when they are mimicking a bird); with flexed

arms, ask him/her to abduct their arms while you resist abduction; if there is

pain or weakness, it may suggest supraspinatus pathology.

Testing the power of Infraspinatus and Teres Minor (external rotators): Ask the

patient to flex the elbow to 90⁰ with forearms in front of the body; then ask

the patient to externally rotate the arm at the shoulder as you provide

resistance along the forearm. Keep the elbows bent at 90 degrees against the

body during this maneuver. Pain and/or weakness suggest tear or tendinitis of

either muscle.

Lift off test for Subscapularis(internal rotator): To test the power of

subscapularis muscle, ask the patient to place his/her hand behind his/her

back with the palms facing outwards; push his/her hand down against the back

and ask the patient to resist you (resisted internal rotation)

At this point, if you run out of time, you will still have likely been able to deduce certain things

about the pain. Important factors to consider include:

1. Pain or limitation in ROM with active & passive ROM:

suggests intra-articular process (in the shoulder joint)

2. Pain or limitation in ROM with active (but NOT

passive ROM): suggests extra-articular process (eg.

rotator cuff tendonitis or bursitis)

3. Resisted rotator cuff testing can then help isolate the

involved muscles/tendons.

Other Special Tests (Consider only if you have time)

Drop arm test: Tests supraspinatus function. Supraspinatus initiates arm

abduction. In this test, fully abduct the patient’s arm then ask him/her to

slowly adduct it. In cases of supraspinatus tear, the patient will slowly lower

Page 92: CANADIAN IMGS GUIDE TO OSCE AND PRACTICE · in an OSCE scenario You can use this study guide to prepare for the NAC OSCE and MCCQE2. The content was reviewed for the most recent updates;

92

the arm to 100⁰, after which the movement becomes irregular and the arm

may suddenly drop down due to the disrupted supraspinatus tendon.

Empty Can test: Another test for supraspinatus. Ask the patient to abduct the

arm to 90 degrees in the scapular plane, internally rotate the arm (as if

emptying a drink from a can). Exert a downward pressure at the elbow or wrist

which the patient resists. If there is pain or weakness, this suggests

supraspinatus pathology.

Testing for anterior shoulder dislocation: The following 3 tests can all be done

in a single maneuver:

1- Anterior Apprehension test: Preferably done with the patient sitting; ask the

patient to hold his/her arm in the throwing position (shoulder abducted 90⁰,

elbow flexed 90⁰) and apply pressure to the posterior aspect of the humerus,

as if you are trying to anteriorly dislocate the shoulder. Apprehension or pain

indicates a positive test

2- Relocation test: With the patient lying down and the arm in the throwing

position, apply anterior pressure to the humerus as if you trying to relocate an

anteriorly dislocated shoulder. Feeling relief is a positive test

3- Anterior release test: After attempting to relocate the shoulder, suddenly

release your arm. Pain or apprehension is a positive test.

Sulcus sign for inferior shoulder dislocation: Pull the patient’s shoulder

downwards and look for an indentation between the glenoid rim and humerus

suggestive of inferior shoulder dislocation

Posterior apprehension test for posterior dislocation: With the patient supine

and the arm in the throwing position, apply downward pressure to the humerus

as if you are trying to dislocate the shoulder posteriorly. Pain and/or

apprehension are positive signs.

Note that anterior dislocation is much more common than posterior and/or inferior

dislocations. You can start the shoulder P.E with the patient sitting or standing and do all the

maneuvers except for relocation and release tests and posterior apprehension test. If you need

to complete these tests, ask the patient to lie down and perform them. The less you move the

patient the more organized your approach will be.

Page 93: CANADIAN IMGS GUIDE TO OSCE AND PRACTICE · in an OSCE scenario You can use this study guide to prepare for the NAC OSCE and MCCQE2. The content was reviewed for the most recent updates;

93

Yergason’s Test: Biceps is the main supinator. In this test, you stand in front of

the patient, shake the patient’s hand, and ask the patient to supinate it while

you resist them (ie. try to pronate their arm). Pain suggests bicipital tendinitis.

I will examine both shoulders but will focus on the right for the sake of the exam. I will then

examine the neck and the elbow and the neurovascular compartment of the upper limbs. I will

be looking, feeling and moving your shoulders, please let me know at any point if you feel

uncomfortable. I start P.E with inspection, comparing both sides looking from the front, back

and sides; no swelling, erythema, muscle atrophy, no deformities and no skin changes. I will

now palpate the shoulder, noticing any temperature difference, and there isn’t any. I’m starting

at the sternoclavicular joint, palpating along the clavicle, till the acromioclavicular joint, feeling

the coracoid, the humeral head, the bicipital tendon, spine of the scapula, supraspinatus,

infraspinatus, and medial border of the scapula. There is no tenderness or swelling. Next I will

test active movements. Can you please copy my movements and let me know if you feel pain.

Flexion, extension, abduction (I notice there is no painful arc), adduction, internal rotation,

external rotation. There is no pain or limitation of movement. Can you relax your shoulder and

let me move it for you? Please let me know if you feel any pain. Flexion, extension, abduction,

adduction, internal rotation, and external rotation. Next I will complete resisted rotator cuff

testing. I will test the power of the supraspinatus rotator cuff muscle. Can you bend your hands

at the elbows like this and try to move the elbows out while I resist you? Next I will test the

power of infraspinatus and teres minor. Can you go like this for me and resist me as I try to push

your arms towards your stomach. Next I will test the power of subscapularis. Can you bring your

hand behind your back like this and try to lift off my hand as I push down on yours. I will move

next to other special tests starting with the dropped arm test looking for supraspinatus tears.

Can you please lift your arms to the side all the way above your head then drop them back to

the sides slowly. The test is negative. Next I will do Yergason’s test to detect bicipital tendinitis.

Can you twist your arm this way while I resist you as I hold your hand. The test is negative. Next

is the Empty Can test for the supraspinatus tendon. Can you please go like this and resist me as

I push down on your arms. The test is negative. I will now test for inferior shoulder dislocation.

I’m going to pull your arm downwards. The sulcus sign is negative. Next I will do the anterior

apprehension test for anterior shoulder dislocation. Can you go like this please as if you are

throwing a ball, I’m going to press on your shoulder let me know if it’s painful or uncomfortable.

The test is negative. Can you lie down for me please? I’m doing the relocation test for anterior

shoulder instability by applying anterior pressure to the humerus and seeing if there is a feeling

of relief. I notice that the test is negative. I pull my arm quickly in the release test. No

Page 94: CANADIAN IMGS GUIDE TO OSCE AND PRACTICE · in an OSCE scenario You can use this study guide to prepare for the NAC OSCE and MCCQE2. The content was reviewed for the most recent updates;

94

apprehension or pain so the test is negative. Lastly I will test for posterior shoulder instability by

doing the posterior apprehension test and the test is negative.

Back P.E;

Back pain is very common, it is very important to keep in mind the red flags for back pain;

1- Age >50

2- Previous or current history of malignancy

3- Stool incontinence or urinary retention

4- Saddle anesthesia

5- IV drug use

6- Motor or sensory deficit

7- Constitutional symptoms (significant unexplained weight loss, night sweats,

fever)

8- Immunosuppression

9- Steroid use

I will focus in this section on P.E for patients with low back pain. It is very important to examine

the motor and sensory systems of the lower limb, and to mention that you would do a rectal

exam. It is helpful to practice the back exam to ensure you can fit it in 4 minutes.

- Ask for permission then wash your hands and respectfully drape the patient by taking

the gown off, keeping pants and shorts on, and keeping the bra on in female patients.

Mention that you would examine the hips and the neurovascular compartment of the

lower limbs and would do a rectal exam at the end

Page 95: CANADIAN IMGS GUIDE TO OSCE AND PRACTICE · in an OSCE scenario You can use this study guide to prepare for the NAC OSCE and MCCQE2. The content was reviewed for the most recent updates;

95

- Inspection: With the patient standing look for SEADS: Swelling, erythema, muscle

atrophy, deformities (kyphosis, scoliosis, lumbar lordosis), skin changes (especially tuft

of hair and Café-au-lait spots). Inspect the gait; ask the patient to walk, turn around

and come back, walk on tip toes and on heels and observe the gaits’ smoothness, arm

swing, balance, any antalgic gait. Make sure to walk along with the patient to support

him/her in case of a fall.

- With the patient standing for inspection do the following tests

Occiput to wall distance: Ask the patient to stand with his/her back against the

wall and the heels touching the wall and measure the distance between the

occiput and the wall. The occiput should touch the wall. The distance is

increased in Ankylosing Spondylitis (AS).

Rib- pelvis distance: Measure the distance between the lower ribs margin and

the upper hip in the midaxillary line; >2cm is suggestive of vertebral fracture.

Modified Schober test: With the patient standing straight, ask the patient if

you can draw a horizontal line between the 2 posterior superior iliac spines

(Dimples of Venus). Then mark 2 points in the midline, one 10 cm above the

line and one 5cm beneath. Ask the patient to bend forward with the legs

straight and measure the distance between the 2 midline points. An increase

of at least 3-4 cm is expected, less is seen in AS.

Chest expansion: Measure the chest circumference at the level of the

diaphragm during inspiration and expiration ( normal is 5cm or more)

- Palpation: Palpate the whole spine with the dorsum of your hand for temperature.

Then feel the spinous processes with your palm. Notice any tenderness or deformity.

Palpate the paravertebral muscles for spasm. Then palpate the iliac crest, posterior

superior iliac spine, sacroiliac joints, anterior superior iliac spine, and indicate that you

would palpate the inguinal lymph nodes and pubic symphysis (the examiner will stop

you, but you need to mention it for completeness). If you are unsure, you can ask the

examiner if they want you to complete a particular examination.

- Percussion: Make a fist; let the patient know that you are going to tap on his/her back.

Percuss the whole spine for tenderness.

- Range of motion: Stand by your patient and demonstrate the active ROM first. Then,

ask the patient to do the following movements:

Page 96: CANADIAN IMGS GUIDE TO OSCE AND PRACTICE · in an OSCE scenario You can use this study guide to prepare for the NAC OSCE and MCCQE2. The content was reviewed for the most recent updates;

96

Forward flexion: ask the patient to bend forward as much as he/she can

without bending his/her knees.

Extension: Bend backwards

Side or lateral flexion: Ask the patient to bend to the right and left by sliding

his/her hand down his/her leg.

Rotation: Ask the patient to sit down and rotate to each side. Sitting prevents

you from involving the hips and ensures it is the thoracolumbar area you are

looking at.

Evaluate the cervical spine in the same way you completed the lumbosacral

spine. Ask the patient to bend their head so their chin touches their chest

(forward flexion), bend the head back (extension), touch their ear to each

shoulder (or as close as possible ) (lateral flexion) and “shoulder check” or look

to each side (lateral rotation).

- Special tests: If the patient complains of back pain radiating down one leg, you should

complete one or more of the following especially straight leg raise test;

Straight leg raise test: With the patient lying supine, elevate one leg and see if

the patient develops back or leg pain. Pain at <60⁰ is indicative of sciatica.

Crossed straight leg raise test: Elevation of one leg causes pain of the

contralateral limb, also indicative of sciatica.

Femoral stretch test: The patient lies prone, and with the knee flexed, the hip

is lifted into extension. Positive test reveals anterior thigh pain/back pain and

can reflect higher lumbar nerve root irritation.

- Motor and sensory examination of the lower limbs: Refer to page 29 chapter 3 for

power and reflexes examination, and to page 32 chapter 3 for sensory exam.

- If you still have time, feel the pulses of the lower limbs; palpation of pulses is detailed

in chapter 3 page 10

Page 97: CANADIAN IMGS GUIDE TO OSCE AND PRACTICE · in an OSCE scenario You can use this study guide to prepare for the NAC OSCE and MCCQE2. The content was reviewed for the most recent updates;

97

- Less commonly, if the patient complains of nerve discomfort or weakness of the upper

extremity and you have concerns about their cervical spine, perform motor and

sensory examinations of the upper limbs as described for the lower limbs.

- Indicate again that you would do a rectal exam

I start P.E of the back with inspection; no swelling, no erythema, no muscle atrophy, no

deformities namely no scoliosis, no lordosis or kyphosis or any other deformity. No skin

changes. Next I will palpate. I’m going to feel and tap on your back let me know if it’s painful.

I’m feeling for temperature difference over the spine. Then palpating the spinous processes and

paravertebral muscles, no tenderness and no muscle spasm. I’m palpating the iliac crest, the

posterior superior iliac spine and the sacroiliac joint, also feeling the anterior superior iliac

spine. I will check for inguinal lymphadenopathy and palpate the pubic symphysis. The

examiner will say “pass”. I’m percussing the spine looking for any tenderness. Can you walk for

me please? The gait is smooth with normal stance and arm swing, no antalgic gait. Can you turn

around and come back? Can you please walk on your tip toes? (S1) and your heels? (L4). Next I

will measure the occiput wall distance. Can you please stand with your back against the wall

and your heels touching it? There is no space between the occiput and the wall, as expected in

normal individuals. Next I will do the modified schober test. Do you mind if I put a small mark

on your back? I’m marking the horizontal line between the 2 superior posterior iliac spines, and

in the midline marking one point 10 cm above and another one 5 cm below. Can you bend

forward, keeping your legs straight? I’m measuring the distance between the 2 midline points.

It is 20 cm, so increased by more than 3 cm and this is normal. With the patient standing I will

measure the fingerbreadths between the ribs and hip. It is 4 (more than 2 is normal). Now I will

test the active ROM: Can you bend forward? Forward flexion. And backwards? Extension. Can

you bend to the side like this? Lateral flexion and the other side please?. Can you sit down

please? Can you rotate to the left and right like this? Thoracolumbar rotation is normal. Can

you take your chin and touch it to your chest as you bring your head down? Normal cervical

forward flexion. Can you extend your head back as far as it comfortably goes? Extension of the

cervical spine is normal. Can you look to each side as far as you can? Normal lateral rotation of

the cervical spine. Can you bend your head, trying to touch your ear to your shoulder for each

side? Lateral flexion of the cervical spine is normal. Can you lie down please? I will do straight

leg raise test. I’m going to raise your left leg let me know if it hurts anywhere. No pain in the

back and leg with the lower limb elevated >60⁰. I will now do the crossed straight leg test, lifting

the right leg straight, noticing if the patient has any pain in the left leg. The test is negative.

Next I will do the femoral stretch test. Can you roll over onto your abdomen, and I am going to

bend your knee and lift your hip up? Let me know if it hurts anywhere. There is no pain with left

Page 98: CANADIAN IMGS GUIDE TO OSCE AND PRACTICE · in an OSCE scenario You can use this study guide to prepare for the NAC OSCE and MCCQE2. The content was reviewed for the most recent updates;

98

hip extension, the test is negative. Next I will test the power of the lower limbs. I’m going to

move your legs to test the strength of your muscles. Resist me please. Hip flexion, abduction and

adduction; knee flexion and extension; ankle dorsiflexion and plantar flexion are normal. I will

test the reflexes of the lower limbs. I’m going to test your nerves, let me know if it hurts. Ankle

jerk S1, S2, Knee Jerk L4,5. I will do the other side as well. Next I will test light touch of the

lower limbs with a cotton ball. Do you feel this? Close your eyes and say yes each time you feel

it. Let me know if it feels different on either side. L1 comparing both sides, L2, L3, L4, L5, and S1.

Finally I would do a rectal exam and examine the hips

Hips P.E

- Ask for permission, wash your hands and respectfully drape the patient by taking the

gown off and keeping his/her shorts on. Indicate that you will examine both sides but

will focus on the affected side for the sake of the examination. You will also examine

the back, knees and the neurovascular compartment of the lower limbs

- Inspection: Compare both sides, looking from the front, back, and sides for SEADS;

swelling erythema, atrophy, deformity, skin changes. Ask the patient to walk (make

sure you support your patient) and look for any antalgic or Trendelenberg gait. Do the

Trendelenburg test with the patient standing; stand behind the patient, place your

hands on the iliac crest and ask the patient to stand on one leg at a time. Test both

sides. The unsupported (non-weight bearing) side stays up in normal individuals but

drops in a positive test, i.e. the affected side (the one the patient is weight bearing on)

has a weak abductor (gluteus medius or minimus) or nerve palsy (superior gluteal

nerve palsy).

- Palpation: Feel for temperature difference, and then palpate the iliac crest, anterior

superior iliac spine, posterior superior iliac spine, sacroiliac joint, greater trochanter

and bursa. Mention that you would palpate the inguinal ligament and symphysis

pubis, the examiner will stop you.

- ROM: Start with active range of motion doing the first 2 movements with the patient

standing and the rest lying down. Ask the patient to copy your movements:

Extension: ask the patient to move each leg backwards with the knees straight

Page 99: CANADIAN IMGS GUIDE TO OSCE AND PRACTICE · in an OSCE scenario You can use this study guide to prepare for the NAC OSCE and MCCQE2. The content was reviewed for the most recent updates;

99

Rotation: ask the patient to make a circle with each foot

Flexion: ask the patient to bring his knees to his chest (supine)

Abduction: ask the patient to move his leg outwards with the knee straight

Adduction: ask the patient to move the leg inwards all the way crossing the

opposite leg with the knee straight.

Then ask the patient to relax, and move his hip in the above-mentioned directions. For passive

internal and external rotation, you will bring the patient’s knee to approximately 90 degrees,

and then internally and externally rotate it at this point to passively evaluate hip rotation. If

they have knee problems, you can log-roll the resting leg in the same motions of internal and

external rotation. You may want to ask the patient to lie on his side to test passive extension.

Next, you can test the power by asking the patient to resist you as you repeat the above

movements.

- Special tests: Done with the patient lying down:

Leg length: Using a tape measure, measure the true leg length (from anterior

superior iliac spine to medial malleolus). Compare both sides, then measure

apparent leg length (from the umbilicus to the anterior superior iliac spine)

Patrick’s test: With both knees extended, place the foot of one leg over the

contralateral knee. Support the hip of the extended limb while you try to lower

the other limb to the same level, which is normal.

Thomas test (Fixed flexion deformity): Place your hand under the lumbar spine

with the palms facing up, then passively flex the patient knee to his/her chest.

The opposite leg should remain straight; if not, it suggests fixed flexion

deformity of that hip.

I will examine both hips but will focus on the right for the sake of time. I will also examine the

lumbar spine and the knee and the neurovascular compartment of the lower limbs. I start with

inspection looking from the front, back and sides, comparing both sides. No swelling, erythema,

muscle atrophy, deformity or skin changes. Can you walk for me please? The gait is smooth, no

antalgic gait and no trendelenberg gait. I’m going to feel your hip. Let me know if it hurts

anywhere. I’m palpating the iliac crest, the posterior superior iliac spine, the sacroiliac joint, the

greater trochanter and its bursa, the anterior superior iliac spine, and will also palpate the

inguinal ligament and pubic symphysis. The examiner will say “pass”. With the patient standing,

Page 100: CANADIAN IMGS GUIDE TO OSCE AND PRACTICE · in an OSCE scenario You can use this study guide to prepare for the NAC OSCE and MCCQE2. The content was reviewed for the most recent updates;

100

I will do the trendelenberg test, placing my hands on the iliac crests. Can you stand on one foot

please? And now the other? The test is negative. Next is the active range of motion. Can you

copy my movements? Extension, Rotation. Can you lie down please?. Can you bring your knees

to your chest? Flexion. Can you keep your leg straight and bring it out? Abduction. Can you

bring it all the way in crossing the other leg? Adduction. Can you relax and let me move your

leg. I’m testing the passive ROM: flexion, abduction, adduction, internal and external rotation.

Can you lie on your left side? And extension. Can you lie again on your back?. I will test the

power at the end. I will move now to special tests. I’m measuring the true leg length from the

anterior superior iliac spine to medial malleolus. And will measure the other side. Both are the

same (you can give the number) then I’m measuring apparent leg length from the umbilicus to

medial malleolus, no difference. Next I will do Patrick’s test looking for sacroiliac joint

pathology, or Iliopsoas spasm. The test is negative on the right. Then I will do the Thomas test

looking for any fixed flexion deformity of the hip and the test is negative on the right.

Knees P.E;

Knee pain and injuries are very common. It is useful to review Basic anatomy of the knee so

that P.E makes sense. Follow the link below for basic knee anatomy;

http://www.athleticadvisor.com/Injuries/LE/Knee/knee_anat.htm

- Ask for permission, wash your hands and respectfully drape the patient by exposing

both lower limbs to mid thighs

- Start by stating that you will examine both knees but will focus on the affected one for

the sake of time. State that you will examine the hips and ankles and the

neurovascular compartment of the lower limbs.

- Inspection: Done with the patient standing, walking and lying down. Look from the

front, back and sides for SEADS; Swelling, Erythema, Muscle atrophy, Varus or valgus

deformity, and skin changes. Ask the patient to walk and support him/her if necessary.

Look for antalgic gait or limitation of movement. Ask the patient to stand on one leg at

Page 101: CANADIAN IMGS GUIDE TO OSCE AND PRACTICE · in an OSCE scenario You can use this study guide to prepare for the NAC OSCE and MCCQE2. The content was reviewed for the most recent updates;

101

a time and see if he/she can’t bear weight (make sure you are there for your patient).

Ask the patient to squat down and up if they can and see if there is any pain or

limitation in their ROM. Ask the patient to lie down and look at the knees again. Ask

the patient to flex his/her knees and look behind the knee for sag.

- Palpation: Feel for temperature difference. Then palpate the knee for swelling and

tenderness feeling the following structures:

Quadriceps tendon

Patella and patellar tendon

Tibial tuberosity

Head of Fibula

Medial and lateral joint lines

Medial collateral ligament

Lateral collateral ligament

Popliteal fossa

- ROM: The knee moves in 2 directions: flexion and extension. Do active and passive

ROM and test the power. Notice any limitation of movement, pain or crepitation.

- Special test:

In this section you will test the main ligaments shown in Figure 3-5 in addition to knee effusion.

Make sure you practice and master the techniques.

Effusion: Do the test you feel more comfortable with. You can do patellar tap

where you milk the fluid down from the lower thigh into the knee, keep your

hand above the knee and tap the patella. The other test is called the milk or

wipe test where you basically milk/move fluid from the medial to lateral

compartments of the knee with one hand, and look for any bulge on the

medial side as you milk/push the fluid from the lateral side back to the medial

side. This only works with small effusions. If there is a larger effusion, you can

also perform the Ballottement test where you move fluid from the

suprapatellar pouch (lower thigh) of the knee to the main part of the knee. You

are feel fluid shift back and forth between your hands which confirms swelling.

You can look for swelling in the back of the knee (ie. Baker’s cyst) when you are

Page 102: CANADIAN IMGS GUIDE TO OSCE AND PRACTICE · in an OSCE scenario You can use this study guide to prepare for the NAC OSCE and MCCQE2. The content was reviewed for the most recent updates;

102

inspecting the patient and they are standing. Look behind their knee (ie.

Popliteal space) and see if it looks distended or red suggesting swelling.

Anterior Drawer sign for anterior cruciate ligament stability: With the patient

lying supine, flex his/her knee to 90 degrees and sit on the ipsilateral foot. Pull

the leg forward by holding it with both hands just below the knee. Note any

excessive movement to sugesst ACL instability.

Posterior drawer sign for posterior cruciate ligament instability: Similar to

anterior drawer sign but you push backwards this time. Note any excessive

movement.

Testing medial and lateral collateral ligaments: Flex the knee 20⁰ and apply

valgus stress to the knee to test for medial collateral ligament instability and

varus stress to test for lateral collateral ligament instability. Excessive

movement implies a positive test.

McMurray test for medial meniscus injury: Place one hand on the knee joint

line and one hand on the foot. Fully flex the knee, and externally rotate the

foot as you slowly extend the knee. A snapping sensation indicates a positive

test.

McMurray test for lateral meniscus injury: Same maneuver as above but

internally rotate the foot.

I will examine both knees but will focus on the right for now. I will also examine the hips and

ankles and the neurovascular compartment of the lower limb. I start P.E by inspecting the knees

with the patient standing comparing both sides, looking from the front, back and sides; no

swelling, no erythema, no deformity no muscle atrophy, and no skin changes. Can you walk for

me please? No antalgic gait. Can you squat for me and then come back up? I do not see any

abnormal movement. Can you lie down and bend your knees? I’m inspecting the knees again

looking for any sag and there isn’t. I’m going to feel your knees, let me know if it hurts

anywhere. I’m palpating the knees for temperature difference and there isn’t. I’m now feeling

for any swelling or tenderness; feeling the quadriceps tendon, patella, patellar tendon, medial

collateral ligament, lateral collateral ligament, tibial tuberosity, head of femur, and popliteal

fossa. No abnormality. Next is ROM, I will start with active ROM. Can you bring your knees to

your chest then fully straighten them. ROM is full with no limitation, pain and I feel no crepitus

in the medial, lateral and patellofemoral compartments. Can you relax your knees while I move

them this time? Flexion and extension. Then I will test power. Resist me as I bend your knee.

Page 103: CANADIAN IMGS GUIDE TO OSCE AND PRACTICE · in an OSCE scenario You can use this study guide to prepare for the NAC OSCE and MCCQE2. The content was reviewed for the most recent updates;

103

Resist me as I straighten it. I will now do some special tests starting with patellar tap looking for

effusion. Its negative, and the milk/wipe test also looking for effusion is negative. Next I will

test the cruciate ligaments. Can you bend your knee please? With the knee flexed to 90⁰, I’m

doing the anterior drawer test for anterior cruciate ligament and the posterior drawer test for

posterior cruciate ligament. Both tests are negative. Next I will check the collateral ligaments,

flexing the patient knee 20⁰ and applying varus stress looking for lateral collateral ligament

instability and valgus stress for the medial collateral ligament. The test is negative. Next I will do

the McMurray test, fully flexing the patient knee, and externally rotating the foot looking for

medial meniscus pathology as I extend the knee. The test is negative. Now I’m fully flexing the

knee, internally rotating the foot and extending the knee looking for lateral meniscus tear. The

test is negative.

Ankles P.E

- Ask for permission, wash your hands and respectfully drape the patient exposing the

legs and feet

- Indicate that you would examine both ankles, but will focus on the affected for the

sake of time. State that you will examine the knees, and the neurovascular

compartment of the lower limbs.

- Inspection: Done in 3 positions including standing, walking and lying down. With the

patient standing look from the front, sides, and back for SEADS; swelling, erythema,

muscle atrophy, deformity, and skin changes. Slide your hand under the foot and

noticed if the patient has high arch or flat foot. Ask the patient to walk and observe

for dropped foot or inability to bear weight on one foot. Ask the patient to walk on tip

toes and heels (always support your patient when you ask him/her to walk). With the

patient lying down, look between the toes for any skin changes, notice any nail

changes, and check for edema, look at the sole and heels for ulcers.

- Palpation: Feel for temperature difference at the ankle and foot. Then palpate for

masses or tenderness feeling 6 cm above each malleoli and feeling the medial and

lateral malleolus, the Achilles tendon (note continuity), the calcaneus, base of fifth

Page 104: CANADIAN IMGS GUIDE TO OSCE AND PRACTICE · in an OSCE scenario You can use this study guide to prepare for the NAC OSCE and MCCQE2. The content was reviewed for the most recent updates;

104

metatarsal, navicular, metatarsal heads. Compress the forefoot between thumbs and

fingers for MTP tenderness. Feel the posterior tibial and dorsalis pedis pulses

- Range of motion: Do active and passive ROM and test the power. Ankle moves in 4

directions; Dorsiflexion, plantarflexion, inversion, eversion.

- Special tests: Ask the patient to lie prone with the ankle hanging at the edge of the

examining table and do the following tests for Achilles tendon:

Palpate the Achilles tendon for gaps in this position.

Calf squeeze test: squeeze the calf - the normal response is ankle plantar

flexion. In the case of Achilles tendon rupture, there will be no movement.

Knee flexion text: Ask the patient to flex his/her knees while lying prone and

measure the angle between the ankle and the leg. If the Achilles tendon is

intact, the ankles remain slightly plantarflexed and the angle is >90⁰. If it is

ruptured, the angle is <90⁰.

Anterior Drawer Test: In the case of an ankle sprain, a common test to evaluate

for anterior talofibular ligament rupture is the Anterior Drawer test. Position

the patient’s foot in slight plantar flexion, brace the anterior shin with the

other hand, pull the heel anteriorly with main hand and check for laxity. There

should be limited movement.

Useful Tip;

Follow the link below for Ottawa Ankle rule;

http://www.ohri.ca/emerg/cdr/docs/cdr_ankle_poster.pdf

I will examine both ankles but will focus on the right for the sake of the exam. I will also

examine the knees and the neurovascular compartment of the feet. I start P.E inspecting the

feet in the standing position, comparing both sides, looking from the front, sides and back; no

swelling, no erythema, no muscle atrophy, no valgus or varus deformity or any other deformity,

no skin changes. I will slide my hand under your foot, let me know if it’s uncomfortable. No high

ach or flat foot. Can you walk for me? No drop foot or antalgic gait. Can you walk on your tip

toes? And on your heels. Can you lie down please? I’m inspecting the plantar surface and heels

for ulcers, looking between the toes, and inspecting the nails, no changes. I will now palpate the

ankles, checking first for temperature difference and there is no difference. Now I will palpate

Page 105: CANADIAN IMGS GUIDE TO OSCE AND PRACTICE · in an OSCE scenario You can use this study guide to prepare for the NAC OSCE and MCCQE2. The content was reviewed for the most recent updates;

105

for tenderness and swelling; the medial malleolus, the lateral malleolus, the Achilles tendon,

calcaneus, the base of the fifth metatarsal, the navicular, the metatarsal heads. I’m squeezing

the forefoot looking for MTP tenderness or swelling. Next is the range of motion. Can you bring

your foot up? Dorsiflexion. Now down as if you are pressing on the gas pedal of a car? Plantar

flexion. Can you bring your foot out like this? Eversion. And in? Inversion. I will now do the

passive range of motion. Can you relax your foot for me? Plantarflexion, dorsiflexion, inversion,

eversion. Next I will test the power. Can you resist me as I move your foot? Doing the same

movements, the power is 5/5. Next I will do special tests. Can you lie on your tummy for me and

hang your foot off the edge of the bed?. I’m palpating the Achilles tendon, there are no gaps; I

am doing the squeeze test, noticing plantar flexion - the test is negative. Can you bend your

knees? With the knees flexed 90⁰, I notice slight plantar flexion, the knee flex test is negative.

Can you relax your foot while I check how stable the ankle is? Anterior Drawer test is negative

for a ligamentous sprain or tear

Breast Physical Examination;

Edited by; Dr. Sadik Salman

The MCC website specifies that you will not be asked to perform a genital or rectal P.E, but

doesn’t say so for breast examination. You may encounter a male or female breast exam.

- Ask permission, wash your hands and respectfully drape the patient by exposing the

chest (ask female patient to take her bra off). Keep the patient covered from the

umbilicus down.

- Make sure vital signs are stable and the patient is comfortable.

- Inspection and palpation need to be done in two positions; sitting and standing

- With the patient sitting;

Inspection; start with the patient hands resting on the thighs, look from the

front and sides noticing the breasts size and symmetry, changes of the shape,

skin redness, Peau’ d’orange, vascularity or skin rash or ulceration. Look at the

nipples noticing any retraction or distortion, inversion or nipple discharge.

Look for swelling or ulcerations of the areola. Inspect the axilla for masses,

Page 106: CANADIAN IMGS GUIDE TO OSCE AND PRACTICE · in an OSCE scenario You can use this study guide to prepare for the NAC OSCE and MCCQE2. The content was reviewed for the most recent updates;

106

redness or hyperpigmentation. Ask the patient to press against his/her hips

and look for any retraction. Ask the patient to put his/her hands behind his/her

back and inspect again. Pay particular attention for the inferior side of the

breasts.

Palpation; You will palpate for 3 things;

1- Temperature; with the dorsum of your hand notice any temperature

difference

2- Palpate the breasts; start with the normal first. Palpate from the

midaxillary line between 2nd and 6th ribs to mid sternum. Divide the breast

into 4 quadrants (superior outer and inner, inferior outer and inner), use

the palmar surface of the tip of your fingers and make small clockwise

circles as you feel each quadrant. Feel under the breasts, ask the patient to

lift her breast if needed. Look for tenderness, masses. If a mass is felt

determine exact location, consistency, shape, edges, size, temperature,

tenderness and mobility. Note if the mass is attached to the skin or the

underlying muscles.

3- Lymph nodes (L.N); Palpate the supra and infra clavicular L.N, and the

axillary lymph nodes, note any tenderness. If you feel something try to

determine how many nodes, the size, consistency, exact location and

tenderness. Ask the patient to tilt his/her head down and to the right for

right supraclavicular L.Ns. and down and to the left for the left ones. It

makes it easier and more comfortable for the patient. For axillary L.Ns ask

the patient to relax his/her arm on top of yours with the elbows flexed,

slightly flex and abduct the patient’s arm.

- With the patient lying down

Inspect again; both breasts and axilla

Palpate only the breasts using the same technique

- Pay particular attention to the patient’s comfort and privacy

I’m going to start a breast examination; Please let me know if you feel uncomfortable. Vital

signs are stable, I’m comparing both sides, looking from the front and sides noticing that both

sides are symmetrical, no swellings, redness, Peau’ d’orange, ulceration or skin rash, no nipple

deformity or retraction, no nipple discharge. I’m inspecting the axilla; no masses, no redness.

Can you put your hands on your hips like this and press? I’m inspecting the breasts for any

Page 107: CANADIAN IMGS GUIDE TO OSCE AND PRACTICE · in an OSCE scenario You can use this study guide to prepare for the NAC OSCE and MCCQE2. The content was reviewed for the most recent updates;

107

retraction or any of the signs I just mentioned. Can you put your hands behind your head?

Inspecting the breasts and axilla. I’m going to feel your breast for masses please let me know if

it feels uncomfortable. I’m going now to palpate with the patient sitting; Feeling for

temperature difference and I don’t appreciate any. Now palpating the right breast starting from

the right upper outer quadrant, right lower outer quadrant, lower inner and upper inner; no

masses or tenderness. I will now palpate the left breast. Next I will palpate the right

supraclavicular nodes, infraclavicular nodes, now I will palpate the left side. Next I will palpate

the right axillary lymph nodes, can you relax your arm on mine; I’m feeling the medial group,

lateral, and superior, inferior, apical. Next I will feel the left axillary L.N. No palpable nodes.

Can you lie down please? With the patient lying down I’m inspecting again, no abnormalities,

and I will palpate the breasts again in this position. Let me know if you feel uncomfortable. No

masses, no tenderness.

Fundoscopy;

Edited by; Dr. Sadik Salman

- Use the rule of right, right, right when you do ophthalmoscopic examination; hold the

ophthalmoscope with your right hand, use your right eye to examine the patient’s

right eye

- Ask permission, wash your hands, no draping is warranted

- Ask the patient to focus on a point on the wall, dim the light. Start looking in the

ophthalmoscope 1 foot away adjusting the focusing wheel as needed. Note the red

reflex and come close to the eye at 45⁰ (between an imaginary vertical and horizontal

planes). Come close to the patient as much as you can, you can use place the thumb of

the contralateral hand on the patient’s forehead to avoid hitting the patient with the

ophthalmoscope accidentally. Adjust the focusing wheel as needed until you see a

retinal vessel, follow the vessel to the optic disc, note the following;

Optic disc size, shape, margins, note any edema

Page 108: CANADIAN IMGS GUIDE TO OSCE AND PRACTICE · in an OSCE scenario You can use this study guide to prepare for the NAC OSCE and MCCQE2. The content was reviewed for the most recent updates;

108

Blood vessels; look at the retinal veins and arteries, note any tortuous vessels,

neovascularization and any AV nodding

The macula if you can find it

Retinal background; Look for any cotton wool exudates or flame hemorrhages

or abnormal pigmentation.

I’m going to do fundoscopic examination, dimming the light, using my right hand and eye to

examine the patient’s right eye, I notice a normal red reflex, and as I come closer I notice a

blood vessel that I’m following, now I see the optic disc and note that it has sharp borders, no

edema, normal shape and size. I’m looking at the veins and arteries, no AV nodding, tortuosity

or neovascularization. No flame hemorrhages or cotton wool exudates. The macula looks

normal

Page 109: CANADIAN IMGS GUIDE TO OSCE AND PRACTICE · in an OSCE scenario You can use this study guide to prepare for the NAC OSCE and MCCQE2. The content was reviewed for the most recent updates;

109

References;

1- Heart murmurs, Medical Exam essentials , accessed on May 1st/2014,

http://www.medical-exam-essentials.com/heart-murmurs.html

2- Elevated JVP, Clinical exams, accessed May 1st/2014,

http://clinicalexams.co.uk/images/elevated_jvp.jpg

3- Simel, David L. The Rational Clinical Examination Evidence-based Clinical Diagnosis. New

York: McGraw-Hill Medical, 2009.

4- Macleod, John. Macleod's Clinical Examination. 12th ed. Edinburgh: Churchill

Livingstone/Elsevier, 2009.

5- McGee, Steven R. Evidence-based Physical Diagnosis. 3rd ed. Philadelphia:

Elsevier/Saunders, 2012.

6- Douglas Gelb, The detailed neurological examination in adults, In: UpToDate, Topic 5095

Version 5. , Waltham, MA. ,Accessed on October 25/214,

http://www.uptodate.com/contents/the-detailed-neurologic-examination-in-

adults?source=search_result&search=neurology+exam&selectedTitle=1~150

Page 110: CANADIAN IMGS GUIDE TO OSCE AND PRACTICE · in an OSCE scenario You can use this study guide to prepare for the NAC OSCE and MCCQE2. The content was reviewed for the most recent updates;

110

Page 111: CANADIAN IMGS GUIDE TO OSCE AND PRACTICE · in an OSCE scenario You can use this study guide to prepare for the NAC OSCE and MCCQE2. The content was reviewed for the most recent updates;

111

Chapter 4

Writing and

Counseling Tips Edited by; Dr. Anca Tapardel

Page 112: CANADIAN IMGS GUIDE TO OSCE AND PRACTICE · in an OSCE scenario You can use this study guide to prepare for the NAC OSCE and MCCQE2. The content was reviewed for the most recent updates;

112

Introduction;

This chapter will cover common writing tasks that you will do on a daily basis as a resident.

They may also be tested in the writing part of MCCQE2. All written information becomes a legal

document. It is important to be organized and write legibly. You will not be asked to type on a

keyboard in Canadian exams, and will always be provided with a pen/pencil when you sign in.

Familiarize yourself with the Canadian style by going over the detailed examples of writing;

- Admission orders

- Progress Note

- Discharge summary

- A letter to an employer

- A prescription

- A referral letter

The next section of the chapter is about counseling. You may counsel your patient about

anything from smoking to laboratory results to life threatening diagnoses. Counseling is a very

important part in communication with our patients. Detailed examples are provided in chapter

5.

Sample writing scenarios

Admission orders

You may be given a scenario, or asked to write orders for a patient you just assessed. Make sure

you write down the patient name, hospital number and date of birth. Usually this information is

provided by labels. Remember to date and sign the orders and leave your pager number with

your name written clearly at the end. Most of the hospitals have template orders to help

standardize the charts.

In the body of the orders you need to be specific as to whom and where the patient is

admitted. Clearly state the diagnosis. Then list their recommended diet, activity level, IV fluids

Page 113: CANADIAN IMGS GUIDE TO OSCE AND PRACTICE · in an OSCE scenario You can use this study guide to prepare for the NAC OSCE and MCCQE2. The content was reviewed for the most recent updates;

113

and oxygen if needed, and the frequency with which you want their vital signs checked. Then

list the investigations and medications and indicate if any consult is warranted.

Organize investigations into blood work and imaging or others. Write the medication name,

dose, route and frequency of administration and if it is given around the clock or as needed

(prn). If you want your patient to continue her/his home medications, add them to the

admission medications.

A consult sheet request is filled by the physician (example provided in this chapter page 7). In

real life the admitting physician calls the consulted service. In rare cases the consult is faxed,

and in this case you should order the consult to be faxed. Anyhow, it is a good idea to indicate

what service is consulted and if they were called already or if the consult needs to be faxed.

This way the medical team know exactly what is being done on admission by just looking at the

orders.

Note; this example is only for demonstration. In most surgical cases in real life general surgery

will be consulted by the emergency room physician and will admit the patient under their care.

Example;

You have just assessed Mr. Smith, a 30 year old male who presented to the emergency

department with right lower quadrant pain. He had guarding and tenderness of the right lower

quadrant, and was febrile. Other vital signs were normal. He had positive signs of appendicitis.

You were informed by the examiner that he had a normal rectal exam. His WBCs were elevated.

Write admission orders for Mr. Smith.

Patient name--------------

Hospital number---------

Date of birth--------------

Date;---------

- Admit Mr Smith under Dr (Name of admitting physician) to ward (Name or ward number)

- Keep NPO (nothing per mouth), except for medications with sips of water

- Activity as tolerated

- IV fluids; normal saline 0.9% 100cc/hour for 24 hours then reassess by physician

- Vital signs every 4 hours

Page 114: CANADIAN IMGS GUIDE TO OSCE AND PRACTICE · in an OSCE scenario You can use this study guide to prepare for the NAC OSCE and MCCQE2. The content was reviewed for the most recent updates;

114

- Blood tests; CBCD, Creatinine, Urea, electrolytes (Na, K, CL),

- Urine analysis

- Abdominal Ultrasound

- Medications;

- Tylenol 325mg P.O q 4 hours prn

- Gravol 50mg IV q 6 hours prn

- Consult surgery- Surgical team was informed

Your name, signature and pager number

Progress Notes;

You have probably heard about the famous SOAP format for progress notes. It stands for;

S; Subjective (what the patient is complaining from)

O; Objective (Physical exam findings and laboratory results)

A; Assessment (Your assessment of the patient status)

P; Plan (management plan, including changing medications, or ordering further investigations

as well as the need for physical or occupational therapy or consults and the discharge plan)

Example;

Mr. Smith is a 40 year old male admitted with right middle lobe pneumonia. He was febrile,

with oxygen saturation of 92% on room air, respiratory rate of 23/min and strongly coughing

yellow-green non bloody sputum on admission. His WBCs were 16. When you saw him today he

said he is feeling much better and that his cough is not as bad. You still hear right mid lung zone

crackles, and he is afebrile with oxygen saturation of 98% on room air and respiratory rate of

Page 115: CANADIAN IMGS GUIDE TO OSCE AND PRACTICE · in an OSCE scenario You can use this study guide to prepare for the NAC OSCE and MCCQE2. The content was reviewed for the most recent updates;

115

14/min. His CXR is still the same with right middle lobe consolidation. And his sputum gram

stain and culture came back positive for Strep Pneumonia sensitive to Levofloxacin you

prescribed on admission. Blood cultures were negative.

Mr. Smith is a 40 year old male not known previously to have any chronic medical illnesses.

Admitted on (date) with right middle lobe community acquired pneumonia (CAP) and was

started on empiric Levofloxacin 750mg/ day P.O

S; He is feeling much better, his productive cough is improving no hemoptysis, no chest pain

O; His vital signs are all within normal. He still has right middle lobe crackles. His WBCs are now

11 from 16, and his sputum grew Strep Pneumonia sensitive to Levofloxacin. No complication

on his chest XR and the right middle lobe consolidation remains unchanged.

A; 40 years old previously healthy smoker male admitted with right middle lobe CAP due to

Strep Pneumonia sensitive to Levofloxacillin. Shows clinical and laboratory improvement, the

plan is;

-Continue Levofloxacin 750mg P.O daily for 7 days

-Continue to check vital signs every 6 hours

- Repeat the chest XR after 6 weeks to make sure the consolidation is fully resolved and there is

no underlying malignancy

- Counsel about smoking cessation

-Discharge plan; Plan to discharge home tomorrow with follow up with his family physician

At different levels in your training or practice you may choose to write more succinct notes.

Discharge summary

Each hospital has a form to fill in for discharge summary. You generally need to mention;

- Patient’s name, hospital number and date of birth

- Treating physician’s name

- Admission and discharge dates

- Names of all other physicians involved in patient care (It is important to fax a copy to

each one of them)

Page 116: CANADIAN IMGS GUIDE TO OSCE AND PRACTICE · in an OSCE scenario You can use this study guide to prepare for the NAC OSCE and MCCQE2. The content was reviewed for the most recent updates;

116

- Admission diagnoses

- Course in hospital including treatments given and any outstanding investigations

- Medical and surgical illnesses prior to admission

- Discharge medications

- Condition at discharge

- Follow up plan

- Recommendations for diet, activity

Example;

Will use the case of Mr. Smith page 3 of this chapter

-Patient name

Hospital number

Date of birth

-Admitting physician

Admission date

Discharge date

-Patient’s family Dr name (A copy is faxed)

-Admission diagnoses; Right middle lobe community acquired pneumonia

-Course in hospital and treatment; Mr Smith sputum grew Strep Pneumonia sensitive to

Levofloxacin. He was treated with Levofloxacin 750mg P.O daily during his hospital stay. His

Page 117: CANADIAN IMGS GUIDE TO OSCE AND PRACTICE · in an OSCE scenario You can use this study guide to prepare for the NAC OSCE and MCCQE2. The content was reviewed for the most recent updates;

117

chest XR showed right middle lobe consolidation. He improved clinically and didn’t have any

complications

- Medical illness; not known to have medical illnesses prior to admission, but is a smoker 1 pack

per day for 20 years (20pack-years)

-Discharge medications; Levofloxacin 750mg P.O daily till --------- (total 7 days)

-Discharge condition; stable, with normal vital signs including an oxygen saturation of 99% on

room air

-Follow up plan; Repeat chest XR in 6 weeks. Mr Smith doesn’t feel ready to quit smoking yet

but will call the clinic when he decides to. Mr Smith was advised to return to the emergency

department if he develops fever, chest pain, worsened cough or bloody sputum. He will follow

up with his family physician

-Diet and activity upon discharge; as tolerated

Letter to employer

Your patient may request a letter to his/her employer. This letter should be printed on a paper

with the hospital/clinic heading.

Example;

You assessed Mr. Smith, a 40 year old male in your clinic for back pain. After full investigations,

his pain deemed mechanical in origin, and no surgical intervention is warranted at this stage.

You think Mr. Smith would benefit from one week of rest. Mr. Smith works at a grocery store

Page 118: CANADIAN IMGS GUIDE TO OSCE AND PRACTICE · in an OSCE scenario You can use this study guide to prepare for the NAC OSCE and MCCQE2. The content was reviewed for the most recent updates;

118

where he often lifts heavy objects. Mr. Smith requests a letter to his employer for one week of

rest.

Clinic heading

Work absence certificate

Re; patient name and address

This letter is to certify (patient’s full name) was assessed in this clinic on (day and date) and was

unable to work due to illness/injury from (date) to (date)

Physician name and position

Date and sign

Writing a prescription

You need to include the patient name, age and health number (you can use a sticker), and the

Medication information; drug name, dose, route of administration, frequency and duration.

Sign and date the prescription and leave your pager number in case the pharmacist had any

questions.

Example;

You assessed Ms. Smith. A diabetes patient of yours who was found to have an uncomplicated

lower respiratory tract infection (cystitis). You decide to treat her as an outpatient so you

prescribe her an antibiotic and renew her diabetes medication.

Patient name

Date of birth

Health care number

Page 119: CANADIAN IMGS GUIDE TO OSCE AND PRACTICE · in an OSCE scenario You can use this study guide to prepare for the NAC OSCE and MCCQE2. The content was reviewed for the most recent updates;

119

1-Metformin 850mg P.O BID for 6 months

2-Ciprofloxacin XL 500mg P.O daily for 3 days

Date

Physician name and signature

Pager number

Writing a referral letter;

When requesting a consult from a specialist you need to ask a specific question. You need to

write the consultant a comprehensive assessment of the patient including a relevant history,

physical exam and investigations. Be sure to mention the pertinent positives and negatives. If

given investigation results in the test, write them down in your consult. I made up the example

below, so I will not detail investigations but will mention that a copy is attached. Again; write

down the investigations in the referral letter if given to you in the test.

Example;

You just assessed Mrs. Smith, a 35 years old female in your clinic. She is complaining of

recurrent swelling of her hands and fingers for the last six months. She is finding it harder to do

her job as a secretary and worried she has rheumatoid arthritis. Her mother had disabling RA .

You do a full history, physical exam and order some investigations. You decide to refer her to a

rheumatologist.

Re; Patient name, age, hospital number and contact information

To; Name and contact information of the consultant

Dear colleague;

I saw Mrs. Smith in my clinic on (date). She was complaining of recurrent MCP and IP joints

swelling that seem to be symmetrical. No other joints involved. She describes 60 min morning

stiffness that started 6 months ago when the swelling began. No skin rash, eye involvement,

Page 120: CANADIAN IMGS GUIDE TO OSCE AND PRACTICE · in an OSCE scenario You can use this study guide to prepare for the NAC OSCE and MCCQE2. The content was reviewed for the most recent updates;

120

headache, chewing or swallowing problems. No skin tightness or Raynaud’s phenomenon, no

photosensitivity and no hair loss.

Her past medical and surgical history is negative. There is a history of severe disabling

rheumatoid arthritis in her mom. She is on birth control bills and is not allergic to any

medication. She lives with her husband and not planning to have kids any time soon. Her

symptoms are affecting her performance at work. She works as a secretary downtown.

On physical exam she had normal vital signs. Her weight was---- and her height was---. She had

symmetrical swelling of the first MCP of both hands which was also tender. Other joints were

normal. No rheumatoid nodules. No deformities. No skin rash or eyes involvement. Cardiac,

pulmonary and abdominal exams were all normal.

Initial investigations included a normal CBCD, Creatinine, urea and electrolytes. Her RF, ANA

and ACCP are pending. XR of the hands showed 1st MCP soft tissue swelling and no bone

destruction. A copy of her investigations is attached.

Can you please assess her for definitive diagnosis and management?

Your help is much appreciated

Regards;

Your name, signature, date and contact information.

Counseling;

Counseling and patient education are very important aspects of patient care commonly tested

in Canadian OSCEs. These stations mainly aim at assessing your communication skills. Studies

have shown that patient education and counseling promote behavioral change. (1)

There are many ways to counsel a patient. You need to develop and practice your own style.

Key elements to keep in mind when counseling;

Explore the patient’s view;

- Patient’s medical condition

Page 121: CANADIAN IMGS GUIDE TO OSCE AND PRACTICE · in an OSCE scenario You can use this study guide to prepare for the NAC OSCE and MCCQE2. The content was reviewed for the most recent updates;

121

- Patient’s knowledge about his/her disease, intervention

- What information is the patient seeking?

- What are the patient’s expectations?

- Social support

Educate the patient about the subject (disease, intervention)

- Definition, mechanism, prevalence and causes

- Risk of the subject

- Options to act on the subject. Examples; treatment of disease, ways to quit smoking

- Complications and risks of acting

- Complications and risks of not acting

- Any alternatives

- What support systems are available?

- Make sure the patient understands the information

- Beware not to overwhelm the patient

- Provide reassurance as appropriate

- Do not be judgmental

- Offer links and hand outs

- Offer your support and emphasize your availability

Counseling subjects range from educating a diabetic patient about the disease and treatments

to motivating a patient to quit smoking to counseling about a Pap test results. It should be

interactive, and patient thoughts and expectations must be explored. Never lecture a patient!

Talk to your patient in a language he/she understands, and ask them what they got from what

you said to make sure you conveyed the message. Listen carefully to your patient’s responses

and observe body language at the same time. Avoid having your own agenda so rigid in your

Page 122: CANADIAN IMGS GUIDE TO OSCE AND PRACTICE · in an OSCE scenario You can use this study guide to prepare for the NAC OSCE and MCCQE2. The content was reviewed for the most recent updates;

122

mind that you forget to listen, clarify and respond to your patient. The quality of your

counseling is improved by the information you have gathered.

In OSCE the case’s stem could look like this; Counsel/Educate the patient about--------- or in XX

minutes take a focused history and counselIn such stations you must first take relevant and

brief history then counsel. You need to know your patient and give a personalized advice.

Sample cases are presented in chapter 5.

Examples of counseling subjects;

- Diabetes

- Birth Control

- Fertility Issues

- Menopause

- Genetics

- Abuse (sexual and physical)

- Hypertension

- Dyslipidemia

- Counsel a patient who had a heart attack

- Smoking Cessation

- Substance abuse

- Alcohol abuse

- Investigation results; like a pap test or incidental finding of a nodule on chest X-ray

- Medication side effects

- Obesity

- Cancer screening

- Risky sexual practices

- Immunization

Page 123: CANADIAN IMGS GUIDE TO OSCE AND PRACTICE · in an OSCE scenario You can use this study guide to prepare for the NAC OSCE and MCCQE2. The content was reviewed for the most recent updates;

123

- Allergies

- Health Screening

- Any subject the patient needs information about

References;

1- Mullen, Patricia Dolan, Denise G Simons-Morton, Gilbert Ramırez, Ralph F Frankowski,

Lawrence W Green, and Douglas A Mains. "A Meta-analysis of Trials Evaluating Patient

Education and Counseling for Three Groups of Preventive Health Behaviors." Patient

Education and Counseling 32, no. 3 (1997): 157-73. Accessed November 28, 2014.

http://www.sciencedirect.com/science/article/pii/S0738399197000372.

2- Hill, Edith, and Susan Fryters. Bugs & Drugs. Edmonton: Capital Health, 2006.

Page 124: CANADIAN IMGS GUIDE TO OSCE AND PRACTICE · in an OSCE scenario You can use this study guide to prepare for the NAC OSCE and MCCQE2. The content was reviewed for the most recent updates;

124

Page 125: CANADIAN IMGS GUIDE TO OSCE AND PRACTICE · in an OSCE scenario You can use this study guide to prepare for the NAC OSCE and MCCQE2. The content was reviewed for the most recent updates;

125

Chapter 5

Sample clinical cases Edited by; Dr. Subrata Datta

Dr. Sadik Salman

Dr. James Yeung

Dr. Abbeir Hussain

Dr. Erin Toor

Dr. Abdullah Saleh

Dr. Erica Paras

Page 126: CANADIAN IMGS GUIDE TO OSCE AND PRACTICE · in an OSCE scenario You can use this study guide to prepare for the NAC OSCE and MCCQE2. The content was reviewed for the most recent updates;

126

Introduction

This chapter is written to give you the opportunity to practice clinical cases as you prepare for

the exams. Most cases in this chapter are real patients whom I encountered during my training.

Some cases are made up, but all are based on common complaints and reviewed and edited by

Canadian physicians.

Each case is written with completeness in mind. However, I don’t guarantee that it covers all

the points on the Medical Counsel of Canada checklists. I strongly encourage you to review the

sample cases, sample checklists and common mistakes posted on the MCC website (refer to

Chapter 1 for link) before practicing the cases in this chapter.

I tried my best to simulate the exam, and presented the cases in a way that is going to make it

easy for you to practice alone or within groups. Each case starts with the door sign, then

suggested notes to write for yourself, most importantly what you are asked to do. The clinical

encounter is then presented in an interactive way to mimic reality. I will indicate which physical

exam exactly needs to be done, details of individual organ system P.E are however, provided in

chapter 3. Examples of possible questions asked by the examiner and their answers are

presented next. Then I will list basic communication and professional points that are part of

your overall evaluation. At the end included variation of the case where other differentials are

more likely when the same chief complaint is presented in a different context. Some tips and

comments are added to this last section.

Although I’m presenting the typical scenario for certain pathology, other differentials are still

possible and it is important to try to rule them in or out by focused history and physical

examination.

The cases cover both history and physical. Some cases history and counseling. Your practice

target should be to take a focused and relevant history in 4-5 min and conduct a focused and

relevant physical exam in 4 min. In this case you should be good in 10 min cases with a question

at 9 min.

Read questions carefully, and perform the task you are asked to do only. In most cases it is

straightforward as the task is history or physical or both. Nevertheless, other terms maybe used

and the scope of what you need to do may expand. Examples;

- Manage this patient; means take a relevant and focused history and perform a focused

and relevant physical exam, order investigations and treat as appropriate.

- Counsel this patient; means take a focused and relevant history and counsel.

- Explore the patient’s concerns; means take a brief history around the problem the

patient wants discussed and dig deep on what he/she knows already and what he/she

Page 127: CANADIAN IMGS GUIDE TO OSCE AND PRACTICE · in an OSCE scenario You can use this study guide to prepare for the NAC OSCE and MCCQE2. The content was reviewed for the most recent updates;

127

wants to know. Listen respectfully to the patient’s concern and answer his/her

questions. If you don’t know the answer simply say; I don’t know but I will research it

and get back to you. Never give false information to patients.

- Regardless of the type of question, always make sure the patient is stable and

comfortable

You may run into challenging situations. Those range from an angry patient who is not happy

with your care, to ethical dilemmas where patient’s confidentiality or other basic rights are at

stake. Stay calm and professional in these situations. Other challenges may test your

communication skills and sensitivity to your patient, like when a patient coughs; offer him/her

water. Or if the patient cries show empathy and offer some tissues.

Use a mix of open and closed ended questions and listen attentively to the patient. Respect the

patient’s ideas and beliefs even if they differ from yours. Answer the patient’s questions but

never give false information. Pay attention to the patient’s physical comfort during P.E.

It is a good idea to try to make a closure of the case. This is not possible in many cases as the

time is usually tight. If you finish early explain to the patient what you think is going on, what

investigations you will order and ask if he/she has any concerns. If the patient smokes briefly

mention the risks and offer a counseling appointment if the patient is interested. This applies

to other behaviors like substance abuse, or risky sexual practices. However, do not counsel in a

history and PE station – just mention it.

It is impossible to cover every single possible complaint. The best way to help you pass the

exams is to practice the cases in this chapter, and then create more cases based on other

common complaints. Please note that there are no sample psychiatry cases in this chapter.

READY?

Let’s get started…….

Page 128: CANADIAN IMGS GUIDE TO OSCE AND PRACTICE · in an OSCE scenario You can use this study guide to prepare for the NAC OSCE and MCCQE2. The content was reviewed for the most recent updates;

128

Case 1; Cough

Edited by; Dr. Subrata Datta

Door sign

Mrs. Stephanie Edwards is a 58 years old female who comes to your walk in clinic complaining

of cough.

Her vital signs are; BP 130/75, PR 87, RR 20, Temperature 37.3

In the next 10 min conduct a focused and relevant history and conduct a focused and relevant

physical exam. As you do the P.E explain to the examiner what you are doing and your findings.

At 9 min the examiner may ask you a question or questions.

Note; The NAC exam may have a similar stem but the last 2 line will look like this; in the next 11 minutes

conduct a focused and relevant history and conduct a focused and relevant physical exam. As you do the

P.E explain to the examiner what you are doing and your findings. At 8 min the examiner may ask you a

question or questions

Suggested notes to write for your self

Stepphanie Edwards 52 F RR 20 other V.S stable O2 sat?

C/C coughs URTI, Bronchitis, Pneumonia, HF, AECOPD

Hx and P.E Smoking?

Patient encounter

Questions Suggested verbalizing Patient response

History of presenting problem

Opening start How can I help you today? My cough is getting worse and really bothersome

Onset Tell me more about it. When did it start?

Well, I always cough because I’ve got this COPD but it is been bad the last week

Frequency How often do you cough? A lot.. Patient coughs (offer a glass of water)

Sputum Do you bring up any phlegm? The usual Amount Did the amount increase Yes indeed

Page 129: CANADIAN IMGS GUIDE TO OSCE AND PRACTICE · in an OSCE scenario You can use this study guide to prepare for the NAC OSCE and MCCQE2. The content was reviewed for the most recent updates;

129

Color What color was it and what is it now?

It was clear but now it’s yellowish

Hemoptysis Did you cough up any blood? No never

Dyspnea Do you feel short of breath? Only when the cough is too bad

Chest pain Do you have chest pain? Ahh, kind of

Tell me more about it It’s right here (patient points to the left anterior axillary line at the level of sixth rib with the tip of his finger), and not too bad

Radiation Does it go anywhere? No

Onset When did it start? Few days ago

Progression Is it getting worse over time? No

Frequency Is it there all the time? No, it comes and goes Aggravating factors What makes it worse? Cough and moving

Relieving factors What makes it better? If I’m not coughing I guess

Relation to breathing Does it get worse if you take a deep breath?

I haven’t noticed, I don’t think so

Quality How would you describe it? It’s like needles

Wheezing Do you have noisy breathing or hear wheezing?

Sometimes, but not very often

URTI Do you have runny nose or feel congested?

No

Constitutional Have you had fever? Have you lost weight recently? Do you get drenching night sweats that you need to change the sheets?

I don’t think so No, I gained 2 lb No

HF and PE Does it become difficult to breathe when you lie flat? Do you have legs swelling? Have you been bed bound recently?

No No No

Idea component of FIFI; Function

Is the cough affecting your function?

Yes, I missed 2 days of work

Sick contacts Have you been in contact with sick people?

Not that I know of

Past medical history

COPD History When were you diagnosed with COPD

5 years ago

When was your last breathing test?

I think I had one a year ago

Do you remember the result? No

Who follows your COPD My family doctor

Have you ever been hospitalized?

Yes, I had a bad chest infection a year ago and was in the hospital for a week

Page 130: CANADIAN IMGS GUIDE TO OSCE AND PRACTICE · in an OSCE scenario You can use this study guide to prepare for the NAC OSCE and MCCQE2. The content was reviewed for the most recent updates;

130

What treatment you received in hospital?

Antibiotics and they had me on the CPAP machine

Did you need intubation or ICU? No, none of that

Have you had any other exacerbation?

My cough gets worse 2-3 times each year but gets better with antibiotics and steroids

Did you take your flu shot? Yup, I take it every year

Did you take your pneumonia shot?

Yes, they gave it to me in the hospital the last year

Other medical problems And surgeries

Do you have any other medical problems particularly a clot in your leg or lungs, heart disease or cancer? Have you had any surgeries?

No No

Medications and allergies

Type What medications are you on Symbicort inhaler Dose and frequency How much and how often do

you use it? Always compliant? I take 2 inhalations in the morning and 2 in the evening

Side effects Do you have mouth thrush or any other side effects? Have you ever been tested for osteoporosis? Have you had any fractures?

No No No

Allergies Do you have any food or medications allergies?

Not that I know of

Family history

Does anyone in your family have any of; heart disease, diabetes, high blood pressure? Or Cancer?

My dad died of a heart attack when he was 80

Social history

Smoking Do you smoke Not any more

When did you quit? 4 years ago How much did you smoke? A pack a day

For how long? For 30 years

Alcohol Do you drink alcohol? Very occasional

Illicit drug use Do you use recreational drugs? I used to smoke marijuana occasionally but stopped 5 years ago

Occupation What do you do for living? I’m a secretary Exposures Have you ever been exposed to

chemicals or asbestos? Have you been in contact with Tuberculosis patients? Have you traveled recently?

No No Not for the last 10 years

Pets? Do you have pets? No

Page 131: CANADIAN IMGS GUIDE TO OSCE AND PRACTICE · in an OSCE scenario You can use this study guide to prepare for the NAC OSCE and MCCQE2. The content was reviewed for the most recent updates;

131

Sexual activity Are you sexually active? Not after my husband died a year ago

STOP!! Respond.. do not just continue asking your questions.

Sorry to hear that

Social support Do you live by yourself, any other family support?

I have one daughter, she lives 2 blocks away, I see her every day

Idea component of FIFI; Expectations

Thank you very much for all the information, Do you have any questions for me?

Not really, thank you doctor

Physical examination;

- Take permission, wash your hands and use respectful draping

- Say that you noticed that the patient is tachypnic with a RR of 20 and that other vital

signs are normal. Indicate that you would like to get the pulse oxymetry checked. The

examiner says; 95% on room air,

- Do general inspection, looking for signs of respiratory distress, the body habitus, hands,

face, and skin. Pay particular attention for cyanosis and clubbing

- Inspect the chest, palpate, percuss and auscultate the lungs

- Auscultate the heart

- Examine JVP, if you ran out of time indicate that you would

Sample questions you may be asked by the examiner;

1- What is the most likely diagnosis?

Answer; Acute exacerbation of COPD (AECOPD)

2- What investigations would you order?

Answer; CBCD, Chest XR, Creatinine, lytes, sputum gram stain and culture, ECG, ABG,

Spirometry when the patient returns to base line

3- How would you manage this patient?

Answer;

- Short coarse corticosteroids; prednisone 40mg P.O daily for 5 days

- Antibiotics; Amoxicillin 1g P.O TID for 7 days

- Bronchodilators; Continue Symbicort, add Atrovent 500ug INH MDI q 6 hours for 7 days

Rating scales points;

- Examinee introduced self and position

Page 132: CANADIAN IMGS GUIDE TO OSCE AND PRACTICE · in an OSCE scenario You can use this study guide to prepare for the NAC OSCE and MCCQE2. The content was reviewed for the most recent updates;

132

- Addressed patient with name

- Used proper non-verbal communication

- Good organizational skills

- Examinee spoke clearly (accent didn’t get in the way)

- Listening and questioning skills

- Showed rapport with patient

- Attentive to patient physical comfort

- Medical knowledge adequate

- No misinformation was provided to patients

- No concerning ethical/legal issues

- The examinee is respectful of other health care members

Variations of the case and some tips;

1- Case; 23 years male complaining of cough

History; had flu like symptoms then persistent cough for 2 weeks. Past medical history is

significant for treated Syphilis a year ago. When you ask about sexual history you find

that he had many partners over the last year and doesn’t use condoms.

Physical exam; Temperature 37.5, bilateral crackles and wheezing. No respiratory

distress.

At 9 min the examiner tells you that the patient chest XR showed bilateral diffuse

reticular infiltrates (Note that you may be asked to read a chest XR yourself) and asks

you about the most probable diagnosis

Discussion; this presentation is typical for PJP Pneumonia (an HIV defining illness). The

history of unprotected sex and CXR findings make the diagnosis more likely. However,

some patient’s may have a normal chest XR, so it is always a good idea to check for HIV

in patients with a history of unprotected sex and lower respiratory symptoms. Keep in

mind that other differentials like pneumonia, viral bronchitis are possible, and show the

examiner you are thinking about them

2- Case; 65 male, history of 60 pack-years smoking presenting with 2 weeks history of

productive cough

History; the patient had 2 episodes of hemoptysis coughing up sputum mixed with

blood. He lost 20 lb unintentionally over the last 3 months and is complaining of

Page 133: CANADIAN IMGS GUIDE TO OSCE AND PRACTICE · in an OSCE scenario You can use this study guide to prepare for the NAC OSCE and MCCQE2. The content was reviewed for the most recent updates;

133

persistent fatigue. He may have been exposed to Tuberculosis (TB) from a co-worker

years ago

Physical exam; normal vital signs. Respiratory exam revealed dullness, increased tactile

fremitus, bronchial breath sounds and positive egophony over the right middle lobe

Discussion; this patient’s presentation has 2 differentials on the top of the list; lung

cancer and TB. The patient has right middle lobe consolidation by physical exam along

with hemoptysis and constitutional symptoms. Include chest CT scan and sputum for

acid fast Bacilli in your investigations. Note that the right middle lobe is not a typical

location for TB which tends to involve the apices. And that the patient didn’t report

fever and had normal temperature on P.E. Nevertheless, you still need to exclude this

deadly infection with a history of exposure.

3- Case; 22 year old female complaining of recurrent episodes of cough

History; For the last several months the patient has had recurrent bouts of cough and

shortness of breath especially after running for more than 5 blocks. She feels wheezy

sometimes, and coughed few times in the middle of the night. She had similar episodes

when visiting one of her friends. On further questioning you discover that the patient’s

friend has a cat, the patient has eczema and her mother is asthmatic.

Physical exam; normal

Discussion; the patient most likely has asthma, although GERD and atypical infections

are also possible. It is wise to ask about heart burn and sour taste of saliva. Initial

investigations include; CBCD, chest XR, Spirometry and a Methacholine challenge test. In

some cases you may encounter a patient with established diagnosis of asthma who had

recent deterioration of symptoms, make sure you assess severity by asking about

frequency of symptoms, the need for Ventoline, night time symptoms and days missed

from school/work. Also ask about triggers. It is necessary in such cases to watch the

patient using his/her own inhalers to make sure the technique is correct.

4- Familiarize yourself with the technique for MDI and PDI inhalers. In some cases you may

need to show the patient how to use them, or watch the patient use his/her own. The

following you tube videos are helpful;

MDI; http://www.youtube.com/watch?v=YWNcPReibZA

PDI; http://www.youtube.com/watch?v=-tyF-MC1qQo

Page 134: CANADIAN IMGS GUIDE TO OSCE AND PRACTICE · in an OSCE scenario You can use this study guide to prepare for the NAC OSCE and MCCQE2. The content was reviewed for the most recent updates;

134

5- The following website contains the Canadian guidelines for respiratory diseases;

http://www.respiratoryguidelines.ca/home

6- Always keep your differential wide. The above cases illustrate how a single complaint

can be a manifestation of a variety of diseases. You will be able to narrow down your

differential to two or more possibilities. It is important to show the examiner that you

are trying to do so. One of my patients presented with cough, lung cavities on chest XR

and a history of strong exposure to active TB from her mom. She was put on isolation

and admitted as a case of TB. To our surprise all her TB tests came back negative and

lung biopsy confirmed the diagnosis of Cryptogenic Organizing Pneumonia (COP)

Page 135: CANADIAN IMGS GUIDE TO OSCE AND PRACTICE · in an OSCE scenario You can use this study guide to prepare for the NAC OSCE and MCCQE2. The content was reviewed for the most recent updates;

135

Case 2; Fatigue

Edited by; Dr. Subrata Datta

Door sign

Anya Glenn, a 35 years old female presenting with fatigue.

Vital signs;

BP 110/70

PR 67

RR 12

Temperature 36.5

In the next 10min take a focused and relevant history and conduct a focused and relevant P.E.

As you do the P.E explain to the examiner what you are doing and your findings. At 9min the

examiner may ask you a question or questions.

Note; The NAC exam may have a similar stem but the last 2 line will look like this; in the next 11 minutes

conduct a focused and relevant history and conduct a focused and relevant physical exam. As you do the

P.E explain to the examiner what you are doing and your findings. At 8 min the examiner may ask you a

question or questions

Suggested notes to write for your self

Anya Glenn 35 F DDx; Depression

C/C; Fatigue Hypothyroid

Cancer

Chronic infection

CTD

Hx. GI

P.E

Sleep, diet, exercise, stress

Patient encounter

Questions Suggested verbalizing Patient response

History of presenting complaint

Page 136: CANADIAN IMGS GUIDE TO OSCE AND PRACTICE · in an OSCE scenario You can use this study guide to prepare for the NAC OSCE and MCCQE2. The content was reviewed for the most recent updates;

136

Opening question How can I help you today? I’m feeling very tired, I just have no energy

Onset and duration When did you start to feel tired?

The last 6 months

Associated symptoms Have you had other symptoms?

I’m also constipated

Details about constipation How often do you poop? Is your stool hard or soft? When did this start? How often did you poop before? Do you have diarrhea as well? What color is your stool? Did you notice any blood?

Once a week if I was lucky Sometimes it’s hard Few months ago Every day! I was very regular No Normal brown No

GI system Do you have abdominal pain? How about nausea/vomiting? Did you notice that your eyes are turning yellow?

No No No

Hypothyroidism symptoms? Do you feel more cold than usual? Have you noticed any hair or skin changes? Do you have difficulty getting up from a chair? Have you noticed swelling of your neck? Did people tell you your voice changed? Has there been any change to your periods?

Very much, my 13 years old makes fun of me layering up when its sunny outside My hair is thinner and falling and my skin is dry No No No Yes, they are not regular any more, the last 4 months I had only one period! And I can assure you I’m not pregnant!

Constitutional symptoms? Do you have fever? Have you lost weight recently? Do you have drenching night sweats that you have to change the sheets?

No No, I think I gained 4 lb No

Anemia Note that questions about menses/bleeding are covered

Did people say you look pale? Do you feel your heart is racing?

No No

Page 137: CANADIAN IMGS GUIDE TO OSCE AND PRACTICE · in an OSCE scenario You can use this study guide to prepare for the NAC OSCE and MCCQE2. The content was reviewed for the most recent updates;

137

Heart failure Do you feel out of breath? Do you have swelling around your ankles?

No No

Depression screening Do you feel depressed? I understand this is hard, but do you feel very sad? Have you lost interest in activities you enjoyed before? Did your sleep change? Do you wake up very early recently? Did your appetite change? Did you notice a change to your concentration and memory?

Well, it is depressing when you don’t seem to get anything done Not really, it’s more of frustration No No No No I’m slower than before

CTD Note that some questions are already covered

Do you have swelling or pain in your joints? Do you feel stiff in the morning that you need some time to get going?

No No

Chronic infection Note that a lot of the questions are already covered

Do you have cough? Do you feel burning when you pee? Have you travelled recently? Do you have skin rash?

No No No No

Past medical history Have you been diagnosed

with a medical disease like diabetes?

No. I’ve been healthy so far

Medications

Do you take any medications?

No

Allergies

Do you have allergies to medications or food?

No

Family history

Does anyone in your family have a medical condition?

My mother has high blood pressure

Page 138: CANADIAN IMGS GUIDE TO OSCE AND PRACTICE · in an OSCE scenario You can use this study guide to prepare for the NAC OSCE and MCCQE2. The content was reviewed for the most recent updates;

138

Does anyone have a thyroid disease?

No

Cancer? Or heart disease? No

Social History Smoking Do you smoke? No

Alcohol Do you drink alcohol? No

Recreational drugs Do you use recreational drugs?

No

occupation What do you do for living? I’m a sales representative Living conditions With whom do you live? With my husband and 13

years old son

Stress Any stress at work or home? Not really no Idea component of FIFE; FUNCTION: Effect of fatigue on work/home

How is fatigue affecting your life?

It makes me frustrated, I push myself to get things done but haven’t got in trouble yet

Diet Do you eat healthy meals with good portion of protein, carbs, fruits and veggies?

I do, I eat very healthy and only buy organic food

Exercise Do you exercise? I wish!

FIFE; Expectation Thank you for sharing all this information. Do you have any questions for me? I understand your frustration, I need to examine you first and run some tests and hopefully we will get an answer

Well, what is wrong with me? I never felt like this before

Physical Examination;

- Mention that the vital signs are normal

- Do a general exam starting with the hands, feel the radial pulse, inspect the face; Look

for jaundice or pallor; look at the buccal mucosa for hemorrhage or infection as well as

hygiene status. Inspect the skin. Examine the JVP

- You need to cover multiple organ systems; inspect the chest and auscultate the heart

and lungs (no need for palpation and percussion). Inspect and palpate the abdomen

paying particular attention for masses or splenomegaly. Feel the lymph nodes. Inspect

the lower limbs and check for edema

Page 139: CANADIAN IMGS GUIDE TO OSCE AND PRACTICE · in an OSCE scenario You can use this study guide to prepare for the NAC OSCE and MCCQE2. The content was reviewed for the most recent updates;

139

- Examine the thyroid; inspect the neck with and without the patient swallowing and

palpate the thyroid for temperature, tenderness, masses and enlargement. Examine the

cervical lymph nodes. Check for manifestations of thyroid disease; Inspect the eyes from

the side for exophthalmos, examine extraocular movements and examine for lid lag.

Test the proximal muscles of the upper and lower limbs for weakness. Check the pulse.

Test DTR. Inspect the lower limbs for pretibial myxedema

Sample questions you may be asked by the examiner;

- What is the most likely diagnosis?

Answer; Hypothyroidism

- Name a single investigation that you would do to confirm this diagnosis?

Answer; TSH (Thyroid stimulating hormone)

Rating scales points;

- Examinee introduced self and position

- Addressed patient with name

- Used proper non-verbal communication

- Organizational skills

- Examinee spoke clearly (accent didn’t get in the way)

- Listening and questioning skills

- Showed rapport with patient

- Attentive to patient physical comfort

- Medical knowledge adequate

- No misinformation was provided to patients

- No concerning ethical/legal issues

- The examinee is respectful of other health care members

Variations of the case and some tips;

1- Case; 60 year old female complaining of fatigue

History; positive for drenching night sweats and newly diagnosed breast cancer in

her sister at age 55. Screening for depression was also positive

P.E; palpable 1*1.5 right axillary lymph node. Otherwise normal including breast

examination

Page 140: CANADIAN IMGS GUIDE TO OSCE AND PRACTICE · in an OSCE scenario You can use this study guide to prepare for the NAC OSCE and MCCQE2. The content was reviewed for the most recent updates;

140

Discussion; this patient presentation strongly suggests Breast cancer, or other

malignancy. At the same time she is depressed, which could be primary or secondary

to malignancy or grief. You need to do thorough investigations including CBCD, urea,

Creatinine, lytes, LDH, INR, PTT, ALT, AST, ALK, Albumin, total protein, bilirubin, CK,

CXR, ECG. I would go straight to chest CT with axillary view in this case (no need for

mammogram or breast US, as you would end up doing CT if they were positive for

further details and if negative because malignancy is highly likely). I would still get a

chest XR because it’s easier and faster and may need it in the future if complications

arise (good to know her baseline). The psychological component is very important;

address and treat depression, and dig deep into social support. Early diagnoses and

referral could save this patient’s life. Remember to keep other differentials in mind.

2- Case; 25 year old female presenting with fatigue

History; Heavy menstrual cycles, diet is mainly fast and frozen food. Has occasional

shortness of breath and palpitations, especially on exertion. No chest pain. Wants to

get pregnant but thinks she wouldn’t be able to handle it. Appears very anxious and

worried that something serious is going on. When asked why she feels it’s serious

she shares with you that her sister in law died with cervical cancer a year ago

P.E; Pale conjunctiva and buccal mucosa. Tachycardia at 110 with regular pulse. JVP

and cardiac exams are normal. No other findings

Discussion; this patient most likely has iron deficiency anemia from her heavy

periods. B12 and folate deficiency are also possible and could be concomitant given

her diet history. The shortness of breath and tachycardia could be secondary to the

anemia or to concurrent cardiac condition that could be unrelated to or caused by

anemia. You need to know why she has menorrhagia, and obtain detailed menstrual

history, and endocrine system review.

Get a CBCD, iron, total iron binding capacity and ferritin, B12, folate, TSH, Creatinine,

urea, and lytes, and a baseline ECG and chest XR. With negative family history of

heart disease and absence of risk factors you can treat the anemia and reassess the

patient’s palpitations and shortness of breath; if persistent, an echocardiogram will

help to sort it out.

3- Case; 42 year old female complaining of fatigue and pain every where

Page 141: CANADIAN IMGS GUIDE TO OSCE AND PRACTICE · in an OSCE scenario You can use this study guide to prepare for the NAC OSCE and MCCQE2. The content was reviewed for the most recent updates;

141

History; Feeling tired most of the time for the last 5 months with diffuse poorly

localized pain that seems to migrate. Sleep is interrupted. The patient is very anxious

about it, and wants to know what’s wrong. She made some mistakes at work

because she can’t focus and is afraid she may lose her job. She saw three other

physicians before coming to see you and had a lot of investigations done, she is

frustrated because they said it’s all in her head. Comprehensive review of systems

was negative apart from the above mentioned symptoms. Screening for depression

was negative.

P.E; normal vitals, general, cardiac, respiratory, abdominal and joints P.E. You test 3

fibromyalgia trigger points; occiput at the nuchal ridge, trapezius, cervical and find

them tender

Discussion; Fibromyalgia is a debilitating condition that is poorly understood.

Exclude possible underlying diseases before arriving at the diagnosis. It is a good

idea to get the previous investigations and make sure age appropriate cancer

screening is done. Keep in mind hypothyroidism and obstructive sleep apnea.

Depression and anxiety are commonly seen in fibromyalgia patients, screen and

treat as appropriate. If everything else is negative and you think fibromyalgia is most

likely refer to a rheumatologist where multi-disciplinary individualized treatment

plan can be initiated.

Page 142: CANADIAN IMGS GUIDE TO OSCE AND PRACTICE · in an OSCE scenario You can use this study guide to prepare for the NAC OSCE and MCCQE2. The content was reviewed for the most recent updates;

142

Case 3; Chest pain in the ER

Edited by; Dr. Jay Shavadia

Door sign;

Mr. Arthurs, 65 year old male, brought by EMS with chest pain

BP 120/75

HR 91

RR 14

Temperature 37

There is a nurse in the room

In the next 9 min manage the patient. Ask the nurse to do any orders you deem

necessary. As you examine the patient explain to the examiner what you are doing

and your findings. At 10min the examiner will ask you a question or questions

Note; In this case you will be communicating with the nurse and the patient. Communication

with the nurse is presented in italic. You need to make sure the patient is stable, give emergency

medications, take a focused and relevant history, perform a focused and relevant P.E and order

investigations. The P.E is included in the case. Treat the nurse with respect, and give clear orders

specifying the dose and route of medications.

Even though it is an ER case, it is important that you introduce yourself to the nurse and patient.

Suggested notes to write for your self

Mr. Arthurs/65

ABC MI

P.E

Aortic dissection

Page 143: CANADIAN IMGS GUIDE TO OSCE AND PRACTICE · in an OSCE scenario You can use this study guide to prepare for the NAC OSCE and MCCQE2. The content was reviewed for the most recent updates;

143

General screening exam Pericarditis/Pleuritis

O₂, IV access, ASA, nitrate

Patient encounter

Your actions Suggested

verbalizing

Patient or

nurse

response

ABC

Can you open your

mouth Sir? The

airways are clear,

and the patient is

breathing, I’m

feeling the pulse and

it is regular

O₂ and

monitoring

Can you put the

patient on 4L oxygen

and attach him to

the cardiac monitor

Can I get another set

of vital signs

What is the O₂

saturation

O₂ given,

monitor

attached

Unchanged

100% on 4L

IV and

investigations

Can you establish an

IV access and draw

blood for CBCD,

Creatinine, urea,

lytes, Troponin, CK-

MB, INR, PTT

Can you start the

patient on normal

saline 0.9% at

75cc/hour

Done

IV in, fluids

given

Page 144: CANADIAN IMGS GUIDE TO OSCE AND PRACTICE · in an OSCE scenario You can use this study guide to prepare for the NAC OSCE and MCCQE2. The content was reviewed for the most recent updates;

144

Chest pain Do you have chest

pain right now?

No, it’s gone,

those guys

where magical

EMS history I understand EMS

brought the patient

What was done? They did an

ECG and gave

him 2 doses of

nitroglycerine

SL 0.4 mg

The nurse

hands you the

ECG

Read the ECG,

if you don’t

the examiner

will ask you to

do so

The ECG of Mr

Arthur was done at

10:50 am today.

Normal sinus rhythm

and axis. No ST

changes. P wave, PR,

QRS and QT, are

within normal

There is T wave

inversion in leads

V2, V3, V4

Order ECG and

CXR

Can I get another

ECG and stat CXR

Ordered, ECG

will be

available in 5

min

History

Sir tell me more

about this pain (ask

for character,

continuous /

intermittent)

It was right

here (patient

uses his hand

to point to the

center of his

chest) and felt

like a heavy

block lying on

my chest.

Page 145: CANADIAN IMGS GUIDE TO OSCE AND PRACTICE · in an OSCE scenario You can use this study guide to prepare for the NAC OSCE and MCCQE2. The content was reviewed for the most recent updates;

145

Continuous

over 20mins,

till the NTG

spray by EMS

Did it go anywhere?

(shoulders, jaw,

neck, elbow, arms,

back)

I felt it over

my shoulder

How severe was it

on a scale from one

to 10 with 10 being

the worst pain ever?

It was 9

What were you

doing when you had

it?

I was shoveling

the snow

Did anything make it

worse? Like

movement,

breathing, cough,

laying down

No

Have you had similar

pain before?

Not as bad, I

had the same

pain once

before but it

was 4, I was

also shoveling

but it went

away in less

than a minute

when I sat

down

Did you get any

treatment for it?

No, I didn’t see

a doctor for it

Did you feel dizzy

today?

I felt

lightheaded

when I had the

pain

Page 146: CANADIAN IMGS GUIDE TO OSCE AND PRACTICE · in an OSCE scenario You can use this study guide to prepare for the NAC OSCE and MCCQE2. The content was reviewed for the most recent updates;

146

Did you lose

consciousness?

No

Did you feel your

heart racing?

No

Did you feel short of

breath?

No

Did you have cough?

Did you cough up

blood?

No

No

Did you have nausea

or vomiting?

I felt

nauseated

Do you have

diabetes?

No

Do you have high

blood pressure?

Yes

Are you on

medication for it?

I take Coversyl

8mg every day

Do you have high

cholesterol?

How much?

Yes, I take

Lipitor for it

Not sure I

think 20mg

Do you smoke? Yes

How much and for

how long?

1 pack a day

since I was 20

Do you drink

alcohol?

I enjoy a glass

of wine with

dinner

Did anyone in your

family have a heart

disease?

My father died

of a heart

attack when

he was 50

The patient

asks you a

question

Did I just have a

heart attack doctor?

Well, It’s

possible; we

need to do

some tests to

know exactly

Page 147: CANADIAN IMGS GUIDE TO OSCE AND PRACTICE · in an OSCE scenario You can use this study guide to prepare for the NAC OSCE and MCCQE2. The content was reviewed for the most recent updates;

147

what’s going

on.

I need to ask few

more questions

Patient nods

his head yes

Do you have allergy

to medications?

Not that I

know of

Do you have allergy

to aspirin?

No

Do you have Asthma

or other lung

disease?

No

Are you still doing

ok? Have you had

any more chest

discomfort?

I’m fine I don’t

feel any pain

Have you had a

surgery or accidents

in the last 6 months?

How about bleeding

from your stomach

or in your head?

Have you been

diagnosed with

cancer?

Do you use any

‘street drugs’?

No

No

No

No

Can you please give

the patient Aspirin

325mg orally

Can you give Plavix

300mg orally now

Given

Given

Is any of the

investigations back

The ECG is

back

You look at the

ECG, and find

it’s

unchanged, if

the examiner

ECG unchanged

Page 148: CANADIAN IMGS GUIDE TO OSCE AND PRACTICE · in an OSCE scenario You can use this study guide to prepare for the NAC OSCE and MCCQE2. The content was reviewed for the most recent updates;

148

asks you to

read then

specify details

You share a

summary with

the nurse

The patient came

with ischemic chest

pain with ST

changes, resolved

with 2 doses of

Nitro, given ASA,

Plavix, Oxygen and is

now pain free,

Attached to the

monitor, vitals are

stable. He is on IV

N.S. The second ECG

is normal, other

investigations are

pending

Yes

Physical

examination

Sir do you mind if I

examine you?

Remember to

wash your

hands before

P.E

General I’m looking at the

hands for nicotine

stains, cyanosis, and

palmar erythema all

negative. I’m feeling

the pulse noticing it

is regular, no

collapsing pulse. I’m

comparing the radial

pulse of both arms

and notice it is

symmetrical. I’m

inspecting the face

looking for central

cyanosis and pallor.

Page 149: CANADIAN IMGS GUIDE TO OSCE AND PRACTICE · in an OSCE scenario You can use this study guide to prepare for the NAC OSCE and MCCQE2. The content was reviewed for the most recent updates;

149

JVP I’m going to lift the

bed up a little bit.

I’m looking at the

JVP between the 2

heads of the

sternocleidomastoid.

Measuring the

height from the

sternal angle it is 3

cm. Can you take a

deep breath? It is

going down with

respiration, is double

impulse, not

palpable and

disappears with a

pressure to the root

of the neck

Chest

examination

I’m going to feel for

your heart and listen

to your heart and

lungs. I’m inspecting

the chest, no visible

pulsations, scars,

deformities, dilated

veins or skin

changes. I’m feeling

the apical beat and

notice it is located in

the 5th intercostal

space midclavicular

line as expected. No

heaves, no thrills.

I’m auscultating the

heart, normal S1, S2,

no S3, S4 and no

murmurs. Can you

take a deep breath

Page 150: CANADIAN IMGS GUIDE TO OSCE AND PRACTICE · in an OSCE scenario You can use this study guide to prepare for the NAC OSCE and MCCQE2. The content was reviewed for the most recent updates;

150

in and out?

Symmetrical

vesicular breath

sounds bilaterally,

no crackles or

wheezing.

Abdomen Cover the chest, and

lift the gown up.

Lightly then deeply

palpate the

abdomen

Lower limbs Look for edema,

asymmetry, redness,

swelling calf

tenderness

Consultation

I’m going to consult

cardiology

Thanks

Sir, will get the heart

specialists involved

and take good care

of you , do you have

any questions?

You’re most

welcome

Will page

them

Not now,

thank you

Page 151: CANADIAN IMGS GUIDE TO OSCE AND PRACTICE · in an OSCE scenario You can use this study guide to prepare for the NAC OSCE and MCCQE2. The content was reviewed for the most recent updates;

151

Sample questions you may be asked by the examiner;

- What is your differential?

Answer: Acute coronary syndrome, Aortic dissection

Rating scales points;

- Examinee introduced self and position

- Addressed patient with name

- Used proper non-verbal communication

- Organizational skills

- Recognized and managed emergency effectively prioritizing actions

- Examinee spoke clearly (accent didn’t get in the way)

- Listening and questioning skills

- Showed rapport with patient

- Attentive to patient physical comfort

- Medical knowledge adequate

- No misinformation was provided to patients

- No concerning ethical/legal issues

- The examinee is respectful of other health care members

Variations of the case;

1- Case; 42 male, hypertensive, not on treatment presenting with chest pain

History; Sudden onset tearing chest pain radiating to the back. The patient is a

smoker and known to have hypertension but not on treatment

P.E; BP right arm; 190/100, left arm 168/95. Pulse weaker on the left. Diastolic

murmur is heard over the aortic area

Discussion; Aortic dissection can be easily missed, and a lot of patients don’t have

typical presentation. Always keep it in the back of your mind when assessing

patients with chest pain especially if they were hypertensive on presentation. CT is

the gold standard for diagnosis

2- Case; 35 year old female, presenting with sudden onset chest pain

Page 152: CANADIAN IMGS GUIDE TO OSCE AND PRACTICE · in an OSCE scenario You can use this study guide to prepare for the NAC OSCE and MCCQE2. The content was reviewed for the most recent updates;

152

History; The patient suddenly felt chest pain while watching TV, she was as well out

of breath and coughing. No hemoptysis. She is a smoker and has a previous history

of 3 spontaneous abortions and one still birth. Not previously diagnosed with a

medical condition

P.E; The patient was tachycardic at 120, tachypnic at 24. BP was 140/85. Normal

temperature. O₂ saturation was 88% on room air. General and chest examination

were normal. Lower limb examination revealed that the right leg was 3cm bigger

than the left.

Discussion; A history of recurrent spontaneous abortions and still birth is highly

suspicious for coagulopathy. Along with discrepancy of the size between the 2 legs

and the nature of pain makes a pulmonary embolism high on the differential.

3- 24 male, presenting with chest pain

History; Stabbing left sided chest pain worse with respiration and cough. He had a

cold 10 days ago. Otherwise healthy non- smoker, non- drinker and doesn’t use

recreational drugs

P.E: Stable vital signs. A triphasic leathery rub is heard over the apex. Otherwise

unremarkable

Discussion; Pericarditis is high on the differential. Try to rule out secondary causes

like a viral infection or connective tissue disease.

4- Although atherosclerosis is the most common cause of ischemic chest pain, keep

in mind other causes like Prinzmetal angina or cocaine induced coronary artery

spasm especially in younger populations.

Page 153: CANADIAN IMGS GUIDE TO OSCE AND PRACTICE · in an OSCE scenario You can use this study guide to prepare for the NAC OSCE and MCCQE2. The content was reviewed for the most recent updates;

153

Case 4; Chest pain in an out patient

Edited by; Dr. Jay Shavadia

Door sign;

Mr Adam Bailey, 52 years old male, comes to your clinic because he had 2 episodes

of chest pain.

Vital signs are;

BP 145/90

PR 88

RR 14

Temperature 37⁰C

O₂ saturation 99% on room air

In the next 9min take a focused and relevant history and conduct a focused and

relevant medical exam. As you do the P.E explain to the examiner what you do and

your findings.

At 9min the examiner may ask you a question or questions

Sample notes to write for yourself;

Mr Bailey 52M Details about the pain

Cardiovascular risk factors

GI system

Chest pain Respiratory

Hx and P.E

Patient encounter;

Your actions Suggested

verbalizing

Patient response

Page 154: CANADIAN IMGS GUIDE TO OSCE AND PRACTICE · in an OSCE scenario You can use this study guide to prepare for the NAC OSCE and MCCQE2. The content was reviewed for the most recent updates;

154

History of present

illness

How can I help

you today?

I’m worried

doctor, I’ve had

chest pain

How many times

have you had

chest pain?

2 times

Tell me about the

first time

I was walking my

dog as usual, it

was a bit cold that

day, then I had

pain in my chest

right here (patient

uses his hand to

point to the

center of his

chest). I stopped

and took a couple

of deep breaths

then it went away

How long did it

last?

Maybe one or two

minutes

Did it go

anywhere?

No

How did it feel? It felt like a rock

on my chest, it

was horrible

How severe was it

on a scale from 1-

10, 10 being the

most severe pain

ever?

It was 8

Did you feel dizzy? No

Did you sweat

when you had the

pain?

A little bit

Did you feel

nauseated or had

vomiting?

I felt a bit

nauseated, but

didn’t throw up

Page 155: CANADIAN IMGS GUIDE TO OSCE AND PRACTICE · in an OSCE scenario You can use this study guide to prepare for the NAC OSCE and MCCQE2. The content was reviewed for the most recent updates;

155

Did you feel your

heart racing?

No

Did you feel out of

breath?

A little

How far did you

walk before you

had the pain?

4 blocks

How about the

second time you

had pain?

The same day

when I was just

about to enter the

house, it felt

exactly the same

Did you have a

similar pain

before?

No, that’s the first

time, it happened

the last week

Did you get

medical help?

No

So it came up by

walking 4 blocks in

the cold air, and

went away by rest,

is that right?

yes

Did it get worse

with breathing?

No, it got better

Do you cough? No

Did you have

fever?

No

Did you have a

cold recently?

No

Do you feel your

breathing is

wheezy?

No

Do you cough up

blood?

No

Do you get heart

burn?

No

Page 156: CANADIAN IMGS GUIDE TO OSCE AND PRACTICE · in an OSCE scenario You can use this study guide to prepare for the NAC OSCE and MCCQE2. The content was reviewed for the most recent updates;

156

Do you feel a sour

taste in your

mouth?

No

Do you have

abdominal pain?

No

Do you have

diarrhea?

Constipation?

No

No

Have you ever lost

consciousness?

No

Past medical

history?

Do you have any

medical

conditions?

I have a high

blood pressure

that I try to

control with diet

Do you have

diabetes?

High Cholesterol?

Have you ever

been diagnosed

with a heart

disease?

No

Not that I know of

No

Have you ever

been hospitalized

or had surgery?

No

Family history?

Did anyone in your

family have a

heart attack?

My mom died of a

heart attack when

she was 65

How about your

dad?

He had prostate

cancer

Any other diseases

in the family?

No

Did anyone die

suddenly in your

family?

No

Page 157: CANADIAN IMGS GUIDE TO OSCE AND PRACTICE · in an OSCE scenario You can use this study guide to prepare for the NAC OSCE and MCCQE2. The content was reviewed for the most recent updates;

157

Medications and

allergies?

Do you take any

medications?

No

Do you take any

over the counter

medications or

herbs?

No

Do you have

allergy to

medications or

food?

No

Social history

What do you do

for living?

I’m a biology

teacher

With whom do

you live?

Is she healthy?

With my wife

yes

Do you have any

children?

No

Do you smoke? I tried it once

when I was young

and didn’t like it

Do you drink

alcohol?

Very occasionally

Do you use

recreational

drugs?

No

Do you exercise? I take the dog for

a walk every day

but haven’t done

so for the last

week

Tell me about your

diet

Well, my wife is

healthy and

always tries to get

me to eat like her,

Page 158: CANADIAN IMGS GUIDE TO OSCE AND PRACTICE · in an OSCE scenario You can use this study guide to prepare for the NAC OSCE and MCCQE2. The content was reviewed for the most recent updates;

158

I cheat and eat a

lot of junk food

Closure Thank you for sharing all this information with me. This pain is likely angina. We need to do some tests. Do you have any questions?

Not at the moment, thank you

Physical Exam;

- Mention that the vital signs are normal

- Do a general exam of the hands and face; Look for nicotine stains, palmar erythema,

cyanosis, Osler nodes, Janeway lesions, feel the elbows for tendon Xanthomas. Inspect

the face for pallor, cyanosis, corneal archus and Xanthelasma

- Examine the JVP, Listen for carotid bruits and feel the carotid pulse

- Examine the precordium

- Auscultate the lungs paying particular attention to wheezing and basal crackles

- Feel the radial and brachial pulses, comparing both sides and checking for Brachioradial

delay

- Inspect the lower limbs, check for edema, and feel the dorsalis pedis, posterior tibial

and popliteal pulses

- Inspect the abdomen for flank fullness, asymmetry or masses, Feel the abdominal aorta

and listen for bruits. Palpate for enlarged liver and check for ascites

Sample questions you may be asked by the examiner;

- What is your differential?

Answer; Acute coronary syndrome, prinzmetal angina, esophageal spasm

- What investigations would you order?

Answer; ECG, chest XR, CBCD, urea, Creatinine, lytes, INR, PTT, troponin. If ECG has

no ischemic changes, and troponin negative, then a stress test

Rating scales points;

- Examinee introduced self and position

- Addressed patient with name

- Used proper non-verbal communication

Page 159: CANADIAN IMGS GUIDE TO OSCE AND PRACTICE · in an OSCE scenario You can use this study guide to prepare for the NAC OSCE and MCCQE2. The content was reviewed for the most recent updates;

159

- Organizational skills

- Examinee spoke clearly (accent didn’t get in the way)

- Listening and questioning skills

- Showed rapport with patient

- Attentive to patient physical comfort

- Medical knowledge adequate

- No misinformation was provided to patients

- No concerning ethical/legal issues

- The examinee is respectful of other health care members

Variations of the case;

1- Case; 30 year old male, complaining of chest discomfort

History; Retrosternal burning sensation, typically felt when overeating, or upon

consuming fatty or spicy food. On occasion, the burning is also felt upon lying down.

Symptoms started a year ago, but worsened the last 3 months. The patient also has

dry cough for the past 3 months. Screening for cardiovascular risk factors was

negative.

P.E; Normal

Discussion; Even in low risk patients, always rule out cardiac causes of chest pain.

This patient’s history is typical for GERD. However, he could have esophagitis or

other concomitant complications. The cough could be a manifestation of GERD or

could be totally unrelated; the best test is improvement with GERD treatment.

2- Case; 23 F complaining of chest pain

History; One week of sharp left sided chest pain localized anteriorly from the second

through the 4th intercostal spaces. Worsened by deep breathing and moving. The

patient had the flue one month ago. Review of the cardiovascular, respiratory and

musculoskeletal system was negative. She is healthy, doesn’t use drugs and plays a

variety of sports on daily bases.

P.E; Left sided chest wall tenderness from the 2ndd to 4th spaces. Otherwise

unremarkable, V.S stable

Page 160: CANADIAN IMGS GUIDE TO OSCE AND PRACTICE · in an OSCE scenario You can use this study guide to prepare for the NAC OSCE and MCCQE2. The content was reviewed for the most recent updates;

160

Discussion; Chest wall tenderness makes Costochondritis highly likely, Other causes

of chest pain must, however, be excluded. Trauma and a viral infection are

implicated as possible causes for Costochondritis, no specific tests required. Treat

the patient with rest and anti-inflammatory medications.

Page 161: CANADIAN IMGS GUIDE TO OSCE AND PRACTICE · in an OSCE scenario You can use this study guide to prepare for the NAC OSCE and MCCQE2. The content was reviewed for the most recent updates;

161

Case 5; Back pain

Edited by; Dr. James Yeung

Door sign

Mrs. Elizabeth Peter’s is a 45 years old female, presenting with low back pain.

Vital signs;

BP 120/80

PR 82

RR 12

Temperature 36.5

In the next 9min take a focused and relevant history and conduct a focused and

relevant medical exam. As you do the P.E explain to the examiner what you do and

your findings.

At 9min the examiner may ask you a question or questions

Note; the NAC may have a similar stem but the last 3 line will look like this; In the

next 11 min take a focused and relevant history and conduct a focused and relevant

medical exam. As you do the P.E explain to the examiner what you do and your

findings. At 8min the examiner may ask you a question or questions

Sample notes to write for yourself

Elizabeth Peters Herniated disk?

45 F Infection

Malignancy

Back pain AS

Hx, PE Spinal stenosis

Degenerative disc disease

Fracture

Mechanical back pain

Cauda equina

Patient encounter

Page 162: CANADIAN IMGS GUIDE TO OSCE AND PRACTICE · in an OSCE scenario You can use this study guide to prepare for the NAC OSCE and MCCQE2. The content was reviewed for the most recent updates;

162

Your actions Suggested

verbalizing

Patient response

Opening start How can I help

you today?

I have back pain

doctor

Tell me more

about it

It is right here

(patient points to

the center of

lower back)

Radiation Does the pain

radiate

anywhere?

I feel it here

(patient points to

the right lateral

leg and foot)

Onset When did it

start?

Tell me about

the first time

you had it, did

you have

trauma, lift

something

heavy?

Was it sudden

or insidious?

Almost a year now

I don’t think so ,

can’t remember

doing anything

specific

I would say

insidious

Intensity and

progression

How severe is

it on a scale

from 1-10, 10

being the

worst pain

ever?

Do you get it

every day?

Is it getting

worse over the

last year?

It’s about five to

seven

I get it once or

twice a week,

especially when I

clean the house

Not really

I take Tylenol or

Advil and rest, it

helps

Page 163: CANADIAN IMGS GUIDE TO OSCE AND PRACTICE · in an OSCE scenario You can use this study guide to prepare for the NAC OSCE and MCCQE2. The content was reviewed for the most recent updates;

163

What do you

do when it’s

very bad?

Aggravating and

relieving factors

What makes it

worse?

What makes it

better?

So it worsens

with activity

and improves

with rest?

Moving, especially

bending forward

Rest

That’s right

quality How would

you describe

it?

It’s aching

Constitutional

Fever

Weight loss

Night sweats

Have you had

fever?

Did you lose

weight over

the last year?

Do you get

drenching

night sweats

that you have

to change the

sheets?

No

No

No

Associated symptoms

CTD

Do you have

joints pain or

swelling,

particularly the

shoulders or

hips?

Do you have

skin rash?

No

No

No

Page 164: CANADIAN IMGS GUIDE TO OSCE AND PRACTICE · in an OSCE scenario You can use this study guide to prepare for the NAC OSCE and MCCQE2. The content was reviewed for the most recent updates;

164

Motor/sensory deficit

Urinary

retention/Stool

incontinence

Saddle anesthesia

GI

Urinary

Do you get

tired when you

chew food?

Do you feel

stiff in the

morning and

need some

time to get

going?

Do you feel

your legs are

weak?

Is it hard to get

up from a

chair?

Do you feel

numbness or

tingling in your

legs and feet?

Is it hard to

pass urine?

Did you

become

incontinent of

stool?

Do you feel

numbness in

your buttocks?

Do you have

tummy pain?

Vomiting?

Diarrhea or

constipation?

No

No

Not at all

No

The same area

that hurts feels

numb sometimes

No

No

No

None of that

Page 165: CANADIAN IMGS GUIDE TO OSCE AND PRACTICE · in an OSCE scenario You can use this study guide to prepare for the NAC OSCE and MCCQE2. The content was reviewed for the most recent updates;

165

Obstetric/gynecologic

Is it burning or

painful to pass

urine?

Were you ever

diagnosed with

kidney stones?

Have you

noticed a

change in urine

color or smell?

Are your

periods

regular?

Have you

noticed a

change in the

amount or

timing of

bleeding?

Do you have

vaginal

discharge or

pelvic pain?

No

No

No

Yes

No

None of that

The rest of red flags

for back pain

Cancer diagnosis

IV drug use

Steroids use

Trauma

Have you ever

been

diagnosed with

cancer?

Do you use

Intravenous

drugs?

Do you take

steroids?

Have you ever

had a trauma

to your back?

No

No, I would never

do that

No

No, never

Page 166: CANADIAN IMGS GUIDE TO OSCE AND PRACTICE · in an OSCE scenario You can use this study guide to prepare for the NAC OSCE and MCCQE2. The content was reviewed for the most recent updates;

166

Osteoporosis Have you ever

had fractures?

No

Function component

of FIFE

Is the pain

affecting your

function or

mood?

I understand,

we will do our

best to figure it

out, is it ok if I

ask you few

more

questions?

Not my function,

but it’s

frustrating, I want

to know what’s

wrong

Absolutely

Previous treatment Have you seen

a doctor or

chiropractic

before?

No

Past Medical and

surgical history

Do you have

any other

disease like;

heart disease,

diabetes or

high blood

pressure?

Were you ever

diagnosed with

Psoriasis,

Inflammatory

bowel disease

or a sexually

transmitted

infection?

Have you ever

had a surgery?

No

No

No

No

Family history

Page 167: CANADIAN IMGS GUIDE TO OSCE AND PRACTICE · in an OSCE scenario You can use this study guide to prepare for the NAC OSCE and MCCQE2. The content was reviewed for the most recent updates;

167

Did anyone in

your family

have cancer?

No, they were all

healthy

Medications and

allergy

Do you take

any

medication?

Are you

allergic to food

or medication?

Tylenol once or

twice a week

No

Social history

Smoking Do you smoke? No

Alcohol Do you drink

alcohol?

Occasionally

Illicit drug use Do you use

recreational

drugs?

No

Occupation What do you

do for living?

Did you miss

days of work

because of

back pain?

I’m a teacher

Not yet!

Living conditions With whom do

you live?

With my husband

and 20 years old

daughter

Exercise Do you

exercise?

No

FIFE Thank you for

sharing all this

information

with me. Do

you have any

questions for

me?

No, thank you for

being thorough

Physical examination;

Page 168: CANADIAN IMGS GUIDE TO OSCE AND PRACTICE · in an OSCE scenario You can use this study guide to prepare for the NAC OSCE and MCCQE2. The content was reviewed for the most recent updates;

168

- Examine the back – page 42, chapter 5

- Pay particular attention to sensory and motor deficit and try to determine the level of

the lesion. Most common for herniated disk are; L4-L5 and L5-S1

- Inspect other joints for redness and swelling, note any skin rash or eyes redness

- As included in back examination mention that you would do a rectal exam

- Palpate the epigastrium for AAA which may cause low back pain

- P.E in this case was normal except for positive right straight leg raise

Sample questions you may be asked by the examiner;

1- What are your top 3 differentials?

Answer; Disk herniation, Lumbar muscle strain, Lumbar spinal stenosis

2- What investigations would you order?

Lumbar spine XR

Lumbar spine MRI

3- How would you treat this patient?

Answer; hot compressors, continue daily activities as tolerated, symptom guided

strengthening exercise, analgesics (NSAID like voltaren, Tylenol, opioids if needed.) ,

Cortisone injection if facet arthropathy

Rating scales points;

- Examinee introduced self and position

- Addressed patient with name

- Used proper non-verbal communication

- Organizational skills

- Looked for emergent conditions

- Examinee spoke clearly (accent didn’t get in the way)

- Listening and questioning skills

- Showed rapport with patient

- Attentive to patient physical comfort

- Medical knowledge adequate

- No misinformation was provided to patients

- No concerning ethical/legal issues

- The examinee is respectful of other health care members

Variations of the case and some tips;

Page 169: CANADIAN IMGS GUIDE TO OSCE AND PRACTICE · in an OSCE scenario You can use this study guide to prepare for the NAC OSCE and MCCQE2. The content was reviewed for the most recent updates;

169

1- Case; 67 M, known to have Prostate cancer presenting with sudden onset low back pain

and a limp

History; low back pain radiating to left lateral thigh and leg. Numbness is felt in the

same distribution. The patient had saddle anesthesia and normal bowel and bladder

function.

Physical examination; power of big toe extension and ankle plantar flexion 3/5.

Ankle jerk was absent. The patient had decreased light touch sensation over L5, S1.

When you mentioned you would do a rectal exam and examine sensations of the

perianal area the examiner says; anal sphincter tone decreased and perennial light

touch sensation is decreased.

Discussion; sudden onset Back pain combined with neurological deficit is an

emergency. Differential includes; Cauda equina, conus medullaris and epiconus

syndrome. Cauda equina is typically lower motor neuron while the other two are

upper motor neuron. Some causes include; tumors, abscess, large herniated disk

Prompt diagnosis with spinal MRI (preferred) or CT and surgical decompression

within 48 hours can save the patient’s function.

2- 28 year old male, complaining of low back pain and stiffness.

History; you find that the patient has 5 months history of moderate-severe non-

radiating low back pain, stiffness and fatigue. The pain is worse in the morning and

sometimes awakens him from sleep. No history of trauma. No constitutional

symptoms. No neurological or urinary symptoms. He had a bout of bloody diarrhea 4

weeks ago that lasted about one week. His back pain was more severe and he took a

lot of Advil. He didn’t see a doctor for the back pain or diarrhea because he doesn’t

like to take drugs but the pain is not going away. No skin, eye or other

rheumatologic symptoms. No other abdominal symptoms. He is a smoker for 8 years

1 pack per day. No constitutional symptoms, infections or unsafe sex practices. His

sister was diagnosed with Crohn’s disease a week ago

Physical exam; back movements are limited. Positive modified Schober test of 3 cm.

Right SI joint was tender

Page 170: CANADIAN IMGS GUIDE TO OSCE AND PRACTICE · in an OSCE scenario You can use this study guide to prepare for the NAC OSCE and MCCQE2. The content was reviewed for the most recent updates;

170

Discussion; As detailed in the history section, rule out neurological deficits and red

flag. This patient probably has Ankylosing Spondylitis (AS); An inflammatory

condition that mainly involves the spine and SI joint but can involve peripheral

joints. If you suspect AS make sure you cover extra articular manifestations mainly

inflammatory bowel disease (IBD), psoriasis, uveitis and dactylitis. Investigations

include HLA B27, C-reactive protein, ESR, spine XR, SI XR, and spine and SI MRI.

Encourage exercise, physical therapy and start the patient on full dose regular

NSAIDs. Refer to a rheumatologist for further evaluation and management.

3- 65 year old female complaining of low back pain radiating to both lower extremities

History; moderate low back pain and bilateral buttocks and thigh pain increased by

prolonged standing and walking and improved by sitting or bending forward. No

other symptoms. No cardiovascular risk factors apart from age. The patient had

laminectomy for a herniated disk at age 55.

Physical examination; unremarkable and normal lower limbs pulses.

Discussion; this patient is presenting with neurogenic claudication. Make sure you

rule out intermittent claudication seen in peripheral vascular disease. The presenting

complaint plus a history of back surgery make the diagnosis of lumbar spine stenosis

highly likely. Imaging studies preferably MRI aid in the diagnosis. Treatment includes

physical therapy, analgesics and decompression surgery with or without fusion for

severe cases.

4- Some tips for localization;

L4-L5 (L5 radiculopathy) and L5-S1 (S1 radiculopathy) more common

Foot drop, weak dorsiflexion, big toe extension and hip abduction L5

Weak plantar and knee flexion S1

Ankle jerk S1

Patellar reflex L4

Refer to chapter 3 page 13 for dermatomal distribution

Page 171: CANADIAN IMGS GUIDE TO OSCE AND PRACTICE · in an OSCE scenario You can use this study guide to prepare for the NAC OSCE and MCCQE2. The content was reviewed for the most recent updates;

171

Case 6; Knee pain

Edited by; Dr. James Yeung

Door sign

Mary Smith, 65 years old female, admitted for heart failure that was treated. Now

has knee pain and swelling. You are asked to see her for her knee pain

Vital signs are;

BP 110/70

PR 86

RR 14

Temperature 37⁰C

In the next 10 min conduct a focused and relevant history and conduct a focused

and relevant physical exam. As you do the P.E explain to the examiner what you are

doing and your findings. At 9 min the examiner may ask you a question or questions.

Note; The NAC exam may have a similar stem but the last 2 line will look like this; in the next 11

minutes conduct a focused and relevant history and conduct a focused and relevant physical

exam. As you do the P.E explain to the examiner what you are doing and your findings. At 8 min

the examiner may ask you a question or questions

Sample notes to write for yourself;

Mary Smith Gout

65 F Pseudo gout

Septic arthritis

C/C; Knee pain and swelling Rheumatoid arthritis, Inflammatory arthritis

Hx and P.E Trauma/ligamentous injury/fracture

Patient encounter

Page 172: CANADIAN IMGS GUIDE TO OSCE AND PRACTICE · in an OSCE scenario You can use this study guide to prepare for the NAC OSCE and MCCQE2. The content was reviewed for the most recent updates;

172

Your actions Suggested verbalizing Patient

response

Opening start I understand that heart

failure brought you to

hospital and is under

control

Tell me about your knee

pain

Yes, I feel

much

better

My left

knee is

very

painful

since

yesterday

Onset Did it suddenly become

painful

Yes

Swelling

Redness

Hotness

Is it swollen?

Is it red?

Is it hot?

Yes

Yes

Yes

Severity of pain

and function

How severe is the pain on

a scale from 1-10, 10

being the worst pain ever?

Can you move your knee?

It’s 9

No, it’s

very

painful

Exacerbating

factors

What makes the pain

worse beside movement?

Just

movemen

t

Relieving factors What makes the pain

better

They gave

me some

Percocet,

helped a

little

History of trauma Did you have trauma to

your knee?

No

Swelling/pain of

other joints

Have you ever had

swelling or pain of other

joints?

No

Fever/chills Have you had fever/chills? No

Gout precipitants

Page 173: CANADIAN IMGS GUIDE TO OSCE AND PRACTICE · in an OSCE scenario You can use this study guide to prepare for the NAC OSCE and MCCQE2. The content was reviewed for the most recent updates;

173

Diuretics/dehydra

tion

Meat intake

Etoh

Were you given extra

water pills the last week

for heart failure?

Did you consume a lot of

meat?

Do you drink alcohol

Yes, I’m

back to

my home

dose now

but I was

given an

extra 2

pills every

day

I eat a

serving

every day

No

Rheumatologic

Disease

Morning stiffness

Skin/hair changes

Raynaud’s

Mouth ulcers

Proximal muscle

weakness

Do you feel stiff in the

morning that you need

some time to get going?

Do you get skin rash?

Any loss of hair?

Does your fingers color

change in the cold?

Do you have mouth

ulcers?

Do you find it difficult to

stand up from a chair or

comb your hair?

No

No

No

No

No

No

Additional

symptoms

CVS/Pulmonary

GI

Do you have chest pain?

Do you have shortness of

breath?

Can you lie flat? Do you

wake up at night short of

breath?

Do you have swelling

around your ankles?

Do you cough?

No

Not any

more

Not any

more

All gone

now

No

Page 174: CANADIAN IMGS GUIDE TO OSCE AND PRACTICE · in an OSCE scenario You can use this study guide to prepare for the NAC OSCE and MCCQE2. The content was reviewed for the most recent updates;

174

Urinary

Do you have abdominal

pain/diarrhea/constipatio

n?

Does it burn to pee?

Did the color of urine

change?

No

No

Constitutional

Weight loss

Night sweats

(Fever covered

above)

Have you lost weight

recently?

Do you get drenching

night sweats that you

need to change the

sheets?

No

No

Recent travel Have you traveled

recently?

No

Gonorrhea

infection

Do you have vaginal

discharge?

Have you ever had a

sexually transmitted

infection?

No

No

Osteoporosis Have you ever had

fractures?

Have you ever been

diagnosed with

osteoporosis?

Do you take hormone

replacement therapy?

No

No

I took it

for 5

years

about 8

years ago

Past medical and

surgical history

Besides heart failure do

you have any other

disease?

I have

high

blood

pressure

and high

Page 175: CANADIAN IMGS GUIDE TO OSCE AND PRACTICE · in an OSCE scenario You can use this study guide to prepare for the NAC OSCE and MCCQE2. The content was reviewed for the most recent updates;

175

Were you ever diagnosed

with cancer?

Have you had any

surgeries?

cholester

ol

No

No

Medications and

allergy

What medications do you

take?

Sure, I will check with your

nurse, Are you allergic to

medications or drugs?

I take

aspirin,

water pill,

Lipitor

and a

couple

other

medicatio

ns, you

can check

with my

nurse

because I

believe

they

made

some

changes

No

Family history

Does anyone in your

family have gout?

Rheumatoid arthritis or

other rheumatic disease?

Cancer?

My father

had gout

No

No

Social history

Do you smoke? No, I

never did

Do you drink alcohol? No

Page 176: CANADIAN IMGS GUIDE TO OSCE AND PRACTICE · in an OSCE scenario You can use this study guide to prepare for the NAC OSCE and MCCQE2. The content was reviewed for the most recent updates;

176

Do you use recreational

drugs?

No never

Are you still working? No, I’m

retired, I

used to

be a clerk

With whom do you live?

How is his health?

With my

husband

He is

strong like

a horse

Do you exercise? I walk my

dog every

day

Do you have social

support?

Oh ya,

lots, I

have 3

children

and they

are all

wonderful

and my

husband

helps with

everythin

g

FIFE Thank you for sharing all

this information, do you

have any questions for

me?

Not at the

moment,

thank you

doctor

Physical examination:

- Examine the knee as detailed in chapter 3

- Inspect other joints for swelling, redness, deformities

- Inspect the skin for rash and mouth for ulcers

- Look at the hands and extensor surfaces for tophi seen in gout

Page 177: CANADIAN IMGS GUIDE TO OSCE AND PRACTICE · in an OSCE scenario You can use this study guide to prepare for the NAC OSCE and MCCQE2. The content was reviewed for the most recent updates;

177

Sample questions you may be asked by the examiner;

1- What are your top 3 differentials?

Answer; Gout, Pseudo gout, Septic arthritis

2- What investigations would you order?

Knee XR

Knee joint aspiration; send fluid for; gram stain and culture, crystals, cell count and

differential

Rating scales points;

- Examinee introduced self and position

- Addressed patient with name

- Used proper non-verbal communication

- Organizational skills

- Recognized and managed emergency effectively prioritizing actions

- Examinee spoke clearly (accent didn’t get in the way)

- Listening and questioning skills

- Showed rapport with patient

- Attentive to patient physical comfort

- Medical knowledge adequate

- No misinformation was provided to patients

- No concerning ethical/legal issues

- The examinee is respectful of other health care members

Variations of the case and some tips;

1- Case; 28 year old female presenting with knee pain and swelling

History; Sudden onset left knee pain while skiing; her left foot got planted, and she

felt pain and heard a pop as she was trying to move forward down the hill. Less than

an hour later her knee swelled. No other joints involved. No other symptoms. She is

finding it hard to move her knee because of the pain, Tylenol and Advil helped a

little. No previous injuries or surgeries.

Physical exam; Swollen left knee, positive effusion, flexion and extension limited by

pain, positive anterior drawer test.

Page 178: CANADIAN IMGS GUIDE TO OSCE AND PRACTICE · in an OSCE scenario You can use this study guide to prepare for the NAC OSCE and MCCQE2. The content was reviewed for the most recent updates;

178

Discussion; This patient presents with acute injury most likely of the Anterior

Cruciate ligament. The first thing to do is an XR and orthopedic consult. Pain control

is very important and the patient will most likely need opioids. Rest, cold

compressors are other things that may help till definitive diagnosis and treatment is

confirmed. Note that you need to explore the mechanism of injury in trauma cases,

particularly knee position and direction of force. Rule out previous injuries or

surgeries and do quick screens for other causes of knee pain especially infection.

2- Case; 20 year old male presenting with right knee pain and swelling

History; The patient describes 3 days history of knee pain, swelling and redness. No

history of trauma. He had chills and fever. No skin rash. He has a history of syphilis

treated a year ago. He had multiple sexual partners the last 3 months and doesn’t

use condoms. No previous or other joint pain and swelling, no rheumatologic or

other symptoms. No IV drug use.

Physical exam; Temperature 38⁰C. No skin rash. No other joints swelling. Hands and

face exam normal. Auscultation of the heart and lungs was normal. The right knee

was swollen, warm, and red with limitation of movement due to pain and positive

effusion.

Discussion; Septic arthritis is the top differential, make sure to rule out other

possibilities. Get an XR and aspirate the joint to confirm the diagnosis. Screen for

sexually transmitted infection and offer testing for HIV. Septic arthritis is treated

with empiric IV antibiotics till gram stain and cultures are back. Antibiotic choice

depends on the most probable organism and local sensitivities. In this case

gonococcus is the most likely cause; treat with ceftriaxone 1g IV daily. In IV drug

users Strep or Staph are more common in and Vancomycin is a more appropriate

initial treatment.

3- The basic principles for history apply for other joints. Patient age, presentation and risk

factors help sort out the differential. Make sure you master the physical examination of

all joints detailed in chapter 3.

Page 179: CANADIAN IMGS GUIDE TO OSCE AND PRACTICE · in an OSCE scenario You can use this study guide to prepare for the NAC OSCE and MCCQE2. The content was reviewed for the most recent updates;

179

Case 7; Diabetic Ketoacidosis (DKA)

Edited by; Dr. Sadik Salman

Door sign

Nichole Smith, an 18 years old female presenting to ER with abdominal pain

Vital signs;

BP 110/75

PR 100

RR 22

Temperature 38.2⁰C

Blood glucose 22

ABG done; PO₂ 88, PCO₂ 25 , HCO₃ 12 , PH 7.33

There is a nurse in the room

In the next 9 min manage the patient. Ask the nurse to do any orders you deem

necessary. As you examine the patient explain to the examiner what you are doing

and your findings. At 10min the examiner will ask you a question or questions

Note; In this case you will be communicating with the nurse and the patient.

Communication with the nurse is presented in italic. You need to make sure the

patient is stable, give emergency medications, take a focused and relevant history,

perform a focused and relevant P.E and order investigations. The P.E is included in

the case. Treat the nurse with respect, and give clear orders specifying the dose and

route of medications.

Even though it is an ER case, it is important that you introduce yourself to the nurse

and patient.

Sample notes to write for yourself;

Nichole Smith 18F DKA

Gastroenteritis

Appendicitis

Abd pain- Mx Ovarian abscess/torsion

Febrile, tachypnic tachycardic Inflammatory bowel disease

IBD

Page 180: CANADIAN IMGS GUIDE TO OSCE AND PRACTICE · in an OSCE scenario You can use this study guide to prepare for the NAC OSCE and MCCQE2. The content was reviewed for the most recent updates;

180

Patient encounter;

Your actions Suggested

verbalizing

Patient or nurse

response

ABC I will start with

ABC, The patient is

alert, breathing

spontaneously,

protecting her

airways, I’m

feeling the pulse it

is regular. The

patient is

tachypnic and

tachycardic.

Oxygen and IV

fluids

The patient is

maintaining her

oxygenation, can

we get 2 large

pore IVs, and give

her normal saline

0.9% 1 liter IV

bolus then

200cc/hour

Given

Monitor Can you put the

patient on cardiac

monitor please

and check vital

signs every 15 min

Done

Brief History

So Nichole, your

blood sugar is

high, Have you

Yes, for the last 6

years

Page 181: CANADIAN IMGS GUIDE TO OSCE AND PRACTICE · in an OSCE scenario You can use this study guide to prepare for the NAC OSCE and MCCQE2. The content was reviewed for the most recent updates;

181

been diagnosed

with diabetes?

And it’s type 1,

right?

Yes

And what brought

you to the

hospital?

I had tummy pain

and diarrhea,

couldn’t eat and

so didn’t take my

insulin

Details about the

abdominal pain

Is this the first

time you have this

pain?

Where is it?

How severe is it on

a scale from one to

10, 10 being the

worst pain ever?

What makes it

worse?

What makes it

better?

You said you also

had diarrhea, how

many bowel

movements?

Any blood or

mucus in the

stool?

Have you had any

other symptoms

like vomiting,

nausea, burning

urination, change

in urine or stool

Yes

It started

here(patient

points at

umbilicus) then

moved her

(patient points at

right lower

quadrant)

It’s 9

I don’t know

Advil helped a

little

3 watery

No

I felt a bit

nauseated

Page 182: CANADIAN IMGS GUIDE TO OSCE AND PRACTICE · in an OSCE scenario You can use this study guide to prepare for the NAC OSCE and MCCQE2. The content was reviewed for the most recent updates;

182

color or flue like

symptoms?

Did you eat in a

restaurant lately?

Have you been in

contact with

someone sick?

No

No

No

DM history What insulin type

and dose are you

on?

Which insulin dose

did you drop

today?

Do you check your

blood sugar levels?

What are the

values?

Do you know

what’s your

HBA1c?

Have you had

diabetic

ketoacidosis

before?

Were you

admitted to the

ICU?

What was it about

2 years ago?

I take 18 units

NPH in the

morning and 18

units in the

evening and I take

Humalog before I

eat usually 5-8

I didn’t take any

today because I

didn’t eat

I hate it so I don’t

always do it, but I

usually check

once or twice a

day

8-12

It was 7.2 a

month ago

Yes twice; when I

was diagnosed,

and 2 years ago?

No

I had the flu and

couldn’t eat or

take insulin

Page 183: CANADIAN IMGS GUIDE TO OSCE AND PRACTICE · in an OSCE scenario You can use this study guide to prepare for the NAC OSCE and MCCQE2. The content was reviewed for the most recent updates;

183

Did you get your

eyes checked?

Good for you, I’m

glad to hear that.

Have you had any

other

complications

from diabetes?

Yes, a year ago,

and it was fine, I

take very good

care of myself I

don’t want to die

from diabetes

No

Investigations Can I get an ECG,

ABG

And the following

blood work; CBCD,

Creatinine, urea,

Lytes, PO₄, and

urine analysis and

culture

Sure

Ok, will draw

blood now, results

will take 15 min

Management Can you please

give 10 units

regular insulin IV,

then 0.5 U/hour,

and do a chem

strip after an hour

Will do

Relevant review

of systems

Have you had

cough?

Chest pain?

Skin rash?

Vaginal

discharge/itching?

Fever/chills at

home?

When was your

last menstrual

period?

No

No

No

No

I’m not sure

A week ago

Page 184: CANADIAN IMGS GUIDE TO OSCE AND PRACTICE · in an OSCE scenario You can use this study guide to prepare for the NAC OSCE and MCCQE2. The content was reviewed for the most recent updates;

184

Other relevant

history

Do you have any

other disease?

Have you ever had

surgeries?

Do you take any

other

medications?

Do you have

allergies to food or

medications?

Do you smoke?

Do you drink

alcohol?

Do you use

recreational

drugs?

Are you sexually

active?

Do use

contraception?

Have you ever had

a sexually

transmitted

infection?

No

No

Tylenol or Advil

occasionally

No

No

No

No

Yes, with my

boyfriend

We use condoms

No

Order

investigations

Can I get the

following

investigations as

well;

B-hcg

CXR

Will add B-hcg to

the blood I drew

and will order

CXR, Do you want

to look at the

ECG?

The nurse hands

you an ECG

Thank you, Normal

rate, rhythm, and

axis. P waves and P

wave intervals are

normal. QRS, T

Page 185: CANADIAN IMGS GUIDE TO OSCE AND PRACTICE · in an OSCE scenario You can use this study guide to prepare for the NAC OSCE and MCCQE2. The content was reviewed for the most recent updates;

185

wave and QT

normal, No U

waves

Physical

examination

The patient is

tachypnic,

tachycardic, and

febrile. The hands

are normal, no skin

or nails changes.

No pallor, no skin

rash, no fetor

hepaticus, no

jaundice. Can you

open your mouth

please, no throat

redness, no

cyanosis. The

mucous

membranes are

mildly dry. No skin

turgor. I will test

orthostatic vitals

at the end.

I’m going to listen

to your heart and

lungs; normal S1,

S2, no S3, S4, no

murmurs, no rubs.

Breath sounds are

symmetrical

bilaterally, no

wheezes, no

crackles. No rubs.

Next I will examine

the abdomen. I’m

going to examine

your tummy, let

me know if it

Page 186: CANADIAN IMGS GUIDE TO OSCE AND PRACTICE · in an OSCE scenario You can use this study guide to prepare for the NAC OSCE and MCCQE2. The content was reviewed for the most recent updates;

186

hurts; staring with

inspection, no

masses, no

distension, the

abdomen is

symmetrical and

moving with

respiration. No

caput medusa, no

Grey turners or

Cullins sign. No

stria.

I’m now listening

to bowel sounds,

they are normal.

Do you have pain

right now? Yes in

here (right iliac

fossa)

Next I will do

superficial

palpation starting

from the left iliac

fossa, there is

tenderness in the

right lower

quadrant, then

deep palpation, no

masses, there is

tenderness in the

right lower

quadrant. I will

test now for

specific signs of

appendicitis,

pressing on

McBurney’s point,

there is pinpoint

Page 187: CANADIAN IMGS GUIDE TO OSCE AND PRACTICE · in an OSCE scenario You can use this study guide to prepare for the NAC OSCE and MCCQE2. The content was reviewed for the most recent updates;

187

tenderness. Does

it hurt when I let

go? Yes. And

rebound

tenderness. I

noticed that

rouvsings was

negative. I’m going

to move your leg,

relax it for me and

let me know if you

have pain; flexion,

then internal

rotation; Psoas

and Obturator

signs are negative.

Next I will palpate

for hepatomegaly

in the mid

clavicular line, can

you breathe in and

out for me please?

The liver is not

palpable. No I’m

palpating for the

spleen using the

same technique.

The gallbladder is

not palpable. Can

you take a deep

breath for me

please? Murphy’s

sign is negative.

Can you set up for

me? I’m checking

for renal angle

tenderness and it’s

negative.

Page 188: CANADIAN IMGS GUIDE TO OSCE AND PRACTICE · in an OSCE scenario You can use this study guide to prepare for the NAC OSCE and MCCQE2. The content was reviewed for the most recent updates;

188

Next I will examine

the lower limbs;

I’m going to look

at your legs. No

swelling, no

redness, no

deformity, no

dilated or tortuous

veins, no cyanosis,

no ulcers, no skin

or nail changes.

I’m going to press

on your calves, let

me know if it

hurts. No calves’

tenderness. Finally

I would like to do a

rectal and vaginal

exams

The examiner says

vaginal and rectal

examinations

normal.

Investigations and

management

Please give

another bolus

normal saline 0.9%

IV over 1 hour

then run normal

saline at 200

cc/hour.

Give the patient

Percocet 1 tab

now with sips of

water then keep

NPO

Can you order US

of the abdomen to

rule out

appendicular or

ovarian pathology.

I will continue to

manage DKA, but

Page 189: CANADIAN IMGS GUIDE TO OSCE AND PRACTICE · in an OSCE scenario You can use this study guide to prepare for the NAC OSCE and MCCQE2. The content was reviewed for the most recent updates;

189

will consult

surgery in the

mean time for

possible

appendicitis.

Social support Thank you Nichole

for all the

information, You

have a DKA. Your

abdominal pain

could be due to

DKA, or inflamed

appendix. I will ask

the surgeons to

come and take a

look. Would you

like me to talk to

one of your family

or your boyfriend?

Thank you, Can

you please

explain to my

boyfriend what is

going on. He is

very worried.

No problem

Sample questions you may be asked by the examiner

1- What is your preliminary diagnosis?

Answer; DKA secondary to appendicitis, tubo-ovarian abscess/rupture or ovarian

torsion.

Rating scales points

- Examinee introduced self and position

- Addressed patient with name

- Used proper non-verbal communication

- Organizational skills

- Recognized and managed emergency effectively prioritizing actions

- Examinee spoke clearly (accent didn’t get in the way)

- Listening and questioning skills

- Showed rapport with patient

- Attentive to patient physical comfort

Page 190: CANADIAN IMGS GUIDE TO OSCE AND PRACTICE · in an OSCE scenario You can use this study guide to prepare for the NAC OSCE and MCCQE2. The content was reviewed for the most recent updates;

190

- Medical knowledge adequate

- No misinformation was provided to patients

- No concerning ethical/legal issues

- The examinee is respectful of other health care members

Variations of the case and some tips;

1- Case; 18 year old female presenting with abdominal pain, blood sugar of 25 and anion

gap metabolic acidosis

History; Type 1 DM diagnosed at age 10. Well controlled till the last year when she

had 4 episodes of DKA. Patient talks madly about her boyfriend and repeatedly

describes him as a jerk. When you ask you find that they have been together for 1

year but the relationship is not smooth. When troubles arise she threatens with

either not taking insulin or taking too much. All her previous DKAs were due to

missed insulin. She also had 5 iatrogenic hypoglycemic attacks. No ICU admission.

The abdominal pain is non-specific and she doesn’t have other symptoms. No

alcohol or drug abuse. No depression or suicidal intent, she just got mad with her

boyfriend and is seeking attention.

Physical exam; Dry mucous membranes. Otherwise normal

Discussion; This is an emergency case, so you need to focus on managing DKA. And

although the patient intentionally didn’t take insulin you still need to rule out other

possible precipitants, especially intoxication. The patient has some social stressors

and is showing features of border line personality disorder so it is important to

consult psychiatry once she is medically cleared. Ask the patient if you could speak

to her parents for collateral history.

2- In addition to stabilizing patients and initiating management in DKA patients, it is

important that you dig deep into the precipitant; 6Is; insulin missed, iatrogenic (like

glucocorticoids), infection, inflammation, ischemia and intoxication. Once the cause is

recognized, it needs to be treated.

3- Note that DKA patients can be tripped off by precipitants even if they continue to take

insulin

Page 191: CANADIAN IMGS GUIDE TO OSCE AND PRACTICE · in an OSCE scenario You can use this study guide to prepare for the NAC OSCE and MCCQE2. The content was reviewed for the most recent updates;

191

Case 8; DM history and counseling

Edited by; Dr. Sadik Salman

Door sign

Jonathan Adams, a 55 years old male who comes to your clinic for diabetes follow up.

Vital signs;

BP 120/80

PR 80

RR 13

Temperature 36.5

In the next 10 min counsel the patient. At 9 min the examiner may ask you a question or questions.

Note; The NAC exam may have a similar stem but the last 2 line will look like this; in the next 11 minutes

counsel the patient. At 8 min the examiner may ask you a question or questions

Sample notes to write for yourself

Jonathan Adams DM type, onset

55M medications

Diet and exercise

History and counsel DM monitoring and control

Complications

Feet

CV risk factors

Patient encounter

Your actions Suggested verbalizing Patient response

Page 192: CANADIAN IMGS GUIDE TO OSCE AND PRACTICE · in an OSCE scenario You can use this study guide to prepare for the NAC OSCE and MCCQE2. The content was reviewed for the most recent updates;

192

Opening start How can I help you today I’m here to check on my

diabetes

DM history

Type

Diagnosis

Medications

Control and monitoring

Complications

What type of diabetes do u

have?

When was it diagnosed?

What medications are you on?

For how long you’ve been on

them?

Do you check your sugar levels?

How often?

What values do you get?

Do you have your log book?

No worries, Do you know what

was your last hemoglobin A1C

and when?

Did you get your eyes checked?

Have you had problems with

feet ulcers?

Have you ever had a heart

attack?

Do you feel numbness or

tingling in your hands and feet?

Did anyone tell you before you

have protein in your urine?

Do you get low blood sugars?

Type 2

5 years ago

Glucophage I take 850mg twice

a day and Gliclazide 80mg daily

3 years

Yes I do

2 times every day

Usually 6-13

Sorry I forgot it at home

It was over 6 months ago, I

think it was 7.5

5 years ago, they were fine

No never

No

Sometimes I feel the tips of my

fingers are numb

No

Once in a while

Page 193: CANADIAN IMGS GUIDE TO OSCE AND PRACTICE · in an OSCE scenario You can use this study guide to prepare for the NAC OSCE and MCCQE2. The content was reviewed for the most recent updates;

193

Education

Follow up

Diet and exercise

Like how often?

What is the lowest number you

got?

What do you do when you get a

low?

What symptoms do u get when

its low?

Did you ever need help because

of low sugars?

Have you ever had a surprise;

you checked your sugar and it

was low but you feel fine?

Have you been to diabetes

education before?

Who follows your diabetes?

Do you follow a diabetes diet

low in sugar?

Do you exercise?

Once a month usually if I don’t

eat and walk the dog

3

I take some sugars then I eat

I feel dizzy and become shaky

No

No

No

My family doctor used to, but

he retired 6 months ago

I try, but I cheat sometimes

I walk my dog every day

Other cardiovascular risk factors

Hypertension

Dyslipidemia

Do you have high blood

pressure?

Do you take a medication for it?

What is your blood pressure

usually?

Do you have high cholesterol?

Yes

Coversyl 4mg

129/70

Not that I know of

Page 194: CANADIAN IMGS GUIDE TO OSCE AND PRACTICE · in an OSCE scenario You can use this study guide to prepare for the NAC OSCE and MCCQE2. The content was reviewed for the most recent updates;

194

Smoking

Family history of MI

Do you smoke?

Did anyone in your family have

a heart attack?

No

My father died of a heart attack

in his 70s

Quick review of systems Do you get chest pain?

Do you get cough?

Do you get shortness of breath?

Do you have abdominal pain?

Vomiting? Diarrhea or

constipation?

Do you have problems with

urine, like pain or difficulty

when you pass urine, change in

urine color?

Have lost or gained weight

recently?

Do you get fevers?

No

No

No

No

No

No

I gained 2 lb

No

Other past medical and surgical

history

Do you have other diseases?

Have you had any surgeries?

No

No

Other medications and allergy Do you take any other

medications? Or over the

counter medications? Non-

steroidal pain killer like Advil

Do you have allergies to

medication or food?

No

No

Relevant social history Do you drink alcohol?

Do you use recreational drugs?

Occasionally

No, never

Page 195: CANADIAN IMGS GUIDE TO OSCE AND PRACTICE · in an OSCE scenario You can use this study guide to prepare for the NAC OSCE and MCCQE2. The content was reviewed for the most recent updates;

195

What do you do for living?

With whom do you live?

Any stress at work or home?

How is diabetes affecting your

life and function?

I own a travel agency

With my wife and daughter

No, things are fine

Well, I have to watch what I eat,

and try to stay active. It’s not

limiting me, but I have to do my

homework and keep an eye on

my sugar

Counseling

What does the patient know What do you know about

diabetes?

I know I should be very careful

with what I eat and take my

medication or I may lose my

vision and get a heart attack

Patient expectations Do you have specific questions? Well, I trust that you will look

after my diabetes now that my

family doctor retired

Sure I will, thank you for

trusting me

Education and counseling

Background info As you said diabetes damages

body organs silently. In type 2

diabetes the cells become

resistant to insulin- a hormone

secreted by the pancreas to

regulate blood sugar levels, so

blood sugar levels become high.

If it stays high for too long

damage starts

Intervention and its importance That’s why we try to lower the

blood sugar with diet , exercise

and medications, and you are

on 2 good ones

Page 196: CANADIAN IMGS GUIDE TO OSCE AND PRACTICE · in an OSCE scenario You can use this study guide to prepare for the NAC OSCE and MCCQE2. The content was reviewed for the most recent updates;

196

Monitoring In addition to you checking your

blood sugar at home, we do

check the HbA1C to see how the

sugar is doing over the last 3

months, we can lower it even

further in your case and I would

like to see it around 6.5

And how would you do that?

I will increase Gliclazide dose

and if needed add a new

medication

I see

I need you to check your blood

sugar 4 times a day; before you

eat in the morning, before lunch

and dinner and before bed

time. Would you be able to do

that?

Well, I can but do I need to do it

for ever?

It is better to keep checking it 2

times a day, but I need you to

do it more often for the next 3

months because we are trying

to achieve a better control, and

because we are increasing

Gliclazide dose which may cause

your blood sugar to drop. Are

we on the same page?

Yes

Medication side effect Do you get side effects from the

medications like nausea,

stomach upset, and dizziness?

You mentioned you get low

blood sugar occasionally, I need

you to watch that, you may get

dizzy, shaky, flushing, sweating

and have a fast heart rate, take

No, I feel good

Sure, I will

Page 197: CANADIAN IMGS GUIDE TO OSCE AND PRACTICE · in an OSCE scenario You can use this study guide to prepare for the NAC OSCE and MCCQE2. The content was reviewed for the most recent updates;

197

a sugar pill and check your

blood sugar right away

Complications of diabetes

So let’s talk about the damage

diabetes can cause and what we

will do about it

Yes please I’m listening

Retinopathy It can cause blindness but if we

interfere early we can prevent

it, it is the time for an eye exam

for you

I would appreciate that

Nephropathy It can also damage the kidneys

if not well controlled, so we

need to check the protein in

your urine and your kidney

function. You are taking the

right blood pressure pill that

can help decrease the damage

It is better to avoid drugs that

can adversely affect the kidneys

like Advil and voltaren

Yes, please do. We need to

work together on this

I see, glad I’m on the right

medication

I will ask you before I take any

over the counter medication

Neuropathy The nerves can as well be

damaged if blood sugar is too

high for too long, I would like to

examine your nerves if that’s ok

That would be great

Macrovascular Diabetes, combined with other

conditions like high blood

pressure and cholesterol, and

smoking can increase the risk of

Page 198: CANADIAN IMGS GUIDE TO OSCE AND PRACTICE · in an OSCE scenario You can use this study guide to prepare for the NAC OSCE and MCCQE2. The content was reviewed for the most recent updates;

198

heart attacks, but I see that

your blood pressure is

controlled so keep the good

work, and that you don’t

smoke, this will significantly

decrease your risk, I need to

check your cholesterol level, it is

a blood test that you need to

fast 12 hours for, is that ok

For sure, I would love to get

tested

Hypoglycemia (discussed above)

Hyper osmolar coma If the sugar was out of control

and went too high, you may get

dizzy or even lose

consciousness, and in this case

you need to go to the hospital

Ok

Feet ulcers and poor wound

healing

It is a good idea to develop the

habit of looking at your feet

daily, because you may lose

your sensations, and miss some

wounds. Some people lost their

limbs because of diabetes

Oh, I will check them every day!

Summary So we talked about diabetes

and its complications and how

to prevent them. We agreed to

increase Gliclazide and do some

tests and repeat the eye exam,

do you have any questions or

concerns?

How about my diet?

Diet and exercise That’s an excellent point, you

need to avoid sugar and

carbohydrates, and keep track

of your weight. I can refer you

to the dietician for full details

about diet if you like

Yes, I need to see a dietician

Page 199: CANADIAN IMGS GUIDE TO OSCE AND PRACTICE · in an OSCE scenario You can use this study guide to prepare for the NAC OSCE and MCCQE2. The content was reviewed for the most recent updates;

199

Sure, will do. The other thing I

wanted to talk to you about is

exercise; You are doing well by

walking your dog every day. You

do need to exercise, it helps

control your diabetes, but it

needs to be balanced against

your diet so that your blood

sugar doesn’t drop

I see

Do you have any other

questions or concerns

Would I ever need insulin?

Some patients with type 2

diabetes need insulin when pills

are not enough to control their

blood sugar. We try to avoid

that by encouraging patients to

eat healthy, exercise and take

their medications. You will not

need insulin any time soon, and

may never need it if you are

careful enough. Any other

concerns?

No, thank you very much

Give brochure

Assurance and availability

I will give you a brochure about

diabetes, I want you to know

that I’m here to help you, and

that you can contact my clinic

for an appointment at any time.

Thank you, I appreciate it

Sample questions you may be asked by the examiner

1- How would you diagnose type 2 DM?

Answer; one of;

Fasting blood sugar ≥ 7

Page 200: CANADIAN IMGS GUIDE TO OSCE AND PRACTICE · in an OSCE scenario You can use this study guide to prepare for the NAC OSCE and MCCQE2. The content was reviewed for the most recent updates;

200

Random blood sugar ≥ 11.1

2 hours oral glucose tolerance test with 75 g ≥ 11.1

HbA1c ≥ 6.5

2- What is the blood pressure target in DM?

Answer; ≤ 130/80

Rating scales points;

- Examinee introduced self and position

- Addressed patient with name

- Used proper non-verbal communication

- Organizational skills

- Examinee addressed patient concerns

- Educated patient in an easy to understand language

- Examinee spoke clearly (accent didn’t get in the way)

- Listening and questioning skills

- Showed rapport with patient

- Attentive to patient physical comfort

- Medical knowledge adequate

- No misinformation was provided to patients

- No concerning ethical/legal issues

- The examinee is respectful of other health care members

Some tips

1- I strongly suggest you familiarize yourself with the Canadian diabetes guidelines;

http://guidelines.diabetes.ca/

Make sure you master the following;

Screening and diagnosis of diabetes

Target blood sugar and HbA1c

Oral drugs and their side effects

Page 201: CANADIAN IMGS GUIDE TO OSCE AND PRACTICE · in an OSCE scenario You can use this study guide to prepare for the NAC OSCE and MCCQE2. The content was reviewed for the most recent updates;

201

Insulin types, durations of action, indications, initiation of therapy and complications

Complications of diabetes

Page 202: CANADIAN IMGS GUIDE TO OSCE AND PRACTICE · in an OSCE scenario You can use this study guide to prepare for the NAC OSCE and MCCQE2. The content was reviewed for the most recent updates;

202

Case 9; Diarrhea

Edited by; Dr. Erin Toor

Door sign

Lorraine Land is a 28 year old female who comes to your clinic complaining of diarrhea.

Vital signs

BP 100/70

PR 90

RR20

Temperature 37⁰C

In the next 10 minutes, conduct a focused and relevant history and physical examination (P.E). As you do

the P.E, explain to the examiner what you are doing and your findings. At 9 min the examiner may ask

you one or more questions.

Note: The NAC exam may have a similar stem but the last 2 lines will look like this: In the next 11 minutes

conduct a focused and relevant history and physical exam. As you do the P.E explain to the examiner

what you are doing and your findings. At 8 minutes, the examiner may ask you one or more questions.

Sample Notes to write for yourself

Lorraine Land GE

28 F

IBD

Hx and P.E IBS

Patient encounter

Your actions Suggested verbalizing Patient response

Opening start How can I help you today? I have diarrhea

Page 203: CANADIAN IMGS GUIDE TO OSCE AND PRACTICE · in an OSCE scenario You can use this study guide to prepare for the NAC OSCE and MCCQE2. The content was reviewed for the most recent updates;

203

Tell me more about it It is been going on for a month

now

History of presenting illness

Diarrhea

Onset

Duration (mentioned)

Frequency

Nocturnal diarrhea

Stool consistency

Stool color

Presence of blood or mucous

Fatty stool

Relation to diet

Tenesmus

Incontinence

Previous similar episodes

Did it start gradually or

suddenly?

How many bowel movements a

day?

Do you wake up at night to

poop?

Is the stool watery or soft?

What is the stool color?

Is there blood or mucous in the

stool?

Is the stool oily? Does it stick to

the toilet bowl and is difficult to

flush?

Have you noticed that a special

diet triggers diarrhea; like

bread, oats, or dairy?

Do you feel the urge to go to

the bathroom, and then you

can’t pass stool?

Have you had any accidents?

Have you had similar episodes

of diarrhea before?

Does the diarrhea resolve with

fasting?

It was gradual, with the stool

getting more frequent over

time

I was getting 3-5, but it’s been

10 for the last week

Yes, I’m not sleeping well

because of it

Watery

Brown

There is mucous sometimes,

but no blood

No

No, I stopped some stuff, but it

didn’t make a difference

No

No

No

No

Page 204: CANADIAN IMGS GUIDE TO OSCE AND PRACTICE · in an OSCE scenario You can use this study guide to prepare for the NAC OSCE and MCCQE2. The content was reviewed for the most recent updates;

204

Risk factors

Recent travel

Sick contacts

if yes-ask details: where,

duration of travel, consumption

of local water, undercooked

food etc

Eating spoiled food or at

restaurants

Immunosuppression

Have you travelled recently?

Have you been in contact with

people with similar symptoms?

Have you eaten spoiled food or

at restaurants?

Do you have a disease or take

medications that make your

immune system weak?

No,

No

No

No, I’ve always been proud that

I don’t get sick because my

immunity is strong

Dehydration

Water intake

Thirst

Dizziness

Urine output

How many cups of water do you

drink every day?

Do you feel thirsty?

Do you feel dizzy? Did you ever

faint?

Did you notice a decrease in the

amount of urine?

10 or more with this

Sometimes

No

Yes, I’m peeing only 3 times

every day and its dark yellow

Constitutional symptoms

Weight loss

Night sweats

Have you lost weight recently?

2 lb over the last month

Page 205: CANADIAN IMGS GUIDE TO OSCE AND PRACTICE · in an OSCE scenario You can use this study guide to prepare for the NAC OSCE and MCCQE2. The content was reviewed for the most recent updates;

205

Fever

Do you get drenching night

sweats that you need to change

the sheets?

Have you had fever/chills?

No

No

Associated symptoms

Abdominal pain

Nausea/Vomiting

Jaundice

Alternating constipation and

diarrhea

Mouth ulcers

Perianal disease

Eye pain, redness

Joints pain or swelling

Skin rash

Fatigue

Menstrual irregularity

Do you have abdominal pain?

Do you have nausea or

vomiting?

Have your eyes and skin turned

yellow?

Did you have periods of

constipation followed by

diarrhea?

How about mouth sores?

Have you had pain or discharge

from the anus?

Did you have eyes pain or

redness?

Joints pain or swelling?

Skin rash?

Do you feel more tired than

usual?

Are your periods regular?

When was your last one?

Any chances you might be

pregnant?

Occasionally around my belly

button

No

No

No

I get them once in a while

No

No

No

No

Yes, I’m not as energetic as

before

Yes

A month ago

I don’t think so, we use

condoms

Page 206: CANADIAN IMGS GUIDE TO OSCE AND PRACTICE · in an OSCE scenario You can use this study guide to prepare for the NAC OSCE and MCCQE2. The content was reviewed for the most recent updates;

206

Past Medical and Surgical

history

Were you ever hospitalized or

diagnosed with another

disease?

Have you had surgeries?

No

No

Medications and allergies

Do you take any medications?

Do you take non-Steroidal pain

killers like Advil, Voltaren?

Have you taken antibiotics

recently?

Do you have allergies to food or

medications?

No,

I only take Tylenol occasionally

No

No

Family history

Does anyone in your family

have Crohn’s disease, ulcerative

colitis or cancer?

My aunt has Crohn’s disease

Social history

Smoking

Alcohol

Illicit drugs

Occupation

Living conditions

Stress

Effect on function

Do you smoke?

Do you drink alcohol?

Do you use recreational drugs?

What do you do for living?

With whom do you live?

Any stress at home or work?

Is diarrhea affecting your work

or relationship with your

husband?

No

No

No

I’m a teacher

With my husband

No

I’m more tired than usual, and

its embarrassing to leave the

class to go to the bathroom

many times

Page 207: CANADIAN IMGS GUIDE TO OSCE AND PRACTICE · in an OSCE scenario You can use this study guide to prepare for the NAC OSCE and MCCQE2. The content was reviewed for the most recent updates;

207

FIFE Thank you for sharing all this

information, would you like to

add anything?

Do you have any questions for

me?

Multiple things can cause

diarrhea. I need to examine you

and run some tests and

hopefully we will figure it out

You asked everything, thanks

for being thorough

I want to know what’s wrong

Physical examination;

- General examination: Examine the hands, face. Look for pallor, jaundice, skin rash, mouth

ulcers, joints swelling or redness and dry mucous membranes. Feel the pulse and notice if it is

thready. Assess for skin turgor

- Check postural vital signs

- Auscultate the heart and lungs

- Examine the Abdomen

- Mention that you would do a rectal exam

Sample questions you may be asked by the examiner;

1- What is your differential?

Answer: Chronic diarrhea differential: Crohn’s disease, Ulcerative Colitis, Celiac disease, Lactose

intolerance, Infectious diarrhea (particularly parasitic, CMV), Hyperthyroidism, gut malignancy, Irritable

bowel syndrome, medications (laxative abuse, PPI’s), pancreatic insufficiency, Hormonal (VIPoma,

carcinoid)

2- What initial investigations would you order?

Answer: CBCD, electrolytes (Na, K, Cl, Mg, PO4), urea, creatinine, ESR, CRP, Albumin, PT/INR, ALT, Stool

WBC, Stool for ova and Parasite, Stool culture and sensitivity, Abdominal X-Ray, will consider referral for

endoscopy/colonoscopy

3- What is your initial management?

Answer: Manage dehydration, correct electrolyte disturbances, diagnose and treat the underlying cause

of diarrhea

Page 208: CANADIAN IMGS GUIDE TO OSCE AND PRACTICE · in an OSCE scenario You can use this study guide to prepare for the NAC OSCE and MCCQE2. The content was reviewed for the most recent updates;

208

Rating scales points;

- Examinee introduced self and position

- Addressed patient with name

- Used proper non-verbal communication

- Organizational skills

- Examinee spoke clearly (accent didn’t get in the way)

- Listening and questioning skills

- Showed rapport with patient

- Attentive to patient’s physical comfort

- Medical knowledge adequate

- No misinformation was provided to patients

- No concerning ethical/legal issues

- The examinee is respectful of other health care members

Variations of the case and some tips;

1- Case: 10 year old girl presenting with diarrhea and vomiting, obtain history from the mother.

History: Nausea and vomiting started 3 hours after eating fried rice in a local Chinese restaurant.

Associated with abdominal pain, 10 hours later she developed nausea and vomiting followed by watery

diarrhea. History is suggestive of moderate-severe dehydration. No fever or other symptoms. The

mother ate the same food and had similar but less severe symptoms. No previous episodes, no other

symptoms

Physical exam: Although not asked to perform a physical exam, indicate to the mother that you need to

examine her daughter, do some blood tests and give her intravenous fluids

Discussion: Food poisoning/infectious gastroenteritis is usually diagnosed by history. The suspected

pathogen in this case is Bacillus Cereus. Supportive management that includes fluid resuscitation and

correction of electrolytes disturbances. Symptoms last 24-48 hours. If the duration is prolonged consider

other diagnoses.

2- Case: 42 year old male presenting with diarrhea

Page 209: CANADIAN IMGS GUIDE TO OSCE AND PRACTICE · in an OSCE scenario You can use this study guide to prepare for the NAC OSCE and MCCQE2. The content was reviewed for the most recent updates;

209

History: A day after returning from a trip to Mexico, he had abdominal cramps, nausea, vomiting and

diarrhea. He stayed in a 5 star resort over there. His wife, who accompanied him to the trip, had similar

symptoms. His urine output has decreased and he feels thirsty. His father died of colon cancer at age 50.

No constitutional symptoms, no similar episodes previously.

Physical examination: Normal vitals and postural vitals, dry mucus membranes. Normal chest and

abdominal physical exams.

Discussion: Diarrhea in a returning traveler could be due to traveler’s diarrhea, which might be caused

by viruses, bacteria, or parasites. Keep the differential wide. Rehydrate the patient and correct

electrolytes abnormalities. Do stool testing, and treat as needed. If diarrhea persists then investigate

further. Note that this patient is due for colon cancer screening, and once the diarrhea had subsided he

should be screened by colonoscopy. This could as well be his first presentation of malignancy.

3- 35 year old female, is being treated for Pneumonia with Ceftriaxone and Azithromycin. Now has

diarrhea

History: Diagnosed with Pneumonia and started treatment 10 days ago. She has symptoms of watery

diarrhea, cramps and fever. No Previous episodes. Her respiratory symptoms are improving. No other

symptoms. She didn’t eat spoiled food or at restaurants recently.

Physical exam: Temperature 37.9⁰C . Other vital signs normal, no postural drop. She had left lower lobe

crackles, abdominal exam was normal. No blood on rectal exam

Discussion: In addition to the regular testing, it is important to rule out Clostridium Difficile (C.diff) colitis

in cases of recent or current antibiotic use. Test the stool for C.diff toxin. Keep in mind possible

complications like toxic mega colon, electrolyte disturbances, volume depletion and bowel perforation.

Manage with fluid resuscitation, management of electrolyte disturbances if present and treat with

Flagyl. One could continue the previous antibiotic course until finished.

4- Diarrhea has a very wide differential that can be narrowed down by history and physical. Make

sure to develop your own approach.

Page 210: CANADIAN IMGS GUIDE TO OSCE AND PRACTICE · in an OSCE scenario You can use this study guide to prepare for the NAC OSCE and MCCQE2. The content was reviewed for the most recent updates;

210

Case 10; Smoking counseling

Edited by; Dr. Sadik Salman

Door sign

David McLean, is a 33 years old male who comes in to your clinic because he has questions about

cigarette smoking.

Vital signs;

BP 120/80

PR 80

RR 12

Temperature 36.8⁰ C

In the next 10 min counsel him about smoking. At 9 min the examiner may ask you a question or

questions.

Note; the NAC may have a similar stem but the last 2 lines may look like this; In the next 11 minutes

counsel the patient about smoking. At 8 min the examiner may ask you a question or questions.

Sample notes to write for yourself

David McLean General Hx

33 M Hx of smoking

Stage of change

Hx and counseling How much does he know

Risks of smoking, benefits of quitting

What to do to quit, meds details

Patient encounter

Page 211: CANADIAN IMGS GUIDE TO OSCE AND PRACTICE · in an OSCE scenario You can use this study guide to prepare for the NAC OSCE and MCCQE2. The content was reviewed for the most recent updates;

211

Your actions Suggested verbalizing Patient response

Opening start How can I help you today? I want to talk to you about

smoking

And what exactly do you want

to discuss?

I’m thinking to quit and don’t

know where to start

Good for you, I can definitely

help you with this

Reflective questioning and

listening

What prompted you to think

about quitting?

Sorry about that, it must have

been hard for you

I understand, I admire your

determination, that’s a very

strong motivation and I will do

my best to help you.

Do you mind if I ask you few

questions about your health

first?

My friend is 40 years old only,

and he died with lung cancer.

It is, I decided I want to live for

my wife and daughter. I’m

determined I will not let

smoking get to me

Not at all

Current symptoms

Respiratory and cardiac

Do you cough?

Do you have chest pain?

Do you cough up blood?

Do you have shortness of

breath?

No

No

No never

No

Page 212: CANADIAN IMGS GUIDE TO OSCE AND PRACTICE · in an OSCE scenario You can use this study guide to prepare for the NAC OSCE and MCCQE2. The content was reviewed for the most recent updates;

212

GI

Constitutional

Other symptoms

Do you feel dizzy?

Do you have diarrhea or

constipation?

Abdominal pain?

Heart burn?

Blood in the stool or change in

stool color?

Have you lost weight recently?

Do you get fevers?

Do you get drenching night

sweats that you have to change

the sheets?

Do you feel fatigued?

Do you feel depressed?

Have you lost interest in

activities you used to enjoy?

Do you have headache?

Weakness?

Problems with urine?

Skin rash?

No

No

No

No

No

I gained 2 lb

No

No

No, I’m very active

I’m sad I lost my friend but I’m

not depressed

No

No

No

No

No

Comorbidities and other cardiac

and respiratory risk factors

Do you have other diseases like

diabetes or high cholesterol, or

asthma?

None of that

Page 213: CANADIAN IMGS GUIDE TO OSCE AND PRACTICE · in an OSCE scenario You can use this study guide to prepare for the NAC OSCE and MCCQE2. The content was reviewed for the most recent updates;

213

Were you exposed to asbestos

or other materials and

chemicals?

Never

Family history

Did anyone in your family have

cancer?

Heart or lung disease?

Sorry to hear that

Not that I know of

My father died of a heart attack

when he was 70

Medications and allergy

Do you take any medications,

over the counter medications or

herbs?

Do you have allergy to

medications or food?

I take Tylenol occasionally

Not that I know of

Other substance use

Alcohol

Recreational drugs

Do you drink alcohol?

How much is a few?

Have you tried to cut down your

drinking?

Do you feel guilty about

drinking?

Do you get annoyed by people

criticizing your drinking?

Do you need alcohol the first

thing in the morning as an eye

opener?

Do you use recreational drugs?

I drink few beers every day

Two.

No

No

No

No

No

Social history

Page 214: CANADIAN IMGS GUIDE TO OSCE AND PRACTICE · in an OSCE scenario You can use this study guide to prepare for the NAC OSCE and MCCQE2. The content was reviewed for the most recent updates;

214

Occupation

Living conditions

Current stressors

Exercise

Diet

What do you do for living?

With whom do you live?

Any stress at home or work?

Do you exercise?

Good for you

Do you eat healthy?

I’m a lawyer

My wife and daughter

No

I go to the gym 3 times a week

Well, not really I eat a lot of

junk food

History of Smoking

Number of packs per day

Duration

Location and situation where

smoking most

Who he smoke with

Previous investigations

How much do you smoke?

For how long?

Where and when do you smoke

most and who accompanies you

when you smoke?

Have you ever had a breathing

test or chest X-ray

A pack a day

For the last 15 years

I take multiple short breaks at

work to smoke, some of my

coworkers are smokers and we

enjoy a cigarette together. I

avoid smoking at home because

my wife doesn’t like it

No

Previous attempts to quit

Have you tried to quit before?

What did you do and for how

long did you quit?

I tried once

A girlfriend told me I’m

addicted and can’t stay away

from cigarettes so I stopped

smoking for 4 days, I didn’t

want to quit back then, just

wanted to prove a point

Page 215: CANADIAN IMGS GUIDE TO OSCE AND PRACTICE · in an OSCE scenario You can use this study guide to prepare for the NAC OSCE and MCCQE2. The content was reviewed for the most recent updates;

215

Additional questions to consider

when appropriate;

What symptoms did you get and

how did you deal with them?

Why did you bounce back?

How did you feel those 4 days?

I see, did you use anything to

help you stay away from it like

nicotine gums or patches?

Horrible! I just wanted a

cigarette, I was cranky and

couldn’t concentrate

No

Assess stage of change How ready do you feel you are

to quit this time?

So, you feel ready to take

actions to stop smoking

100%

I will do whatever it takes

Good for you, You are giving

your body the best gift.

Can I share some of the benefits

of quitting?

Smoking increases the risk of

lung cancer, heart disease and

chronic obstructive pulmonary

disease. Once you quit Oxygen

level will go back to normal in

less than 10 hours, your risk of a

heart attack will go down, your

lung function will improve and

your risk of lung cancer will

drop; In 10 years your risk of

lung cancer is cut in half!

There are even more benefits;

People will no longer be

affected by your smoking; you

will set a good example to your

daughter and save lots of

money. Sometimes writing

down all the positive things that

Sure

Page 216: CANADIAN IMGS GUIDE TO OSCE AND PRACTICE · in an OSCE scenario You can use this study guide to prepare for the NAC OSCE and MCCQE2. The content was reviewed for the most recent updates;

216

you are getting may help, how

does that sound?

Great, I’m even more

determined than before

Time of change When do you think you want to

stop?

I have a hectic work schedule

this week, I will start next

Monday. I would like to hear if

you have any suggestions that

may make quitting easier

Taking actions

Family and friends support

Support groups

Avoid situations that may

encourage smoking

So we have agreed on a date, I

would suggest you speak to

your wife, friends and

coworkers about your decision,

their support will help you along

the way, explain to your

coworkers that you will not be

smoking with them. And find

something to do during your

break like going to a walk, how

does that sound?

That’s excellent.

It might also be helpful to avoid

situations that might trigger the

urge to smoke.

You will have some symptoms

when you quit; cigarettes

contain nicotine, and your body

is addicted to it, so you may feel

dizzy, shaky. You may get

irritable and feel more tired

than usual. Don’t push yourself,

nap if you need to and avoid

Sounds good, that’s a good

idea, I will join the other group

of coworkers, they eat and go

for short walks during their

breaks

Page 217: CANADIAN IMGS GUIDE TO OSCE AND PRACTICE · in an OSCE scenario You can use this study guide to prepare for the NAC OSCE and MCCQE2. The content was reviewed for the most recent updates;

217

Nicotine withdrawal and how to

cope

extra work during this period.

Your sleep may get disturbed,

avoid caffeinated tea and

coffee. You may cough more

and feel tight the first few

weeks as your lungs try to clear

up tar and other toxins, drink

lots of water and take deep

breaths. If you get chest pain,

cough up blood, have a fever or

your cough is getting worse

rather than better then you

should seek medical help.

You may feel hungry, and a lot

of people gain weight when

they quit. Try to eat healthy

food, and snacks

Do you have any questions?

That’s very helpful, thank you

I’m not worried about my

weight

Are there drugs to help these

symptom?

Details about pharmacotherapy That’s an excellent question

A lot of these symptoms are

due to the fact that your body

craves nicotine. There are

different forms of nicotine

replacement therapy like; gums,

patches, lozenges, inhalers and

nasal spray. I usually prescribe

the patch combined with the

gum to use when urge arises.

Side effects are minimal

including skin irritation,

insomnia and bad dreams; both

can be avoided by removing the

patch at night and rotating the

patch site.

Page 218: CANADIAN IMGS GUIDE TO OSCE AND PRACTICE · in an OSCE scenario You can use this study guide to prepare for the NAC OSCE and MCCQE2. The content was reviewed for the most recent updates;

218

I will give you handouts to read

more. Any questions?

There are 2 drugs that are

effective in helping smokers

quit;

Bupropion (Zyban) and

Varenicline (Champix). Both are

pills

Zyban may cause headache,

insomnia, agitation, and seizure

Champix is probably more

effective than Zyban, it may

cause nausea, skin rash, visual

disturbance and impairment of

motor abilities with increased

risk of accidents and falls. It also

increases the risk of heart

attacks in already high risk

patients and you are not one of

them.

Both these drugs can increase

the risk of suicide.

I will give you some handouts to

read and think about your

choices, if you decide to go with

Champix then we need to get

started one week before you

quit. Questions?

I think I will take nicotine. Are

there other drugs?

Thank you for providing all this

information, I will read the

handouts and see if I want to

use drugs

Dealing with cravings Sure, let me know what you

think or if you have questions.

Now you will crave cigarettes,

strong craving lasts usually for

few minutes then goes away.

Try to engage yourself with

something, like a walk or a drink

Page 219: CANADIAN IMGS GUIDE TO OSCE AND PRACTICE · in an OSCE scenario You can use this study guide to prepare for the NAC OSCE and MCCQE2. The content was reviewed for the most recent updates;

219

of water. You can chew nicotine

gums as well

If you usually smoke with your

beer this may be challenging.

Be aware of this.

Encouraging statements I congratulate you for your

decision, you are on the right

track for optimal health

Support systems available

FU at clinic- visits, phones, texts,

emails, etc…

I or one of our physicians will

always be here for you. We can

arrange for follow up visits

Sure, we can do that. My clinic

can also send you encouraging

emails and text messages if you

like

I will give you the hotline

number for smoking cessation

as well as contact information

of support groups

Sounds good

Sure, emails would be great

I appreciate that , thank you

Falling off the wagon I also want to mention that you

may fall off the wagon, and

that’s ok, the key is to try again

I’m very determined, I want to

stop

Patient’s feelings, questions,

concerns and expectations

Do you have any questions or

concerns?

Not at the moment

I admire your determination,

and look forward to hear you

Sure I will, thank you

Page 220: CANADIAN IMGS GUIDE TO OSCE AND PRACTICE · in an OSCE scenario You can use this study guide to prepare for the NAC OSCE and MCCQE2. The content was reviewed for the most recent updates;

220

success story, please contact

me if you have any questions

P.E is not required in this case

Sample questions you may be asked by the examiner

1- If the patient decides to go with Zyban or Champix, how would you manage the increased

suicide risk?

Answer; I will explain to the patient that this is a possible side effect and educate him/her about

depression symptoms. I will ask the patient to stop the drug and call me immediately if he/she had

thoughts of suicide or depression symptoms.

Rating scales points;

- Examinee introduced self and position

- Addressed patient with name

- Used proper non-verbal communication

- Good organizational skills

- Examinee spoke clearly (accent didn’t get in the way)

- Listening and questioning skills

- Showed rapport with patient

- Explored patient concerns

- Attentive to patient physical comfort

- Medical knowledge adequate

- No misinformation was provided to patients

- No concerning ethical/legal issues

- The examinee is respectful of other health care members

Variations of the case and some tips;

1- It is essential to give a personalized advice in cases of counseling. Make sure you take relevant

history focusing on social aspects, and substance use. Explore the patient’s needs and concerns

and help them regardless of the stage of change they are at. If you encounter a contemplating

or pre contemplating patient, educate him/ her but don’t push for a change. Respecting the

stage your patient is at will build trust and make it easier for your patient to contact you should

he/ she needs.

2- I strongly encourage you to familiarize yourself with the 5 As of 3-5 min tobacco intervention;

Page 221: CANADIAN IMGS GUIDE TO OSCE AND PRACTICE · in an OSCE scenario You can use this study guide to prepare for the NAC OSCE and MCCQE2. The content was reviewed for the most recent updates;

221

http://www.sdta.ca/mrws/filedriver/DentistTobaccoInterventionAlgorithmSept06.pdf

Page 222: CANADIAN IMGS GUIDE TO OSCE AND PRACTICE · in an OSCE scenario You can use this study guide to prepare for the NAC OSCE and MCCQE2. The content was reviewed for the most recent updates;

222

Case 11; Preeclampsia (PET)

Edited by; Dr. Erica Paras

Door sign

Linda Robinson, 23 years old female. 32 weeks pregnant. Was diagnosed with preeclampsia by her

family doctor and referred to the obstetrician. You are the resident working in the clinic

Vital signs;

BP 145/95

HR 88

RR 12

Temperature 37⁰C

In the next 10 min take a focused and relevant history. At 9 min the examiner may ask you a question or

questions.

Note; the NAC exam may have a similar format but the last 2 lines will look like this; In the next 11

minutes take a focused and relevant history. At 8 min the examiner may ask you a question or questions.

Sample notes to write for yourself:

Linda Robinson how was it diagnosed

23 F Risk factors

PET history Severe PET

Complications

Treatment?

Obs/gyne and general Hx

Patient encounter

Your actions Suggested verbalizing Patient response

Page 223: CANADIAN IMGS GUIDE TO OSCE AND PRACTICE · in an OSCE scenario You can use this study guide to prepare for the NAC OSCE and MCCQE2. The content was reviewed for the most recent updates;

223

Opening start How can I help you today? My doctor referred me because

my blood pressure was high

Details about the presenting

complaint

When was it diagnosed?

How high was Your blood

pressure?

Where you well rested and

relaxed when it was measured?

Have you ever had a high blood

pressure before?

Did your doctor check protein in

your urine?

Did he prescribe you any

medication?

Last Tuesday

145/95

Yes

No. this is the first time someone

tell me my blood pressure is high

Yes, he said it’s +1

No

Symptoms of severe PET

Eclampsia?

Do you have headache?

Did you have any change to your

vision like double vision and loss

of vision?

Do you have breathing difficulty?

Do you have chest pain?

Do you have abdominal pain?

Epigastric pain?

Have you had abnormal seizure

like movements?

No

No

No

No

No

No

No

Fetal movement How is the baby moving? She is very active

History of current pregnancy

Page 224: CANADIAN IMGS GUIDE TO OSCE AND PRACTICE · in an OSCE scenario You can use this study guide to prepare for the NAC OSCE and MCCQE2. The content was reviewed for the most recent updates;

224

GTPAL

Last menstrual period

Nausea/vomiting?

Other complications

Vitamins/folic acid/iron

X-ray or medications exposure?

Medical follow up/US

Is this your first pregnancy? Have

you ever had abortions or

pregnancy losses?

When was your last menstrual

period?

Was it your typical period? Are

you sure about the dates?

So your expected date is

February 17 and you are 32

weeks pregnant

How was your date calculated-

by your LMP or early ultrasound?

Do you have nausea and

vomiting?

Have you had any other

complications during this

pregnancy like Infections?

Bleeding? Trauma? Diabetes?

Hospitalizations?

Do you take multivitamins?

How about iron and folic acid?

Did you take folic acid pre-

conception? Or when you found

out about the pregnancy?

How far along were you when

you found out about the

pregnancy?

Were you exposed to X ray?

Did you take medications during

pregnancy?

This is my first

No, never

May 10, Yes

That’s right

By LMP

I had some at the beginning of

pregnancy but not anymore

No, it’s been smooth so far

No

No

I took folic acid

When I found out

About a week after I missed my

period

No

No

No

Page 225: CANADIAN IMGS GUIDE TO OSCE AND PRACTICE · in an OSCE scenario You can use this study guide to prepare for the NAC OSCE and MCCQE2. The content was reviewed for the most recent updates;

225

Blood group

Any exposure to over-the-

counter medications? Street

drugs? Alcohol?

So your family doctor followed

you up so far, correct?

Have you had an Ultrasound

done?

Do you know if it’s one or more

babies?

Was there anything outstanding

on the US?

What is your blood group?

Yes, every month

Yes, When I was 19 weeks

pregnant

It’s one girl

My doctor said everything looks

good

A+

Relevant review of systems

Vaginal bleeding

Vaginal discharge

Constipation/diarrhea

Acid reflux

Jaundice

Stool color

Easy bruising

Dysuria

Change of urine color or amount

Hands/face swelling

Legs swelling

Pre-pregnancy weight

Do you have vaginal bleeding?

Do you have vaginal discharge?

Are you constipated? Do you

have diarrhea?

Do you have heart burn?

Did you notice your eyes and

skin are getting yellow?

Did you notice a change to your

stool color?

Do you bruise easily?

Does it hurt to pee?

Did you notice a change to urine

color or amount?

No

No

A little constipated but not too

bad

The usual, I’m used to it by now

No

No

No

No

No

Page 226: CANADIAN IMGS GUIDE TO OSCE AND PRACTICE · in an OSCE scenario You can use this study guide to prepare for the NAC OSCE and MCCQE2. The content was reviewed for the most recent updates;

226

Headaches?

Abnormal movements?

Behaviours?

Did your face or hands swell?

How about your legs?

What was your weight before

pregnancy?

My hands are puffy

They swell if I stand for too long,

but then go down

56 Kg

Past medical and surgical history

Have you ever been diagnosed

with any disease like problems

with your Heart? Lungs?

Thyroid? Kidneys? Bowels? Etc.

Have you ever had surgery?

Any hospitalizations?

Did you have any problems with

anesthesia?

Did you have any complications

after surgery?

No

I had my appendix taken out 2

years ago

No

No

No

Medications and allergy

Do you take any medications?

Other than vitamins and folic

acid, do you take any other over

the counter medications?

Do you take Herbs?

Do you have allergy to

medications or food?

No

No

No

No

Family history

Did anyone in your family have

twins? Congenital

malformations? Recurrent

No

Page 227: CANADIAN IMGS GUIDE TO OSCE AND PRACTICE · in an OSCE scenario You can use this study guide to prepare for the NAC OSCE and MCCQE2. The content was reviewed for the most recent updates;

227

pregnancy losses? Severe mental

retardation?

Does anyone in your family have

breast, ovarian or colon cancer?

Does anyone in your family have

high blood pressure?

How about high blood pressure

during pregnancy?

No

No

My father does

Not that I know of

Relevant gynecological history

Previous STDs

Pap test

Have you ever had a sexually

transmitted infection like

Syphilis?

When was your last pap test?

Was it normal?

Have you ever had an abnormal

pap test?

No

2 months before I got pregnant

Yes

When I was 20 years old there

was a problem with my test but

my doctor repeated it after 6

months and it was ok

Social history

Smoking

Alcohol

Drugs

Work

Home

Social support

Do you smoke?

Do you drink alcohol?

Do you use recreational drugs?

What do you do for living?

With whom do you live?

No

Not during pregnancy

No

I’m an art student

With my husband

Page 228: CANADIAN IMGS GUIDE TO OSCE AND PRACTICE · in an OSCE scenario You can use this study guide to prepare for the NAC OSCE and MCCQE2. The content was reviewed for the most recent updates;

228

Always screen for Abuse-

pregnancy is a very common

time for abuse

Besides your husband, do you

have any other support?

It is exciting for sure

Have you ever felt unsafe?

Yes, both my parents and in laws

are in town and everyone is

excited about the baby

No

FIFE

Important to say that high BP in

pregnancy is a concern to mom

and baby- we need to the treat

the mom and keep her safe- if

she is not safe the baby can be

affected- G-HTN is serious.

What are your thoughts about

your high blood pressure?

Some babies are affected; they

may be smaller or born earlier

than their time. I’m going to

start you on a medication to

lower your blood pressure, and

follow up with you and your

baby more closely. How does

that sound?

Do you have any other questions

or concerns?

I’m worried about my baby, is he

going to be ok?

Much better

Nothing right now, thank you

No P.E is required

If asked to do a P.E, look for;

- General; Notice any tremor, abnormal movements or positioning and bruising. Check the vitals –

Both arms, while seated- ensuring the mom is relaxed- double check all the BP's- ensure it is not just

white coat hypertension.

- Hands, head and neck; Look at the hands, notice any pallor, skin or nail changes, swelling. Feel the

pulse and compare both sides. Look at the face for pallor or jaundice, notice any bruising. Look into

the mouth, and do fundoscopic examination.

- Auscultate the heart and lungs

- Examine the abdomen; Do inspection and general gentle palpation then do Leopold maneuver to

check fetal position. At earlier stages of pregnancy you can do full palpation. Check fundal height.

Palapate Epigastric area and RUQ for any pain on palpation

Page 229: CANADIAN IMGS GUIDE TO OSCE AND PRACTICE · in an OSCE scenario You can use this study guide to prepare for the NAC OSCE and MCCQE2. The content was reviewed for the most recent updates;

229

- Indicate that you would check fetal heart rate using a Doppler

- Examine the reflexes

- Inspect the legs and palpate the pulses, check for pitting edema

- Indicate that you would do a vaginal examination

Sample questions you may be asked by the examiner

1- What medication would you prescribe to control this patient’s blood pressure?

Answer;

- Labetolol 100-600 mg PO BID-TID, max 1200 mg/day

- Adalat XL 20-60 mg PO daily-BID, max 120 mg/day

- Methyldopa 250-500 mg PO BID-QID, max of 3000 mg/day

- can add Thiazide if inadequately controlled

2- What is the management of Preeclampsia?

Answer;

– If severe, deliver regardless of gestational age

– less than 34 weeks, an adverse symptom, proteinuria >5 g/day; deliver

– adverse symptom: BP > 160/110, Platelets < 100, LFT x 2 increase, Cr x 2 increase,

pulmonary edema, headache/change in vision

For Mild PET

– deliver at 37 weeks

– 34-37 weeks- can do expectant management

– <34 weeks- do daily kick counts, PIH labs 1-2x /week, NST/BPP with Dopplers 1-2x/week,

q3 weeks fetal growth measurements and AFI

1- What investigations would you order;

Answer; To evaluate the mother; CBCD, creatinine, urea, lytes, AST, ALT, Albumin, Bilirubin, INR, PTT,

LDH, -Protein/Creatinine Ratio. Urine Analysis, R/M, C/S

Page 230: CANADIAN IMGS GUIDE TO OSCE AND PRACTICE · in an OSCE scenario You can use this study guide to prepare for the NAC OSCE and MCCQE2. The content was reviewed for the most recent updates;

230

-Fibrinogen- it should be elevated in pregnancy, so a “normal” Fibrinogen is too low, and is a worrisome

sign,

-LDH- sign of hemolysis

-Urate- is typically the first abnormal lab. However once it is elevated we don't typically track it, as

further elevation doesn't mean a worsening PET

To evaluate the fetus; Biophysical profile or non-stress test

-Particularly fetal growth, is abdominal circumference< head circumference = brain sparing, AFI,

Umbilical cord Dopplers – changes in the end-diastolic flow patterns

Rating scales points;

- Examinee introduced self and position

- Addressed patient with name

- Used proper non-verbal communication

- Good organizational skills

- Examinee spoke clearly (accent didn’t get in the way)

- Listening and questioning skills

- Showed rapport with patient

- Explored patient concerns

- Attentive to patient physical comfort

- Medical knowledge adequate

- No misinformation was provided to patients

- No concerning ethical/legal issues

- The examinee is respectful of other health care members

Variations of the case and some tips;

1- Case; 28 year old female 36 weeks pregnant presenting with BP 190/110. Manage

Brief history; GTPAL; Primigravida 36 weeks, symptoms of severe PET; none. Complications during

pregnancy; Preeclampsia diagnosed at 34 weeks. Medications and allergies; Methyldopa 250mg BID. No

allergies. Last meal; 6 hours ago. Fetal movement; ok.

Relevant P.E; Auscultate heart and lungs, Assess reflexes, Fundoscopy, inspect and palpate the

abdomen, do Leopold maneuver, vaginal exam; examiner reports; closed cervix.

Page 231: CANADIAN IMGS GUIDE TO OSCE AND PRACTICE · in an OSCE scenario You can use this study guide to prepare for the NAC OSCE and MCCQE2. The content was reviewed for the most recent updates;

231

Management;

1. NPO

2. Foley catheter- strict monitoring of urine output

3. Call if BP > 160/105- above that stroke risk increases, hold meds if BP <130/80- fetus is

dependent on maternal blood flow, if she is hypo-tensive so will the baby

4. Continuous fetal HR monitoring

5. IVF: Total fluid intake of 100 cc/hr RL – do not want to fluid overload them and put

them into pulmonary edema, as the patient is third-spacing when they have PET

6. To prevent seizures; MgSO₄ 4g IV loading dose over 20 min then 1g /h

continue until at least 24 hours post-partum or BP decreases post-partum and the

patient is diuresing

7. Acute treatment of BP:

a. Labetolol 10-20 mg IV, then double dose q 10 minutes, max 300 mg, or can do

infusion at 0.5-2 mg/min or

b. Hydralazine- will acutely decrease BP, 5 mg IV, then can use 5-10 mg q 20-30

minutes, max 20 mg or

c. Nifedipine- 5 mg PO q 30 mins or

8. See above for investigations to order

Prepare for delivery; if maternal and fetal status stable induction of labor, if unstable emergency C/S

Discussion; One thing to add is to give corticosteroids for fetal lungs maturity if the patient was < 34

weeks pregnant. You can give Betamethasone 12 mg IM q 12 hours total of 2 doses.

- The steroids typically cause a “moon-lighting phase”- post steroids the patients lab abnormalities may

improve and her BP may improve- this is temporary- don't let it fool you, PET patients are sick and the

“moon lighting” shouldn't change your management plan overall

- <32 weeks MgSO4 is also given for Neuro-protection- this pre-eclamptic lady will be getting already.

We can mention to the patient that the MgSO4 will prevent seizures for her and decrease neonatal

death, cerebral palsy and gross motor-dysfuntion

Page 232: CANADIAN IMGS GUIDE TO OSCE AND PRACTICE · in an OSCE scenario You can use this study guide to prepare for the NAC OSCE and MCCQE2. The content was reviewed for the most recent updates;

232

2- Case; 30 years old female, just diagnosed with preeclampsia with BP of 140/90 and +2 protein

on urine dipstick. Counsel

History; The patient is a mother of 2 years old, no abortions or still births. She had a smooth pregnancy

and is 32 weeks pregnant. Not known to have hypertension before, no symptoms of severe PET, and

review of system was unremarkable. She had preeclampsia during her first pregnancy and her baby was

delivered with C section due to fetal distress during labor. Fetal weight was 7 lb and her baby didn’t

need NICU and did well after. Not on medications, and doesn’t have allergies. Doesn’t smoke, drink or

do drugs. She is a stay home mum, and enjoys lots of support from her husband and family.

Counseling; Explore what she already knows and what info she is seeking. See if she has particular

concerns or fears. Talk to her about PET; it’s definition and complications, importance of regular BP

monitoring and more rigorous follow up of herself and the baby. Talk to her about management of

chronic and severe PET. The patient was concerned about the mode of delivery and wanted to avoid C

section if at all possible. Assure her that you will document her wish and honor it, and explain to her that

C section might be needed to save her own or her baby’s lives. Then mention the indications and

complications of C section. Go over the symptoms of severe PET and make sure the patient understands

when to seek help.

3- I recommend that you go over the Canadian guidelines for hypertensive disorders during

pregnancy;

http://sogc.org/guidelines/diagnosis-evaluation-and-management-of-the-hypertensive-disorders-of-

pregnancy/

Page 233: CANADIAN IMGS GUIDE TO OSCE AND PRACTICE · in an OSCE scenario You can use this study guide to prepare for the NAC OSCE and MCCQE2. The content was reviewed for the most recent updates;

233

Case 12; Contraception

Edited by; Dr. Erica Paras

Door sign

Madeline Carter, a 31 years old female. Comes in to your clinic to talk about contraception.

Vital signs;

BP 120/75

PR 82

RR 12

Temperature 36.8

In the next 10 min counsel the patient about contraception. At 9 min the examiner may ask you a

question or questions.

Note; the NAC may have a similar stem, but the last 2 lines may look like this; In the next 11 min Counsel

the patient about contraception, at 8 min the examiner may ask you a question or questions.

Sample notes to write for yourself

Madeline Carter Obs/Gyne Hx

31F General Hx

FIFE

Contraception contraception types, efficacy, prices, S/E,

Hx, counsel

Patient encounter

Page 234: CANADIAN IMGS GUIDE TO OSCE AND PRACTICE · in an OSCE scenario You can use this study guide to prepare for the NAC OSCE and MCCQE2. The content was reviewed for the most recent updates;

234

Your actions Suggested verbalizing Patient response

Opening start How can I help you today? I need to know about

contraception methods out

there other than the pills

Sure, do you have particular

questions in mind?

I had a C section and I’m breast

feeding, I want something safe

for me and my baby

Congratulations, I can talk to you

about contraception methods

and answer your questions as we

go. I need however to ask you

few questions about your health

first

Sure, no problem

Obstetric history So, was this your first

pregnancy? No miscarriages?

When was your baby born?

So the baby is 6 weeks now?

How many weeks were you

pregnant when you gave birth?

Why did you have a C section?

I see, Did your baby need NICU?

Do you know what was the

APGAR score?

That’s not too bad, did you have

a boy or a girl?

Is he healthy?

What was his birth weight?

Yes

May 20

Yes

39 weeks

The baby heart rate was going

slow, apparently the cord was

folded around his neck

No

It was 1, 7 and 9

A boy

Yes

7.3 Lbs

Page 235: CANADIAN IMGS GUIDE TO OSCE AND PRACTICE · in an OSCE scenario You can use this study guide to prepare for the NAC OSCE and MCCQE2. The content was reviewed for the most recent updates;

235

Did you have any problems

during pregnancy like bleeding,

infections, clots?

Did you have any problems after

delivery?

Are you still having vaginal

bleeding?

Have you had sex after your

baby was born?

And how is breast feeding going?

Any soreness or nipple

discharge?

Do you take vitamins?

Do you take folic acid, iron or

other supplements?

No, it was very smooth

No

Very minimal

Not yet

It’s going very well, no problems

I still take my prenatal vitamins

No

Pertinent symptoms Do you have abdominal pain?

How is the wound healing?

Is there any pain or discharge?

That’s good to hear

Do you have vaginal discharge or

itching?

Does it hurt to pee?

Have you noticed any change in

urine color or smell?

Do you have constipation or

diarrhea?

Any problems with

hemorrhoids?

No

Much better than I thought

I don’t feel it any more

No

No

No

No

No. I’m pretty regular

No

Page 236: CANADIAN IMGS GUIDE TO OSCE AND PRACTICE · in an OSCE scenario You can use this study guide to prepare for the NAC OSCE and MCCQE2. The content was reviewed for the most recent updates;

236

Relevant Gyne history

Detailed Contraception history

Have you ever had a sexually

transmitted infection before?

Have you had ovarian or uterine

surgery?

When was your last pap test?

Have you ever had abnormal

tests?

At what age you had your first

period?

Are your periods regular? How

often

What contraception did you use

before?

The combination pills of

estrogen and progesterone can

do that. But there are other pills

made only of progesterone that

wouldn’t do that.

It must be tiring. There are other

methods out there that I will

come to in a minute, I need to

ask you few more questions

No

No

Just before I got pregnant, it was

normal

No

14

Yes, every 30 days

I’ve been always on the pill, but

the nurse said it may cause my

milk to go down

Really, that’s convenient. I’m

worried that I may forget to take

the pill because my sleep is all

over the place after the baby is

born.

Sure

Past medical and surgical history Do you have any disease like

diabetes, heart disease or blood

clots?

Any problems with your heart

valves? Migraine headaches?

History of breast/gynecologic

cancers? Liver disease?

No

No

Page 237: CANADIAN IMGS GUIDE TO OSCE AND PRACTICE · in an OSCE scenario You can use this study guide to prepare for the NAC OSCE and MCCQE2. The content was reviewed for the most recent updates;

237

Have you ever had surgery

before?

No

Medications and allergy Do you take any medications?

How about over the counter

medications or herbs?

Do you have allergy to

medications or food?

No

No

No

Family history Did anyone in your family have

ovarian, uterine or colon cancer?

Did anyone have blood clots?

How about stroke and other

diseases?

No

No

My grandfather died of stroke

when he was 87 years old, my

father has high blood pressure

but he is healthy otherwise

Social history Do you smoke?

For how long did you smoke?

Do you drink alcohol?

Do you use recreational drugs?

Who helps you with the baby?

Are you coping well?

Yes, I enjoy 10 cigarettes every

day

10 years

Occasionally

I never did

My husband helps a lot, and our

parents are both in town and

they have been a great help

Yes, I love my son very much

But I do feel tired

Page 238: CANADIAN IMGS GUIDE TO OSCE AND PRACTICE · in an OSCE scenario You can use this study guide to prepare for the NAC OSCE and MCCQE2. The content was reviewed for the most recent updates;

238

Do you feel depressed?

When are going back to work?

No, I’m just tired

I’m taking a whole year off

Counseling part

Summary of relevant history

Importance of effective

contraception after C section

Lactation effectiveness as a

contraception

Information about contraception

So far I know that you are 31

years old, a new mom of 6 weeks

old baby. Had a good pregnancy

and delivered at 39 weeks with a

C section due to cord around the

neck. You are healthy, smoke 10

cigarettes a day. You used the

pills before but would like to

hear about other options.

No problems. Now because you

had a C section, it’s advisable to

use an effective method of

contraception for 18 months if

you would want to have a

vaginal delivery with your next

pregnancy

-Women can begin to ovulate

45-95 days after giving birth, this

is delayed by lactation

-A woman is only considered

anovulatory if she has : < 6

months after giving birth,

exclusive breast feeding and has

amenorrhea- if so the protection

is ~98%

-If don't meet the above criteria,

very important for a women to

be using contraception

That’s correct

I see

Page 239: CANADIAN IMGS GUIDE TO OSCE AND PRACTICE · in an OSCE scenario You can use this study guide to prepare for the NAC OSCE and MCCQE2. The content was reviewed for the most recent updates;

239

(you need to go over types,

mode of administration, cost,

effectiveness, and major benefits

and side effect)

Note that in this case no details

of the combined pill are

provided because the patient

will not take them, they are very

common form of contraception

and if suitable for the patient

then you need to go over

benefits and side effects

Keep in mind the

contraindications as you gather

history and counsel your

patients – If the patient has a

contraindication for a certain

method, you don’t need to go

into details about it because you

don’t want to overwhelm your

patient

There are 2 main groups of

contraception; Hormonal and

non-hormonal.

The hormonal method includes

the 2 types of pills we discussed

before. The mini pill is 99.7%

effective if used perfectly.

It has to be taken at exactly the

same time each day. If you miss

the pill by > 3 hours you need

back-up contraception for at

least 2 days. It is very sensitive to

changes in timing of the pill- not

good if the person is forgetful.

Have you thought about setting

an alarm or reminder system?

They may cause irregular

spotting, headache, bloating,

acne and breast tenderness.

Effectiveness may be decreased

by other medications.

The patch is another hormonal

method; It is applied to the skin

for 3 weeks and removed for a

week where menses occurs

It is a systemic estrogen, just like

the combo pill, therefore, in

theory it can affect milk

production

Do I have to take it everyday?

No, but it sounds like a good

idea. What are the side effects?

Page 240: CANADIAN IMGS GUIDE TO OSCE AND PRACTICE · in an OSCE scenario You can use this study guide to prepare for the NAC OSCE and MCCQE2. The content was reviewed for the most recent updates;

240

There is Estrogen vaginal ring. It

is also a combo, but less

systemic side effects and

therefore may not affect milk

production in the same way

There is also the shot, it is only

progesterone and you need to

get an injection every 3 months,

it’s 99.7% effective. Periods will

stop but there will be irregular

spotting. Weight gain is another

concern for some women. If you

want to get pregnant again after

the shot, it may take 9 months

for the ovaries to start ovulating

again.

I don’t like that one

I see

What other options are there?

And there is the intrauterine

device, one type is hormonal and

contains only progesterone, it’s

called Mirena. It’s 99% effective.

Normal Mirena is still used by

most for 5 yrs, and the low dose

one for 3 yrs.

IUD has to be inserted and

removed by a doctor, may cause

irregular bleeding in the first 6

months and may increase

cramps and bleeding. There is a

small chance that it may fall off.

How do you feel about it

- Expulsion is increased if

inserted < 6 weeks PP, if there

It sounds like a good option

Page 241: CANADIAN IMGS GUIDE TO OSCE AND PRACTICE · in an OSCE scenario You can use this study guide to prepare for the NAC OSCE and MCCQE2. The content was reviewed for the most recent updates;

241

was a C-section and if the px is

breastfeeding- but overall the

risk is low

It’s copper and is non hormonal.

It’s 99% effective and can last for

5 years. Has very similar S/E to

the hormonal intrauterine device

- Copper device more women

have heavier periods, vs. Mirena

where 80% of women have

amenorrhea after the first year.

Do you have questions?

Other contraception methods

that may work for you are non-

hormonal and can be used with

or without a spermicide. They

include barrier methods that can

be used by females like female

Condoms, cervical caps and

diaphragms. None of them

protects against sexually

transmitted infection. The

female condom is the most

affective ranging between 80-

95%. How do you feel about

them?

I understand, this goes for male

condoms too, right?

I see

No, that’s clear

I haven’t tried them before, I

don’t like to worry about

condoms when we are about to

have sex, I want something more

convenient

Page 242: CANADIAN IMGS GUIDE TO OSCE AND PRACTICE · in an OSCE scenario You can use this study guide to prepare for the NAC OSCE and MCCQE2. The content was reviewed for the most recent updates;

242

The last methods are the

natural; like the calendar,

withdrawal and symptom-

thermal control, all but

withdrawal are hard to use in

your case because it’s hard to

predict ovulation when you are

breast feeding, and you want

something more effective at this

stage,

-timing and withdrawal have up

to 24% unintended pregnancies

over one year

Copper IUD cost around 150, and

the hormonal IUD around 350,

some insurance companies do

cover them

The shot is around 35-40 $ but

you need one every 3 months

I will give you a flyer about all

the contraception methods to

read through

Do you have any other

questions?

I can see you after 1 week. If you

think of any questions write

them down for when we meet

I want something more

effective. This doesn’t work for

me

I’m thinking to go with the IUD,

it seems convenient and I don’t

have to worry about it for years.

How much does it cost?

It’s pricy! I don’t have insurance.

What about the Shot?

Sounds excellent, I need some

time to weigh my options.

Not at the moment

I will, thank you

Page 243: CANADIAN IMGS GUIDE TO OSCE AND PRACTICE · in an OSCE scenario You can use this study guide to prepare for the NAC OSCE and MCCQE2. The content was reviewed for the most recent updates;

243

You’re welcome!

Physical exam;

No physical exam is required in this case

Sample questions you may be asked by the examiner;

1- Do you have a concern about the pill if the patient decides to use it after weaning her baby?

Answer; the combined estrogen and progesterone pills are not recommended in women over 35 years

who smoke because of increased risk of blood clots. Absolute contra-indication if > 35 yrs and smoke

>15 cig/day, Relative contra-indication if smoke <15 cig/day

2- How does the copper intrauterine device work?

Answer; impedes sperm transport and fertilization, prevents implantation by producing a foreign body

reaction and chemical changes in the endometrial lining.

3- What contraception methods protect against HIV?

Answer; Male condom

Rating scales points;1. Examinee introduced self and position

2- Addressed patient with name

3- Used proper non-verbal communication

4- Good organizational skills

5- Examinee spoke clearly (accent didn’t get in the way)

6- Listening and questioning skills

7- Showed rapport with patient

8- Explored patient concerns

Page 244: CANADIAN IMGS GUIDE TO OSCE AND PRACTICE · in an OSCE scenario You can use this study guide to prepare for the NAC OSCE and MCCQE2. The content was reviewed for the most recent updates;

244

9- Attentive to patient, allows time for questions

10- Medical knowledge adequate

11- No misinformation was provided to patients

12- No concerning ethical/legal issues

13- The examinee is respectful of other health care members

Variations of the case and some tips;

1- Please be reminded that you need to take relevant history when asked to counsel a

patient, so that counseling is tailored to patient’s needs, and pertinent health issues are

covered

2- In Canada, you can counsel teenagers of both sexes about contraception. In this age

group it is important to educate them about sexually transmitted infections and how to

protect themselves. If the teenager doesn’t want his/her parents involved then you

must maintain confidentiality. Teenagers don’t need consent from their parents for a

contraception prescription. Always do HEADSS screen for this age group (refer to

chapter 2 for more details). Ask your teenage patient the age of her partner to rule out

legal issues. No matter the age of her partner ensure sex is consensual.

3- In cases of counseling, especially when too much medical information is provided, offer

to give your patient handouts to read through

4- SOGC Contraception guidelines @ sogc.org , and the website www.sexualityandu.ca are

great resources

Page 245: CANADIAN IMGS GUIDE TO OSCE AND PRACTICE · in an OSCE scenario You can use this study guide to prepare for the NAC OSCE and MCCQE2. The content was reviewed for the most recent updates;

245

Case 13; Postmenopausal bleeding

Edited by; Dr. Erica Paras

Door sign

Margaret Finlay, is a 52 years old female presenting with vaginal bleeding.

Vital signs;

BP 120/70

HR 80

RR 12

Temperature 37 ⁰C

In the next 10 min conduct a focused and relevant history and perform a focused and relevant physical

exam. At 9 min the examiner may ask you a question or questions.

Note; the NAC may have a similar stem but the last 2 lines may look like this; In the next 11 minutes

conduct a focused and relevant history and perform a focused and relevant physical exam. At 8 min the

examiner may ask you a question or questions.

Sample notes to write for your self

Margaret Finley Atrophy

52 F Cancer

Endometrial hyperplasia

Hx and P.E Fibroid

Non-gyne cause

Patient encounter

Page 246: CANADIAN IMGS GUIDE TO OSCE AND PRACTICE · in an OSCE scenario You can use this study guide to prepare for the NAC OSCE and MCCQE2. The content was reviewed for the most recent updates;

246

Your actions Suggested verbalizing Patient response

Opening start How can I help you today?

Tell me more about it

I understand your concern, we

will find out why you have it.

Do you mind if I ask you few

questions?

I have vaginal bleeding

I have vaginal spotting, I’m kind

of worried, women my age

shouldn’t have it

Not at all

Details about vaginal bleeding

Onset, duration

Amount

Frequency

Precipitating factors

Pain and bleeding after

intercourse

Date of last menstrual cycle

Use of HRT or vaginal creams

When did the bleeding start?

How many pads do you use per

day? Amount of blood on the

pads?

How often do you get it?

Was there a precipitant for the

first bleed like a

trauma/intercourse/infection/

douching?

Are you sexually active?

Do you feel pain during sex?

At penetration or deep inside?

Do you have Bleeding after

sex?

When was your last menstrual

cycle?

About 5 months ago

May be 2-3, It’s just spotting

not much

Almost every day

No

Yes, with my husband

Yes, I have mild pain lately

Deep inside

No

A year ago

Page 247: CANADIAN IMGS GUIDE TO OSCE AND PRACTICE · in an OSCE scenario You can use this study guide to prepare for the NAC OSCE and MCCQE2. The content was reviewed for the most recent updates;

247

Previous episodes

Vaginal discharge

Vaginal itching

Pelvic/abdominal pain

Do you take hormone

replacement therapy, Orally or

vaginally?

Do you use vaginal creams?

Lubricants?

Did you have abnormal vaginal

bleeding before?

Do you have vaginal discharge?

Do you have vaginal itching?

Did you notice an increase in

the number of yeast infections?

Did you notice an increase in

the number of urinary tract

infections? How many in a

year?

Do you have pain in your

pelvis? Pressure? Bloating?

Do you have abdominal pain?

Early Satiety?

No

No

No

No, I feel dry

Sometimes

No, I get 2 each year

No I get it once or twice every

year

No

No

Constitutional symptoms Have you lost weight recently?

Do you get fevers?

Do you get drenching night

sweats?

No

No

No

GI/urinary symptoms

Bleeding diathesis

Does it hurt to pee?

Is there a change in urine color

or blood in urine?

Do you have constipation or

diarrhea?

Is there a change in stool color

or blood with stool?

No

No

No

No

Page 248: CANADIAN IMGS GUIDE TO OSCE AND PRACTICE · in an OSCE scenario You can use this study guide to prepare for the NAC OSCE and MCCQE2. The content was reviewed for the most recent updates;

248

Do you bruise easily?

Does your gum bleed?

No

No

Anemia symptoms and other

precipitants

Do you feel fatigued?

Did other people say you look

pale?

Do you feel your heart is

racing?

Do you have shortness of

breath or chest pain?

Not fatigued but more tired

than usual

Not really

No

No

Obstetric history Do you have children?

Did you ever have a

miscarriage?

So you had a total of 3

pregnancies, is that correct?

Were your kids born via normal

vaginal delivery or C section?

At how many weeks?

Any significant complications

with the pregnancies? Post-

partum bleeds? Blood

transfusions?

I have 3

No

Yes

Normal, I’m very blessed, they

were all born healthy

They were all around 39 weeks

No, they were all very smooth

Gyne history

At what age have you had your

fist menstrual cycle?

Did you have heavy, and or

painful periods?

When was your last pap test?

13

No

The last year

Page 249: CANADIAN IMGS GUIDE TO OSCE AND PRACTICE · in an OSCE scenario You can use this study guide to prepare for the NAC OSCE and MCCQE2. The content was reviewed for the most recent updates;

249

Have you ever had an abnormal

pap test?

Have you ever had a

mammogram?

What was the result?

Did you use contraception?

Did you use the pill or IUD?

Do you have hot flashes?

Have you ever had a sexually

transmitted infection?

Any known uterine/ovary

cysts/growths/fibroids/polyp?

Ever require a D&C?

No

Yes, 2 years ago

Normal

I used the calendar all my life

and it worked well.

I took the pill for 2 months in

my 20s but didn’t like it

No

No

I had an ovarian cyst removed

when I was 37, it was benign

No

Past medical and surgical

history

Do you have a medical disease

like diabetes mellitus, or heart

disease?

Have you ever been diagnosed

with cancer, specifically

ovarian, uterine, breast or

colon?

Have you ever had surgery

particularly to uterus and

ovaries other than the one you

told me about?

No, I’m very healthy

No, God forbid

No

Medications and allergy Do you take any drugs? Any

over the counter medications

or herbs?

No

Page 250: CANADIAN IMGS GUIDE TO OSCE AND PRACTICE · in an OSCE scenario You can use this study guide to prepare for the NAC OSCE and MCCQE2. The content was reviewed for the most recent updates;

250

Do you take soy containing

food supplements?

Do you have allergy to

medications or food?

I only take a multivitamin

No

Family history Did anyone in your family have

cancer of the uterus, breast or

colon?

When did your mother/sisters

enter menopause?

Did they have any post-

menopausal bleeding?

No

I think my mom was 50, I don’t

have any sisters

Not that I know of

Social history

Smoking

Alcohol

Occupation

Do you smoke?

Do you drink alcohol?

What do you do for living?

Any weight-bearing exercise?

No

Occasionally

I’m an accountant

No

FIFE

Effect of bleeding on everyday

life and relationship with

husband

Patient ideas?

Etiologies of post-menopausal

bleeding: Atrophy, Endometrial

hyperplasia/cancer, Polyps,

hormone replacement therapy,

fibroids, adenomyosis, from

How is the bleeding affecting

your daily function and

marriage?

I understand, it is for sure

inconvenient. Have you tried

some creams or lubricants?

I see, some of them have a

natural water base that may

not feel as burning. I will give

you a brochure about vaginal

dryness of menopause and

what’s available to help with it.

It’s very annoying, I have to

always wear a pad, and I’m not

enjoying sex anymore

I did but I felt burning

Thank you that would be great.

But what is causing the

bleeding? Do I have cancer?

Page 251: CANADIAN IMGS GUIDE TO OSCE AND PRACTICE · in an OSCE scenario You can use this study guide to prepare for the NAC OSCE and MCCQE2. The content was reviewed for the most recent updates;

251

other organs, anti-coagulants,

medications, soy, post

radiation, infections, cervical

cancer- Need to rule in and rule

out each of these through the

history

There are many causes of

vaginal spotting after

menopause the most common

is dryness and atrophy of

uterine and vaginal lining.

Malignancy can cause spotting

too, as well as fibroids. We

need to run some tests to know

what the cause is. Do you have

particular questions or

concerns?

Not at the moment, thank you

Physical examination;

3- General; Mention that the vital signs are normal. Comment on the body built and indicate that you

would check the weight and height and BMI. Inspect the hands and feel the pulse. Examine the head

and neck for pallor, jaundice, bruising or bleeding and lymphadenopathy

14- Auscultate the heart and lungs

15- Inspect and palpate the abdomen looking for masses, ascites, hepato and splenomegaly.

16- Mention that you would do a genital and vaginal exam, the examiner will say pass.

17- Mention that you would do a breast exam, the examiner may say pass, or let you do it

18- Examine the rest of the inguinal/pelvic lymph nodes – the examiner may say pass

Sample questions you may be asked by the examiner;

1- What is your differential?

Answer; Gynecologic; Endometrial and vaginal atrophy, Cancer, polyps, post-menopausal hormone

therapy, endometrial hyperplasia, fibroids. Or possibly non gynecological bleeding like urinary tract or GI

bleeding. Adenomyosis- typically resolves post-menopausal but is still on the differential

19- What investigations would you order?

CBCD, lytes, urea, creatinine and urine analysis. Hysteroscopy and endometrial biopsy, trans-vaginal

ultrasound

Page 252: CANADIAN IMGS GUIDE TO OSCE AND PRACTICE · in an OSCE scenario You can use this study guide to prepare for the NAC OSCE and MCCQE2. The content was reviewed for the most recent updates;

252

Cervical cancer screen (PAP test) is also necessary part of the pos-menopausal bleeding work-up

Rating scales points;

- Examinee introduced self and position

- Addressed patient with name

- Used proper non-verbal communication

- Organizational skills

- Recognized and managed emergency effectively prioritizing actions

- Examinee spoke clearly (accent didn’t get in the way)

- Good Listening and questioning skills

- Showed rapport with patient

- Attentive to patient physical comfort

- Medical knowledge adequate

- No misinformation was provided to patients

- No concerning ethical/legal issues

- The examinee is respectful of other health care members

Variations of the case and some tips;

Case; 55 year old female presenting with vaginal bleeding

History; 1 month history of moderate vaginal bleeding, almost daily. The patient describes night fevers

and sweating which she thinks are due to hot flashes of menopause. Menopause at age 51. Menarche at

age 11. No HRT. No symptoms of anemia, no GI/Urinary symptoms. The patient is G1P1, last pap test

and mammogram normal, she used IUD for 20 years. Past medical history is significant for obesity with a

BMI of 40, and a history of polycystic ovarian syndrome. The patient is also a known diabetic for 3 years

on Metformin and Gliclazide. No family history of cancer.

Physical examination; Stable vital signs, BMI 40. Normal cardiac, respiratory and abdominal exam.

Vaginal exam confirmed bleeding and was otherwise unremarkable.

Discussion; Unopposed estrogen is a risk factor for endometrial cancer that must be excluded in all

women presenting with post-menopausal bleeding. Make sure to take a thorough history that covers

endometrial cancer risk factors.

Page 253: CANADIAN IMGS GUIDE TO OSCE AND PRACTICE · in an OSCE scenario You can use this study guide to prepare for the NAC OSCE and MCCQE2. The content was reviewed for the most recent updates;

253

– Risk factors that need to be addressed- obesity, nulliparous, PCOS, early menarche, late

menopause (increased duration of estrogen exposure), OCP use, tamoxifen, systemic estrogens,

HRT, polyps, Lynch syndrome, HNPCC

– Hysteroscopy is not key- Biopsy and/or Ultrasound are key

– Biopsy is warranted: anyone >40 yrs & abnormal uterine bleeding (AUB), >90 kg with AUB, post-

menopausal with any bleeding especially if endometial thickness is >4 m, age 45 to menopausal

average 51 yr with AUB, < 45 yr with persistent AUB/unopposed Estrogen/hyperplasia risk

factors/who have failed medical therapy, pre-menopausal with anovulation/amenorrhea > 6

months, atypical glandular cells on pap smear, endometrial cycles on pap smear, monitoring for

known history of hyperplasia, or screening for a women who is at high risk of endometrial

cancer

Case; 53 year old female presenting with vaginal bleeding

History; Menopause at age 51, started estrogen only HRT almost immediately. She had irregular vaginal

bleeding the first 3 months of hormonal therapy. Couldn’t tolerate progesterone, and refused

progesterone IUD because she didn’t want a foreign body in her uterus. The last 2 months bleeding

started again, it is irregular and of mild-moderate amount. No other symptoms. She underwent

hysteroscopy and endometrial biopsy before HRT was started and refused it afterwards because it’s; a

headache. Review of systems unremarkable. No medical illnesses or family history of cancer. She never

got pregnant in her life and never used contraception as her husband is infertile.

Physical examination; Normal vital signs. BMI 23. Normal cardiac, respiratory and abdominal

examination. Vaginal examination revealed a bulky uterus.

Discussion; It is very important in this case to show respect and understanding of the patient own

choices. She could have cancer secondary to unopposed estrogen in her HRT, but you shouldn’t lecture

her how a progesterone containing IUD could have decreased her risk. In this case the patient asks if she

has to undergo this test again; Hysteroscopy and endometrial biopsy. Explain that you understand that

she doesn’t like it but is necessary at this stage because her bleeding may be due to cancer.

-If she refuses that offer her an ultrasound to assess the lining- it doesn't give histologic evidence of

cancer, but at least it is something to follow the endometrial lining with.

Page 254: CANADIAN IMGS GUIDE TO OSCE AND PRACTICE · in an OSCE scenario You can use this study guide to prepare for the NAC OSCE and MCCQE2. The content was reviewed for the most recent updates;

254

-Persistence is key- reinforcing the importance of the biopsy

Page 255: CANADIAN IMGS GUIDE TO OSCE AND PRACTICE · in an OSCE scenario You can use this study guide to prepare for the NAC OSCE and MCCQE2. The content was reviewed for the most recent updates;

255

Case 14; Abortion

Edited by; Dr. Erica Para

Door sign

Melissa Smith, a 27 years old female, 10 weeks pregnant, presents to the ER with vaginal bleeding. US

confirmed fetal loss

Vital signs;

BP 120/80

PR 80

RR 14

Temperature 36⁰C

In the next 10 min Counsel the patient. At 9 min the examiner may ask you a question or questions

Note; The NAC may have a similar stem, but the last 2 lines may look like this; in the next 11 minutes

counsel the patient. At 8 minutes the examiner may ask you a question or questions.

Sample notes to write for your self

Melissa Smith Emotional support

27 F risk factors

Previous miscarriage

OBS Hx, counseling emotional support

Patient encounter

Your actions Suggested verbalizing Patient response

Page 256: CANADIAN IMGS GUIDE TO OSCE AND PRACTICE · in an OSCE scenario You can use this study guide to prepare for the NAC OSCE and MCCQE2. The content was reviewed for the most recent updates;

256

Opening start Hi Melissa, how are you doing

today?

Miserable... patient crying, I lost

my baby

Offer a tissue and sympathize

You allow a moment of silence

It must be hard for you

I don’t know why, I did all the

right things; I took multivitamins,

folic acid, I didn’t drink I don’t

smoke, I don’t understand, my

doctor said things are all looking

good, I don’t understand

Help vanish feelings of guilt

- Very important to re-

emphasize that the patient

should NOT Blame themselves

for the lose- it is nothing they

did, or could have done to

prevent it- 1/3 of pregnancies

miscarry- there was something

“wrong” with the pregnancy- it

would not have been a healthy

pregnancy- re-enforce to the

patient that you understand it is

difficult, but that it is NOT their

fault.

It is not your fault, You didn’t do

anything wrong, I want you to

understand

The patient looks calmer and

more receptive

Loss of a baby this early is

usually due to abnormal baby

chromosomes and this is not due

to anything the mother had

done. You did all the right things

and it’s not your fault, it was just

an unhealthy pregnancy

But we had sex; did I lose him

because we had sex?

Page 257: CANADIAN IMGS GUIDE TO OSCE AND PRACTICE · in an OSCE scenario You can use this study guide to prepare for the NAC OSCE and MCCQE2. The content was reviewed for the most recent updates;

257

No, sexual activity doesn’t cause

miscarriage

So why?

Spontaneous abortion occurs in

8-20% of pregnancies- of

recognized pregnancies, rates

~1/3 if unrecognized pregnancies

are included. It tends to occur

more in older women who had

many pregnancies, and in

women who had prior

spontaneous abortions. Smokers

and women who consume large

amounts of alcohol or take

cocaine are at higher risk. Other

risk factors include Non-steroidal

pain killers like Advil, Voltaren.

Tylenol is not part of this group

of pain killers and doesn’t cause

abortion. Consuming large

amount of coffee like 10 cups is

a risk factor too

Patient smiles and looks more

comfortable; I don’t belong to

any of those

You smile and nod your head in

understanding; absolutely

But what else may cause a

miscarriage?

In addition to problems with the

chromosomes, (50%), congenital

anomalies of the fetus, trauma,

maternal diseases, specific

infections we call TORCH.

Abnormal structure of the uterus

like having a septum can cause

miscarriage, we can do an

ultrasound to check it out, and a

Page 258: CANADIAN IMGS GUIDE TO OSCE AND PRACTICE · in an OSCE scenario You can use this study guide to prepare for the NAC OSCE and MCCQE2. The content was reviewed for the most recent updates;

258

lot of structural abnormalities

can be treated surgically

Sure, I will arrange for one,

sometimes the mother may have

a disease like a thyroid problem

that may cause a miscarriage

Do you mind if I ask you some

questions about your health to

see if we can identify a cause

that we can treat?

I see, I want an ultrasound done

No, not at all, please go ahead

Obstetric history

GTPAL

Accurate dating of Pregnancy

Symptoms of pregnancy

Symptoms of abortion

Medications/Herbs/X-ray

exposure

Was this your first pregnancy?

When was your last menstrual

period?

Was it your normal period? Are

you certain of your dates?

When did you find out you were

pregnant?

By what method- blood work,

urine sample, ultrasound?

Did you have nausea and

vomiting?

Can you tell me what happened

when you lost the baby?

Yes, I’ve never been pregnant

before

March 10

Yes

Yes

Around April 15

I did a urine test

I felt nauseated in the mornings

but no vomiting

I woke up from sleep with

vaginal bleeding and some

abdominal pain. I came to the

hospital immediately and they

told me I lost the baby

Page 259: CANADIAN IMGS GUIDE TO OSCE AND PRACTICE · in an OSCE scenario You can use this study guide to prepare for the NAC OSCE and MCCQE2. The content was reviewed for the most recent updates;

259

Sorry about your loss.

Did you take any medications or

herbs during pregnancy?

Were you exposed to X ray?

Nothing at all

No

Relevant review of systems

Infection

Endocrine

Do you have fever? Chills?

Do you have vaginal discharge?

Do you have skin or hair

changes? Rashes? Flu-like

symptoms?

Do you feel intolerant to heat or

cold?

Are you constipated or do you

have diarrhea?

Do you feel a change to your

energy level?

No

No

No

No

No, I’m very regular

No

Relevant Gyne history

Contraception

STD

Pap test

Have you used contraception

before?

Have you ever had a sexually

transmitted infection?

When was your last pap test?

Have you ever had an abnormal

pap test?

I was on the pill throughout my

life, I stopped 6 months ago as

we started trying

No

A year ago

No

Past medical and surgical history Do you have any disease? No, I’m very healthy

Page 260: CANADIAN IMGS GUIDE TO OSCE AND PRACTICE · in an OSCE scenario You can use this study guide to prepare for the NAC OSCE and MCCQE2. The content was reviewed for the most recent updates;

260

Have you ever had blood clots?

Have you ever had surgery?

No

No

Medications and allergy Do you have allergies to

medications or food?

No

Family history Did anyone in your family have

blood clots?

No

Social history

As implied above most points

are covered

Is your partner here?

How did he take the news?

Would you like me to talk to

him?

Do you work?

Do you need a note for your

work?

Yes, my husband is in the waiting

room

He looked sad but stayed strong

for me

Yes, that would be great

Yes, I’m a receptionist

Yes please

Closure and some

recommendations

How are you feeling now?

Do you have any questions for

me

It takes about 8 weeks for the

changes of pregnancy (for

example; breast tenderness) to

fully resolve. Your next

menstrual cycle is expected to

resume 4-6 weeks from now. Are

you planning to try again?

I understand. I would like to see

you again in clinic to talk more

about what to expect next time.

In the mean time we will do

Much better, it helps to know it’s

not something I did

Not at the moment

Not at the moment. I will take

the pills again, till we both feel

ready.

Page 261: CANADIAN IMGS GUIDE TO OSCE AND PRACTICE · in an OSCE scenario You can use this study guide to prepare for the NAC OSCE and MCCQE2. The content was reviewed for the most recent updates;

261

some blood work, and book the

Ultrasound. Do you have any

questions for me?

Not at the moment. Thank you

Physical examination;

No physical exam is required in this case, if required see below

Sample questions you may be asked by the examiner;

1- What is your differential for vaginal bleeding in the first trimester?

Answer; Physiologic-implantation bleeding, Complete abortion, Hetero-topic pregnancy,

cervical/vaginal/uterine pathology- polyps etc. cervical ectropion, intercourse/trauma related, septic

abortion, missed abortion, inevitable abortion, non- gynecological source of bleeding like GI or urinary

tract

2- What investigations would you order initially for a patient presenting with vaginal bleeding

in the first trimester?

Answer; CBCD, type and screen and antibodies, B-HCG- quantitative, trans-vaginal ultrasound.

Rating scales points;

- Examinee introduced self and position

- Addressed patient with name

- Used proper non-verbal communication

- Organizational skills

- Recognized and managed emergency effectively prioritizing actions

- Examinee spoke clearly (accent didn’t get in the way)

- Good Listening and questioning skills

Page 262: CANADIAN IMGS GUIDE TO OSCE AND PRACTICE · in an OSCE scenario You can use this study guide to prepare for the NAC OSCE and MCCQE2. The content was reviewed for the most recent updates;

262

- Showed rapport with patient

- Attentive to patient physical comfort

- Medical knowledge adequate

- No misinformation was provided to patients

- No concerning ethical/legal issues

- The examinee is respectful of other health care members

Variations of the case and some tips;

1. As noted in this case, when the patient is sad or upset, the priority is to provide emotional

support. Starting with the history without acknowledging the patient’s feelings is going to make

the patient even more upset and less cooperative. It may be challenging during the exam

because you are under the pressure of time, but remember that SP's are well trained and your

communications skills are tested. Inappropriate behavior or ignoring the patient feelings may

cost you the whole station

2. Case; 30 year old female, 11 weeks pregnant, presenting with vaginal bleeding

History; GA 9 weeks confirmed by date of last menstrual period. Bleeding started 2 hours ago and is

mild. No abdominal pain. This is the second pregnancy, the first pregnancy was 2 years ago; no

complications and ended with vaginal birth of healthy baby at 39 weeks. No trauma, infection, GI,

endocrine or urinary symptoms. The patient is taking folic acid and multivitamins. No exposure to

NSAIDs, smoking, alcohol cocaine or X ray. The patient is healthy, and family history is noncontributory.

Physical examination; stable vital signs, Chest and abdominal examination unremarkable. Pelvic

examination excluded local source of bleeding, vaginal examination confirmed mild vaginal bleeding,

closed cervix, and GA of 11 weeks. Fetal heart beats were detected with Doppler US.

Discussion; Threatened abortion is the most likely diagnoses. Watchful waiting is sufficient in most

cases. It is important to emotionally support the patient and counsel her. B-HCG levels help sort out the

differential along with US. Progestin may be used but its use is controversial. Bed rest is typically

recommended- although evidence on this is lacking- but in practice it is done. Important to remind the

patient that they are at increased risk of miscarriage, pre-term birth, premature rupture of membranes,

Page 263: CANADIAN IMGS GUIDE TO OSCE AND PRACTICE · in an OSCE scenario You can use this study guide to prepare for the NAC OSCE and MCCQE2. The content was reviewed for the most recent updates;

263

antepartum bleeding and growth restricted babies- it is important that if they have any more bleeding

to come directly to hospital to assess what is happening and the viability of the pregnancy.

3. Case; 29 years old female 10 weeks pregnant, presenting with vaginal bleeding and fever

History; 10 weeks GA confirmed by last menstrual period. The patient had 2 sexually transmitted

infection in the past; Gonorrhea and Chlamydia. She is presenting with fever, abdominal pain and

vaginal bleeding. She got pregnant over an IUD that was left in place. No other symptoms. The patient is

non-smoker, non-drinker and doesn’t do drugs.

Physical examination; Temperature 38.8 ⁰C, PR 110, BP 110/70, RR 18. Chest examination was normal,

abdominal examination revealed lower abdominal tenderness. No fetal heart beat detected and vaginal

examination showed an open cervix with vaginal bleeding and passing products of conception.

Discussion; Septic abortion can be fatal, prompt diagnosis and treatment can save the patient’s life. If

suspected draw blood culture and send high vaginal swabs for culture and sensitivity and start the

patient on broad spectrum antibiotics. Evacuation of uterine content is the next step; get an OBS consult

for D&C.

Antibiotic choices are:

a. Clindamycin 900 mg IV q8h & Gent 5 mg/kg/day +/- Ampillicilin 2 g IV q 4h

b. Ampicillin 2 g IV q4h & Gent 5 mg/kg/day & Flagyl 500 mg IV q8h

c. Pipracillin-Tazobactam 4.5 mg IV q6h

d. Imipenem 500 mg IV q6h

e. IV Antibiotics are given for at least 48 hours and symptoms start to improve, they are

then stepped down to PO Antibiotics for at least 10-14 days.

f. Urgent uterine evacuation is key once the patient is hemodynamically stabilized.

Page 264: CANADIAN IMGS GUIDE TO OSCE AND PRACTICE · in an OSCE scenario You can use this study guide to prepare for the NAC OSCE and MCCQE2. The content was reviewed for the most recent updates;

264

4. Induced abortion remains a controversial issue. There are no legal rules to regulate abortion in

Canada. It is considered a medical procedure and the decision is left up to the patient with the

help, guidance and counseling of her physician. Here are some important terms;

Abortion; is the termination of pregnancy before fetal viability (500g or 24 weeks gestation)-

Termination of pregnancy; is the term used to describe pregnancy termination after the age of viability

There are obvious cases where abortion or termination of pregnancy is done for medical reasons (save

the mother’s life, or where fetal anomalies inconsistent with life exist). But there are cases where

controversy arises as in unplanned pregnancy and teenage pregnancy. In all cases it is the mother’s

decision. It is important to keep in mind that such a decision is already hard for the patient and need not

be made more difficult by judgments. You need to explore the patient request for abortion; why does

she want abortion? Is she worried about her career plans? Is she afraid to keep the baby? Is money a

problem? What types of support does she have? Is the father involved in the decision? What is her

relationship with the father? Is there an emotional or physical abuse? Did she share the news of

pregnancy with any one? Would she get help from her family and friends? Would she consider keeping

the baby if more help is provided? Does she know about foster care? You may want to refer the patient

to the social worker if social and financial issues are part of the problem.

5- Be reminded that a full obstetrical and medical history is part of comprehensive assessment in

these cases.

6- You need to put your personal opinions aside and give a pure medical advice; identify medical

indications for abortion if any and go over the risks of the procedure. Timing is very important

and the mother should know that it will be hard to find a practitioner to terminate pregnancy

for no medical reason after the age of viability. However, risks are much less and abortion is

accessible in Canada for personal reasons before 20 weeks. The only option after 20 weeks

might be to go to the States and the patient should be informed about all her options.

7- Physicians have the right to choose not to be involved in abortion and in this case they should

inform the patient in a timely fashion and in a nonjudgmental way. They still have the duty to

treat any medial problem within their scope of practice. - if not refer on to another colleague

that can help

Page 265: CANADIAN IMGS GUIDE TO OSCE AND PRACTICE · in an OSCE scenario You can use this study guide to prepare for the NAC OSCE and MCCQE2. The content was reviewed for the most recent updates;

265

Case 15; Cancer patient refusing treatment

Edited by; Dr Sadik Salman

Door sign;

Mary Christopher, 72 years old female, was diagnosed with a right breast lump highly suspicious for

malignancy 2 years ago. At that time she declined further investigations and treatments including

surgery and opted to treat herself with prayers. She is presenting today after being convinced by her

husband to repeat the CT scan which showed bilateral breast lumps, lung and thoracic spine metastases.

Her family doctor referred her to the oncology clinic for further evaluation and management. You are

the resident working in the clinic.

Vital signs;

BP 120/70

PR 85

RR 14

Temperature 37 ⁰C

In the next 10 minutes explore the patient’s ideas and concerns

Note; the NAC may have a similar stem but the last line will look like this; in the next 11 minutes explore

the patient’s ideas and concerns

Take 2 min to read and understand the question, it is about exploring the patient’s view, and this is all

about your communication skills!

Sample notes to write for yourself;

Mary Christopher Emotional support

72 F Respect and explore patient choice

Hx and counsel Functional status/pain

Does the patient want treatment?

Patient ideas about treatment

Page 266: CANADIAN IMGS GUIDE TO OSCE AND PRACTICE · in an OSCE scenario You can use this study guide to prepare for the NAC OSCE and MCCQE2. The content was reviewed for the most recent updates;

266

Palliative/oncology consult

Patient encounter;

Your actions Suggested verbalizing Patient response

Opening start Mrs. Christopher, It’s my

pleasure to meet you.

I understand that you was

diagnosed with a breast lump

2 years ago and didn’t want

any further investigation or

treatment at that time, is that

right?

I also gather that you had a

recent chest CT scan, what do

you know about it?

It must be hard for you to

hear that

Offer a tissue and a moment

of silence

Pleasure to meet you

That’s correct

My doctor said I have breast

cancer that is now spread to

my lungs, and bones

Patient cries

How have you been feeling? I’m very tired, I need to rest a

lot and it takes forever to get

the house work done

For how long have you been

feeling tired?

Well, I’ve been always tired

but the last 3 months were

bad for me

Page 267: CANADIAN IMGS GUIDE TO OSCE AND PRACTICE · in an OSCE scenario You can use this study guide to prepare for the NAC OSCE and MCCQE2. The content was reviewed for the most recent updates;

267

I see, Are you getting any

help?

What are they helping you

with?

Did you ever need help with

personal care?

My husband and daughter

are always around, they are

doing a lot of things for me

My daughter did the laundry

the other day, and my

husband is going shopping by

himself, I don’t have energy

to accompany him anymore

No, I can take care of myself

Pain Do you have any pain? I get back pain right here

(Patient points to the middle

of her back)

Details about pain and

exploring patient wishes

This is where the tumor has

gone to bone, How often do

you feel the pain?

What do you do about it?

I’m sorry to hear you are in

pain; I would like to help you

feel better. Would you be

open to treatments we can

offer for the pain? I can

prescribe you a stronger

analgesic and we can look

into radiation therapy to the

bone

I will be happy to prescribe

you an analgesic, and if you

don’t mind I would like to

hear more about your

thoughts regarding the

It’s always there but not very

bad, it’s really bad in the

morning

I take Tylenol, but it’s not

helping

I will take an analgesic, but

not your fancy cancer

treatment

I don’t want any

chemotherapy or radiation.

This cancer is going to be

Page 268: CANADIAN IMGS GUIDE TO OSCE AND PRACTICE · in an OSCE scenario You can use this study guide to prepare for the NAC OSCE and MCCQE2. The content was reviewed for the most recent updates;

268

radiotherapy for the pain; it is

not a cancer treatment, it is

only treatment for the pain

I see where you are coming

from, But don’t you think that

a medical therapy can be your

miracle?

I mean you can continue to

pray and it’s great to have

hope in prayer, this can go

along with medical therapies

Why do you refuse medical

therapy?

I’m listening

treated by a miracle and I’m

praying everyday

I never thought about that

Patient gives you a curious

look as if you opened her

eyes to something she never

considered before

Patient sighs and moves her

eyes away from you

I had a sister who died from

breast Cancer, she was only

50 and suffered a lot from

chemotherapy, she didn’t

believe in miracles but the

treatment killed her and I

don’t want to suffer from

your drugs, many people cure

their cancer with prayers

every day. I’d rather go in

peace

Page 269: CANADIAN IMGS GUIDE TO OSCE AND PRACTICE · in an OSCE scenario You can use this study guide to prepare for the NAC OSCE and MCCQE2. The content was reviewed for the most recent updates;

269

You allow a moment of

silence, the patient looks

back at you, you nod your

head encouraging her

I’m sorry about your loss, it is

very hard to see our loved

ones suffer, I see where you

are coming from, and my goal

is to decrease suffering as

much as possible.

Unfortunately, we can ‘t cure

your cancer at this stage, but

we can help you live your

remaining life with minimal

suffering

Cancer cells in your bone are

causing the pain and we can

kill them by radiotherapy.

However, you need to be

evaluated by the

radiotherapist who is

experienced in this sort of

treatment and can tell you if

it can be done for you and

how well it will help

I will refer you if you like

I will prescribe you an

analgesic and refer you. Do

you have any other pain?

What other concerns do you

have about cancer

treatment?

What is this radiotherapy

treatment for my back pain?

And how can I see the

radiotherapist

Yes I would like

No

I don’t want chemotherapy, I

can’t die miserable as my

sister

Page 270: CANADIAN IMGS GUIDE TO OSCE AND PRACTICE · in an OSCE scenario You can use this study guide to prepare for the NAC OSCE and MCCQE2. The content was reviewed for the most recent updates;

270

I understand, there are other

new treatments for breast

cancer like hormonal therapy

and new drugs against

specific receptors of cancer

cells called immunotherapy.

The newer chemotherapy

agents are not as toxic as

older ones

I’m here to help, and I can

refer you to the oncologist

who can get you tested to see

which therapy works for you

because it depends on the

type of cancer, and he can

give you more detailed

information about the drugs

that may help including their

side effects, you can think

about it and decide for your

self

I will make sure you see an

oncologist as soon as

possible. Do you have other

questions or concerns?

No one told me this 2 years

ago, that’s all new for me

I want to see an oncologist, I

can’t believe no one talked to

me about this 2 years ago

Would the oncologist send

me for surgery? I don’t want

to be slaughtered by a

Page 271: CANADIAN IMGS GUIDE TO OSCE AND PRACTICE · in an OSCE scenario You can use this study guide to prepare for the NAC OSCE and MCCQE2. The content was reviewed for the most recent updates;

271

You offer tissues, and allow a

moment of silence

I understand your concerns,

the oncologist and his team

will evaluate your case and

may offer you surgery, and it

is your decision at the end.

Can you tell me more about

your sister’s surgery?

I’m sorry your sister suffered

a lot. However what

happened to her will not

necessarily happen to you.

I will talk to him about your

concerns and I’m sure he is

going to be supportive. Do

you have any other concerns?

We all try to do our best, I’m

here for you if you need

anything

surgeon, my sister suffered a

lot from them

Well, apparently they were

not clean enough, and her

wound got infected.(patient

cries)

This oncologist better be a

human and listen to me

Thank you, I wish I was your

patient 2 years ago, the other

doctor was idiot.

Thank you

Physical examination;

Not required in this case

Page 272: CANADIAN IMGS GUIDE TO OSCE AND PRACTICE · in an OSCE scenario You can use this study guide to prepare for the NAC OSCE and MCCQE2. The content was reviewed for the most recent updates;

272

Sample questions you may be asked by the examiner;

None in this case

Rating scales points;

- Examinee introduced self and position

- Addressed patient with name

- Used proper non-verbal communication

- Organizational skills

- Examinee spoke clearly (accent didn’t get in the way)

- Listening and questioning skills

- Showed rapport with patient

- Attentive to patient comfort

- Was professional in responding to patient frustration

- Medical knowledge adequate

- No misinformation was provided to patients

- No concerning ethical/legal issues

- The examinee is respectful of other health care members

Variations of the case and some tips;

1- I saw this patient during my oncology rotation. It is one of the most challenging cases I saw in

my life. The oncologist treating this patient was a master in communication. I took off the highly

specialized oncology counseling and focused on the communication part. In this case the patient

is seen by a family doctor and is not yet evaluated by the oncologist. As you can see

professionalism is key and can be illustrated in the following points;

- Respect the patient decision to forgo treatment; no matter how odd it may seem to you when a

patient with a deadly disease refuses all sorts of treatment. It is the patient’s decision at the

end. If you were the person assessing the patient when she refused treatment then you need to

make sure she is competent to make the decision by excluding delirium, dementia, psychosis

and depression. If the patient is competent then her decision must be honored. Trying to involve

family members without her consent, or coerce her in a way or another is not acceptable.

Page 273: CANADIAN IMGS GUIDE TO OSCE AND PRACTICE · in an OSCE scenario You can use this study guide to prepare for the NAC OSCE and MCCQE2. The content was reviewed for the most recent updates;

273

- Explore the patient reasoning; a caring warm non-judgmental attitude encourages the patient to

open up. This can be illustrated by asking about pain, showing concern and offering to help. You

can gain your patient’s trust by understanding where she is coming from and being

compassionate and sensitive about her feelings. This way your patient will open up even more

- Non-verbal communication; is very important. Offering a tissue, allowing a moment of silence,

making appropriate eye contact are examples. Being truly sympathetic with the patient will

show in your body language. Treat SPs as you would treat your own patients; body language

can’t be faked

- Explore again; when the patient asked about surgery it was obvious there is a story behind it.

The patient was very much drawn into her sister’s tragedy that she couldn’t see other

possibilities. Helping the patient open up in a safe non-judgmental environment will allow her to

see other facets of the issue and consider new options. Always ask about further concerns

- Dealing with the patient emotions; the patient in this case was sad, frustrated, then she showed

some denial and anger. Allow the patient to ventilate her feelings and stay professional

- Focusing on patient’s well fare; when the patient denied any explanation about her case by her

doctors 2 years ago, you may have said to yourself; I’m sure they told you everything! However,

it is better to ignore the patent statement as arguing with her will unlikely take you anywhere.

The other example is when she concluded that her sister’s wound infection was medical

negligence; not going into details is best, as she is expressing her frustration about an old life

event. Reminding her that everyone is different and that her decisions will be honored

encouraged her to open the door to treatments she was absolutely refusing before is more

positive. It is your goal to help her decide what is best for her, and it’s not always easy

- I encourage you to review the basic ethical principles presented in detail in chapter 6

Page 274: CANADIAN IMGS GUIDE TO OSCE AND PRACTICE · in an OSCE scenario You can use this study guide to prepare for the NAC OSCE and MCCQE2. The content was reviewed for the most recent updates;

274

Case 16; Neonatal jaundice

Edited by; Dr. Abbeir Hussein

Door sign;

Suzan Schmitt, mother of Peter, 1 week old boy, comes to the public health center because her

son has jaundice

Vital signs;

BP 80/42

PR 120

RR 40

Temperature 37 ⁰C

In the next 10 min obtain a focused and relevant history. At 9 minutes the examiner may ask

you a question or questions

Note; The NAC may have a similar stem but the last 2 lines will look like this; in the next 11

minutes obtain a focused and relevant history. At 8 minutes the examiner may ask you a

question or questions

Sample notes to write for yourself;

History source mum Suzan Schmitt Onset, duration, extent

Peter 1 week Feeding, sleep, irritability

Prenatal, natal and postnatal

Pediatric Hx hematoma, blood group

Mum well being

Patient encounter;

Your actions Suggested verbalizing Patient response

Page 275: CANADIAN IMGS GUIDE TO OSCE AND PRACTICE · in an OSCE scenario You can use this study guide to prepare for the NAC OSCE and MCCQE2. The content was reviewed for the most recent updates;

275

Opening start Congratulations on the birth of your son Peter, how can I help you today? Sure, I need to ask you few questions about Peter’s health to make sure everything is ok

My son Peter looks really yellow to me, I know babies get jaundice, but I want to make sure it’s not worrisome No problem

Onset and progression When did he start looking yellow to you?

I noticed his eyes turned yellow yesterday, but today his face and neck are yellow too

And he is one week, right? Yes

Related symptoms Fever Feeding Sleep Cry Urine output Stool Blood in stool Skin rash

Did he have fever? Are you breast or formula feeding? How often do you feed him? For how long do you feed him each time? Does he throw up? Does he wake himself up to feed or do you have to wake him up? Can you feel your let down? Is he fussy? How many times does he poop? And what color are his stools? And how many wet diapers? Did you notice any blood or mucous on the diaper? What color is his urine? Do you have concerns about him other than the jaundice?

No, I checked his temperature and it’s 37 ⁰C Only breast feeding Every 2.5-3 hours About 40 minutes Not really, he spits up a little sometimes after burping No he wakes up by himself Yes No he’s easy to settle A lot, 8-10 times? Watery yellow with seeds About 12 I would say No Light yellow No

Peri-natal history Prenatal

I’m going to ask you few questions about your health during pregnancy

Page 276: CANADIAN IMGS GUIDE TO OSCE AND PRACTICE · in an OSCE scenario You can use this study guide to prepare for the NAC OSCE and MCCQE2. The content was reviewed for the most recent updates;

276

Natal Post natal

Was this your first pregnancy? Were there any problem when you were pregnant? And diabetes or high blood pressure? Did you take any medications during your pregnancy? Any smoking? Any alcohol use Any recreational drug use? Do you know your blood group? How about your husband? How many weeks were you pregnant when you gave birth? Was he born vaginally or by C section? Was there swelling of Peter’s head? For how long did your water break before peter was born? Was it green, or stained with blood? Did you have a fever during your labour and delivery? Do you know whether you were group b strep positive or negative? Any vaginal lesions when baby was being born Do you know what his APGAR scores were? What was his birth weight? Did you weigh him after? Did he need NICU care? When were your baby and yourself discharged from hospital?

Yes No No, I’m very healthy No None A glass of wine before I knew I was pregnant None A+ A+ too 39 weeks It was normal but the doctor had to use vacuum Yes, but it’s going down I would say about 6 hours No, it was clear No Negative No It was 9 and 9 7.6 lb No No The next morning

Page 277: CANADIAN IMGS GUIDE TO OSCE AND PRACTICE · in an OSCE scenario You can use this study guide to prepare for the NAC OSCE and MCCQE2. The content was reviewed for the most recent updates;

277

Did your baby have jaundice before discharge? How about fever?

No No

Development and growth Not much at 1 week- feeding and pooping covered

Does your baby look at you when you breast feed him? it’s precious for sure

Yes, it’s very lovely

Vaccination Nothing at this age unless the mother is Hepatitis B carrier

Medications and allergy Did Peter receive any medications? Does he have any allergies?

They gave him a vitamin K shot in hospital and some drug in his eyes No

Family history Consanguinity Genetic disorders in the family

Are you and your husband related in any way, other than through marriage? Does any genetic disorders run in your or your husband’s families?

No Not that I know of

Social history Smoking, alcohol and drug use by parents covered

How are you coping with a new baby? It can be overwhelming sometimes Who helps you care for him? It’s good to hear you have help How much time off are you taking?

It is tiring but we are very happy My husband is a very involved dad, and my parents are always around I’m taking a whole year off

FIFE From what you told me It seems Peter is doing well, and has what we call physiologic jaundice that some babies develop, it

Page 278: CANADIAN IMGS GUIDE TO OSCE AND PRACTICE · in an OSCE scenario You can use this study guide to prepare for the NAC OSCE and MCCQE2. The content was reviewed for the most recent updates;

278

usually lasts for 1 week. However we need to weigh him, check his temperature and I need to examine him to make sure that’s what it is. Do you have any questions for me?

No, thank you

Physical exam;

Not required in this case

Sample questions you may be asked by the examiner;

1- What is the significance of head swelling the mother described her baby had?

Answer; It could represent soft tissue swelling or a hematoma. If it was a hematoma,

then the baby is at a higher risk for pathological elevation of bilirubin and kernicterus

2- What other risk factors does this baby have for neonatal jaundice?

Answer; He is exclusively breast fed

3- What investigations would you order?

Answer; bilirubin levels either; serum bilirubin (direct and indirect) or transcutaneous,

CBCD, direct coombs test, Peripheral smear, reticulocyte count

Rating scales points;

- Examinee introduced self and position

- Addressed patient with name

- Used proper non-verbal communication

- Organizational skills

- Examinee spoke clearly (accent didn’t get in the way)

- Good Listening and questioning skills

- Showed rapport with patient

- Attentive to patient comfort

Variations of the case and some tips;

Page 279: CANADIAN IMGS GUIDE TO OSCE AND PRACTICE · in an OSCE scenario You can use this study guide to prepare for the NAC OSCE and MCCQE2. The content was reviewed for the most recent updates;

279

1- Case; 8 days old with jaundice

History; term baby born at 38 weeks by normal vaginal delivery after uneventful

pregnancy. The mother is G2P2, and is healthy. Jaundice appeared on day 7, and

is extending to the chest. The baby is exclusively breast fed, the amount of milk

seems to be adequate as his urine and stool outputs are within normal. No other

symptoms and no fever. His older sibling had jaundice during the first week of

life that continued for 3 weeks, he was exclusively breast fed, investigations

were negative and no treatment was required.

P.E; none required

Discussion; This baby may have breast milk jaundice which is seen in some

exclusively breast fed babies for no apparent reason. It usually resolves on its

own. Thorough evaluation is required, and if bilirubin levels are not in the

dangerous zone then the mother is encouraged to continue breast feeding.

Breast milk jaundice is different from breast feeding jaundice. In breast feeding

the baby is not getting enough breast milk, either because of improper latch or

decreased milk supply of the mother. Evaluate breast feeding techniques and

urine output to determine if milk supply is sufficient.

2- Case; 2 days old presenting with jaundice on the day of discharge from hospital

History; Term baby, normal vaginal delivery and uneventful pregnancy. The

mother is primigravida and the baby is on formula as the mother doesn’t wish to

breast feed. Baby’s temperature was 38, and his urine was smelly. The nurse

thinks he is more lethargic than usual newborns. The mother is concerned

something is wrong with her baby and wondering if her choice of not breast

feeding is causing her son’s jaundice.

P.E; not required

Discussion; Sepsis due to UTI or other infections can cause jaundice in newborns.

It is important to do proper evaluation including blood and urine culture and

chest X ray in addition to the investigations mentioned above to evaluate any

newborn with jaundice. Supporting the mother is very important, explain to her

that her choice of not breast feeding has nothing to do with her baby’s jaundice.

It is a good idea to assure the patient that you respect her choice and offer to

Page 280: CANADIAN IMGS GUIDE TO OSCE AND PRACTICE · in an OSCE scenario You can use this study guide to prepare for the NAC OSCE and MCCQE2. The content was reviewed for the most recent updates;

280

talk to her about breast feeding benefits when she is ready. If this was a

counseling case then you need to explore the reasoning behind not breast

feeding, it could be she can’t take time off from work or school in which case you

can involve the social worker.

3- The following guidelines provide a good summary of neonatal jaundice;

http://www.nice.org.uk/nicemedia/live/12986/48679/48679.pdf

Page 281: CANADIAN IMGS GUIDE TO OSCE AND PRACTICE · in an OSCE scenario You can use this study guide to prepare for the NAC OSCE and MCCQE2. The content was reviewed for the most recent updates;

281

Case 17; A toddler with a cleaning agent into his eyes

Edited by; Dr. Abbeir Hussein

Door sign

You are about to see Mary Duggan, Mother of 3 years and 2 months old John Duggan. John accidently

got a cleaning agent into his eyes today. His mother called 911, and he is now being evaluated by

ophthalmology.

Vital signs

BP 90/60

PR 100

RR 30

Temperature 36.9 ⁰ C

In the next 10 minutes, obtain a focused and a relevant history. At 9 minutes the examiner may ask you

a question or questions.

Note; The NAC exam may have a similar stem but the last 2 lines may look like this; In the next 11

minutes obtain a relevant and focused history. At 8 minutes the examiner may ask you a question or

questions

Sample notes to write for yourself

Mary, mother What, when how, how much

John Duggan 3 years both or one eye

Any ingestion? Other injuries

Cleaning agent into eyes Symptoms, intervention

EMS assessment

Page 282: CANADIAN IMGS GUIDE TO OSCE AND PRACTICE · in an OSCE scenario You can use this study guide to prepare for the NAC OSCE and MCCQE2. The content was reviewed for the most recent updates;

282

Hx Pediatric history, screen for abuse

Patient encounter

Your actions Suggested verbalizing Patient response

Opening start I’m sorry about what

happened to john

The patient looks at you and

cries hysterically

I’m a bad mother, my son is

going to be blind

You offer a tissue;

I know this is quite stressful,

but he is getting examined by

the eye doctor now, and we

should have a clearer idea

about the extent of his injury

soon

You allow a moment of

silence, then say; We will do

our best, he is in good hands

Patient sighs, and after about

10 seconds (seems like

eternity in exam settings!)

says; I hope he doesn’t lose

his sight. I don’t know how I

forgot to hide Lysol

Details about the incident Do you mind if I ask you few

questions?

Can you tell me about what

happened?

No, go ahead

John is a good boy, always

trying to help, and likes to do

everything mommy does. I

forgot Lysol open in the

bathroom, he sneaked in and

managed to spray some on

the floor, then it looks like he

tried to wipe it with his bare

hands. I heard him screaming

Page 283: CANADIAN IMGS GUIDE TO OSCE AND PRACTICE · in an OSCE scenario You can use this study guide to prepare for the NAC OSCE and MCCQE2. The content was reviewed for the most recent updates;

283

It must be hard for you

You offer tissues and after a

few seconds ask; Do you feel

good enough to continue?

We will do our best, I promise

How much do you think he

sprayed?

Did he touch both eyes?

When you saw him, were his

eyes red? And tearing?

Was he able to see?

Do you think he ingested

some of it?

Do you think he fell down or

had other injuries?

What type of Lysol?

Good, I will like to look at it,

to see what exactly it has in it

What did you do when you

got to him?

and ran upstairs to find him

rubbing his eyes in pain

Patient bursts in cry again

Yes, I want you to help him

It’s hard to tell, a table spoon

maybe

Yes

Yes

I don’t know, I hope so

It’s hard to tell if he put his

hand in his mouth before I

reached him

No

The bathroom cleaner, I

brought the bottle with me

I rinsed his eyes with water

for about 10 minutes then I

Page 284: CANADIAN IMGS GUIDE TO OSCE AND PRACTICE · in an OSCE scenario You can use this study guide to prepare for the NAC OSCE and MCCQE2. The content was reviewed for the most recent updates;

284

You did the right thing

What did the emergency

responders do when they

arrived?

How long did it take to get to

the hospital?

Did John throw up?

Did he have difficulty

breathing?

Did he complain of pain other

than in his eyes?

called 911, who instructed

me to rinse even more

They checked his heart and

blood pressure and

continued to rinse his eyes

all the way to the hospital

I think the whole thing was

about 30 minutes

No

No

No

Did John have any other

injuries in the past?

Has anything like this ever

happened before?

He is a very active boy, he

had occasional falls and one

small cut wound but nothing

major

No

Perinatal history Do you have other children?

Were there any concerns or

problems with the

pregnancy?

Was John early, late or on

time?

No

No

On time

Page 285: CANADIAN IMGS GUIDE TO OSCE AND PRACTICE · in an OSCE scenario You can use this study guide to prepare for the NAC OSCE and MCCQE2. The content was reviewed for the most recent updates;

285

And when he was born, were

there any problems?

Did he need to go to the

NICU?

As a baby did he have any

problems?

Was he ever admitted to the

hospital?

Does he have any medical

conditions?

No

No he went home right away

with me

No, he is a healthy boy

No

No

Vaccination Has he received all of his

immunizations?

Yes he is up-to-date

Nutrition What is John’s diet like?

He loves food, I make sure

he gets enough meat and

veggies every day, he also

drinks milk

Growth and development Are you still doing ok?

I will let you know as soon as

he is finished

Do you mind if I ask you a few

questions about his

development to make sure

everything is ok?

Do you have any concerns

about his development?

Can he stand on one foot?

Can he ride a tricycle?

I want to know what does

the eyes doctor think

Sure that’s fine

No

Yes, only for few seconds

Yes

Page 286: CANADIAN IMGS GUIDE TO OSCE AND PRACTICE · in an OSCE scenario You can use this study guide to prepare for the NAC OSCE and MCCQE2. The content was reviewed for the most recent updates;

286

Can he draw a circle?

Wow, that’s impressive

Do you have any concerns

about his speech?

How much of what he says do

you think a stranger could

understand?

Does he know his body parts?

Does he play with other kids?

Do you ever see him doing

any pretend play, like for

example pretending to feed a

baby?

Yes, and a sketchy man too

No

At least 75%

Oh ya, it’s his favorite game

Yes, he goes to the day care

3 times a week, and he loves

it

I’ve seen him do that a few

times

Family history Are there any diseases that

run in the family?

No

Medications and allergy Do you give John any

medications?

Does he have any allergies?

No

Not that we know of

Social history I’d like to ask you a few

questions now to get to know

more about John and his

family if that’s okay

Who currently lives at home?

Who looks after John most of

the time?

Does he go to daycare or

preschool?

Is your home childproof?

Sure

My husband, myself and

John

I do

No he stays at home with me

Yes

Page 287: CANADIAN IMGS GUIDE TO OSCE AND PRACTICE · in an OSCE scenario You can use this study guide to prepare for the NAC OSCE and MCCQE2. The content was reviewed for the most recent updates;

287

Where do you typically keep

the Lysol?

What do you and your family

do for work?

Does anyone in the

household smoke?

Do you feel John is being

treated well by all people

looking after him?

It’s usually locked in a

bathroom cupboard he can’t

reach

My husband works full time,

he is a lawyer, I work part

time in a salon

No

Yes

Closure Thank you Mrs Duggan for

sharing all this information

with me. I will go check on

John and let you know how

he is doing. Do you have any

other questions or concerns?

No, thank you, please let me

know what the eye doctor

thinks

Physical exam;

Not required in this case

Sample questions by the examiner;

1- If the cleaning agent was an alkaline, would you attempt to neutralize it with an acid? And

why?

Answer; No, because the heat resulting from the chemical reaction can burn the eyes and make the

injury worse

Page 288: CANADIAN IMGS GUIDE TO OSCE AND PRACTICE · in an OSCE scenario You can use this study guide to prepare for the NAC OSCE and MCCQE2. The content was reviewed for the most recent updates;

288

Rating scales points;

- Examinee introduced self and position

- Addressed patient with name

- Used proper non-verbal communication

- Organizational skills

- Recognized and managed emergency effectively prioritizing actions

- Examinee spoke clearly (accent didn’t get in the way)

- Listening and questioning skills

- Showed rapport with patient, and was sensitive to patient feelings

- Was supportive and non-judgmental

- Attentive to patient comfort

- Sensitive and respectful of patient’s concerns

Variations of the case and some tips;

1- Case; You are the doctor on call. At 2 am you got a call from a freaked out mother

whose daughter fell off the bed

History; parents co-sleep with their daughter. The 2 years old moves a lot during her sleep and managed

to cruise to the foot of the bed, she fell off their 3 feet bed, and most likely bumped her head, No loss of

consciousness, no vomiting. She cried for few minutes but her mother was able to calm her down. The

mother is wondering if she should take her to the hospital for a head CT. There are no other significant

accidents. She is otherwise healthy with normal growth, development, up to date vaccination and

satisfactory nutrition. The only medication she is on is vitamin D

Discussion; Falls remain a major part of injuries in kids. Since it is a phone call, the first thing you need to

do is to document the mothers name, phone number and home address in case the line got

Page 289: CANADIAN IMGS GUIDE TO OSCE AND PRACTICE · in an OSCE scenario You can use this study guide to prepare for the NAC OSCE and MCCQE2. The content was reviewed for the most recent updates;

289

disconnected. Then reassure the mother that help is readily available. Ask the mother to keep her

daughter up for a little while and watch for symptoms. The main concern is head injury and/or fracture.

Ask the mother to watch for seizures, change in level of consciousness and vomiting. If a serious injury is

suspected then the child needs to be brought to the hospital. The mother or the father can drive if calm

enough, if not then you need to send an ambulance. If the daughter is unstable it is as well preferable to

call an ambulance. Ask if one or both parents know CPR or first aid skills, and stay on the line till

ambulance arrives. If the daughter seems stable with no serious injury there is no need to bring her to

the hospital. You may later on counsel the mother about safe sleep practices.

The following website has the most recent Canadian recommendations;

http://www.cps.ca/documents/position/safe-sleep-environments-infants-children

2- Case; 4 months old boy brought in by his mother because of bruising.

History; the mother says her son has been easily bruising for the last week. He has bruises on the face,

neck, trunk and extremities. No blood in stool or urine and no bleeding mucus membranes. The baby is

feeding well; his activity level and sleep are unchanged. No fever, no seizures, no sick contacts and no

congestion. The mother however looks exhausted. Her husband works out of town, and has been away

for 6 weeks. Her parents are out of town, they helped the first month the baby was born but then she

was on her own. She says she is burnt out and is driving her son to a day home for a couple of hours

every day for the last 2 weeks so that she can breathe! Perinatal history was normal. Vaccinations are up

to date. Growth and development are within normal. The mother is a teacher, planning to take a year

off. Neither her nor the father smoke, drink, or do drugs.

Physical exam; make sure the room is warm and inspect the whole body documenting the distribution,

size, and color of bruises. Look for other injuries. Inspect the mucus membranes. Auscultate the heart

and lungs and palpate the abdomen. Examine the tone and primitive reflexes. Do fundoscopic

examination. Palpate the limbs for swelling or broken bones. And of course check the vital signs.

You will not be asked to examine a baby. In my mind I don’t see why not to examine a dummy, in which

case treat it as a real baby, and always wash your hands and be gentle.

Discussion; bruising in a non- cruising baby is suspicious for child abuse. However, medical conditions

like thrombocytopenia must be excluded. Your job as a physician is to take care of medical conditions

and injuries and to make sure the child is safe. Use non- judgmental language; you may say; we need to

do some tests to make sure your baby doesn’t have a disease causing all these bruising. We want to

Page 290: CANADIAN IMGS GUIDE TO OSCE AND PRACTICE · in an OSCE scenario You can use this study guide to prepare for the NAC OSCE and MCCQE2. The content was reviewed for the most recent updates;

290

make sure as well that your child is not being hurt by someone. If you highly suspect abuse after the

investigation results are back, admit the child and call the social worker and child protective agency.

Never accuse or blame parents or anyone else.

In this case, the baby may simply have a medical condition. However, he may as well be abused. Care

giver exhaustion and a new care giver are important factors. Investigations include; CBCD, PTT/INR,

fibrinogen, ALT, AST, urine analysis and skeletal survey.

If the patient was a toddler, take permission from the parent to talk to him//her in private. Parents are

usually ok with it. Refusal raises suspicions for abuse.

3- Injuries in kids are varied; falls, trauma, foreign body ingestion, poisons ingestion, etc….

Take a detailed history of current and previous injuries ask about risk factors for abuse.

Page 291: CANADIAN IMGS GUIDE TO OSCE AND PRACTICE · in an OSCE scenario You can use this study guide to prepare for the NAC OSCE and MCCQE2. The content was reviewed for the most recent updates;

291

Case 18; Motor vehicle accident (MVA)

Edited by; Dr. Abdullah Saleh

Door sign;

David Smith, 23 years old man, brought in by EMS after being involved in a motor vehicle accident.

Vital signs;

BP 100/70

PR 100

RR 16

Temperature 37 ⁰C

There is a nurse in the room. In the next 10 min, manage the patient

Note; In this case you will be communicating with the nurse and the patient. Communication with the

nurse is presented in italic. You need to make sure the patient is stable, give emergency medications,

take a focused and relevant history, perform a focused and relevant P.E and order investigations. The P.E

is included in the case. Treat the nurse with respect, and give clear orders specifying the dose and route

of medications.

Even though it is an ER case, it is important that you introduce yourself to the nurse and patient.

The NAC may have a similar stem but you have 11 min to complete the case

Sample notes to write for yourself;

David Smith, 23 yo Male

Mechanism of injury, SAMPLE hx

Primary Survey: Vitals, ABCDE

Adjuncts to primary survey (CXR, Pelvic Xray, C-Spine imaging, FAST, foley, +/- NG)

Secondary Survey (Head to toe PE)

Vitals recheck

Blood work

Page 292: CANADIAN IMGS GUIDE TO OSCE AND PRACTICE · in an OSCE scenario You can use this study guide to prepare for the NAC OSCE and MCCQE2. The content was reviewed for the most recent updates;

292

Patient encounter

Your actions Suggested verbalizing Patient or nurse response

Obtaining relevant hx from

nurse/paramedics

Need to ask specifically about

the details if not volunteered

by the nurse/paramedic

Mechanism of injury

SAMPLE;

S; signs and symptoms

A; Allergies

M; Medications

P; Past medical and surgical

history

L; Last meal

E; events and environment

leading to injury

Good morning, can you

please tell me about the

mechanism, time of incident,

who brought him in and what

interventions were done in

the field

The accident happened 1

hour ago, he was hit from

behind at 40 km/h when he

stopped to avoid hitting a

deer, he was the driver, and

there were no other

passengers and no

pedestrians. He was belted,

no LOC, and the airbags did

not deploy. There was no

significant intrusion and no

delay in extrication by the

paramedics and firemen. He

was placed in a c-collar and 2

large bore IV’s were started

at the scene and he was

given 1 L of NS.

Intro to the patient Hi Mr. Smith, I’m Dr-----I’m

going to take good care of

you, What is your name?

David Smith

Page 293: CANADIAN IMGS GUIDE TO OSCE AND PRACTICE · in an OSCE scenario You can use this study guide to prepare for the NAC OSCE and MCCQE2. The content was reviewed for the most recent updates;

293

I’m going to examine you and

ask questions at the same

time, is that ok?

Do you remember what

happened? Can you tell me?

Sure

Pt recounts mechanism

again.

Vitals and ABCD (Primary

Survey)

What are his vitals?

His GCS is 15/15 (EVM)

The patient is in an

adequately sized and

positioned C-Spine collar. He

is speaking full sentences. Can

you open your mouth for me

please?

The airway is patent and he is

protecting it. He is breathing

comfortably and on

inspection chest rise is equal

and bilaterally. No

paradoxical movements. No

bruises are seen. On

palpation, no subcutaneous

emphysema is felt and no

deformities notes Chest is not

tender. On auscultation I hear

good AE bilaterally, no

crackles or wheezes. I’m

feeling the pulse centrally, it’s

strong and regular at 110

bpm. Peripheral pulses are

present and palpable x 4.

To the nurse, He is

tachycardic.

Can we get 2 large pore IV

cannulas (if not already

present) and run 2 L of NS and

BP 100/70, PR 100, RR 16,

Temperature 37 ⁰C, O₂ sat

97% on 2L via nasal prongs

Will do

Page 294: CANADIAN IMGS GUIDE TO OSCE AND PRACTICE · in an OSCE scenario You can use this study guide to prepare for the NAC OSCE and MCCQE2. The content was reviewed for the most recent updates;

294

Assessment of vital signs

let’s make sure he is Typed

and Screened.

Repeat vitals after bolus

Can you check the vitals every

15 min.

Can you keep the patient

NPO, and run normal saline

0.9% IV at 150 cc/hour

Sir, do you have pain

anywhere?

How severe is the pain on a

scale from one to 10, 10

being the worst pain ever?

Do you need a pain killer?

I will perform a gross motor

and sensory exam now

Can you left your right arm

up, excellent, now your left.

Squeeze my fingers, flex your

elbow, now extend.

Can you wiggle your toes?

(Test the power of abduction

of the shoulders, flexion and

extension at the elbow, hips

and knees)

Do you feel numbness or

tingling anywhere in your

body?

Can you feel me touching

your skin?

Sure

Will do

IV lines in, Blood requested

Will do

At the back of my neck

about 4

I’m good for now

Page 295: CANADIAN IMGS GUIDE TO OSCE AND PRACTICE · in an OSCE scenario You can use this study guide to prepare for the NAC OSCE and MCCQE2. The content was reviewed for the most recent updates;

295

Exposure

Examining Power (refer to

chapter 3 for details)

Sensory exam (Refer to

chapter 3 for details)

(Full head to toe exam and

ROS)

Investigations

Let me know if you feel it

every time I touch you with it

and if it feels the same on

both sides (examine touch

sensation)

GCS 15, no apparent

disability.

The patient will need to be

exposed and I will do a full

examination from head to

toe, then roll the patient

(with help) while protecting

the C-spine and palpate the

spine while inquiring from the

patient if there is any

tenderness.( Examiner says ;

move on) A DRE will need to

be performed – to check for

tone, sensation and blood per

rectum. Examiner says move

on

H&N (pupils, fundoscopy,

otoscopy), CVS, Resp, Abdo,

Pelvis for stability, MSK,

Neuro (see Physical Exam

section for more detail)

Repeat vitals ?

Can I get the following

investigations done please:

CBC, Creatinine, urea, Lytes,

INR/PTT, AST, ALT, ALP,

Lipase, Calcium, Type and

Screen,

12 lead ECG,

BP 120/80, PR 95, other

vitals unchanged

Yes

Page 296: CANADIAN IMGS GUIDE TO OSCE AND PRACTICE · in an OSCE scenario You can use this study guide to prepare for the NAC OSCE and MCCQE2. The content was reviewed for the most recent updates;

296

Portable CXR

Cross table lateral C-Spine X

ray, pelvis x- ray

Will get lab and x-rays

History

Mechanism of injury ( for

additional details not

covered yet)

SAMPLE;

S; signs and symptoms

A; Allergies

M; Medications

P; Past medical and surgical

history

L; Last meal

E; events and environment

leading to injury

Is the patient intoxicated?

Smoking

Family history of bleeding

diathesis

So, Mr smith, I need to ask

you few more questions

Were you speeding?

Was your car damaged?

What did you do after the

accident?

Were you wearing your seat

belt?

Were other people injured in

the accident?

Do you have pain in your

chest?

Are you still doing ok?

Do you have headache?

Abdominal pain?

Pain elsewhere?

ok

No

There is some damage but

not too bad

I had this pain in my neck, I

called 911,

The ambulance guys placed

this thing around my neck

And tested my nerves just

like what you did

Yes

I was alone in the car, the

other driver seemed ok

No

Yes

No

No

None

Page 297: CANADIAN IMGS GUIDE TO OSCE AND PRACTICE · in an OSCE scenario You can use this study guide to prepare for the NAC OSCE and MCCQE2. The content was reviewed for the most recent updates;

297

Do you feel short of breath?

Did you lose consciousness

after the accident?

Did you have a seizure?

Were you under the effect of

alcohol or drugs when the

accident happened?

Have you ever been

diagnosed with a medical

disease or had a surgery?

Do you take any medications?

Do you have allergy to

medications or food?

When did you last eat?

Do you smoke?

Does anyone in your family

have a bleeding problem?

No

No

No

No

No, I’m healthy

None

No

I had lunch 6 hours ago

No

No

Summary and re assessment We have a 23 years old

healthy male involved in

MVA, with possible whiplash

injury as indicated by the

accident mechanism, midline

posterior neck tenderness

and neck pain. No

neurological deficit and no

evidence of other injuries.

Investigations including

Lateral –spine X-ray are

pending. Can we get a

surgical consult and another

set of vital signs please?

Vitals unchanged

Page 298: CANADIAN IMGS GUIDE TO OSCE AND PRACTICE · in an OSCE scenario You can use this study guide to prepare for the NAC OSCE and MCCQE2. The content was reviewed for the most recent updates;

298

Sir, would you like me to

contact your family or

friends?

Sure I will, is there anything I

can do for you?

It would be great if you can

call my parents, but please

don’t scare them

No thanks

Physical examination;

Included in the case

Sample questions you may be asked by the examiner;

None in this case

Rating scales points;

- Examinee introduced self and position

- Addressed patient with name

- Used proper non-verbal communication

- Organizational skills

- Recognized and managed emergency effectively prioritizing actions

- Examinee spoke clearly (accent didn’t get in the way)

- Listening and questioning skills

- Showed rapport with patient, and was sensitive to patient feelings

- Was supportive and non-judgmental

- Attentive to patient comfort

- Medical knowledge adequate

- No misinformation was provided to patients

- No concerning ethical/legal issues

- The examinee is respectful of other health care members

Page 299: CANADIAN IMGS GUIDE TO OSCE AND PRACTICE · in an OSCE scenario You can use this study guide to prepare for the NAC OSCE and MCCQE2. The content was reviewed for the most recent updates;

299

Variations of the case and some tips;

1- Case; 28 M, brought in by EMS after being involved in MVA, manage

Management; you start with your ABCD, the patient appears intoxicated and smells of alcohol. GCS is 8,

you decide to intubate, the examiner says done, so you assume that the patient is intubated. Since his

airway is secure and protected with the endotracheal tube, you inspect, palpate and auscultate the

chest for breath sounds. There is a large bruise on the R chest wall and you feel subQ emphysema. On

auscultation, you hear decreased breath sounds on R as compared to L hemithorax. The Resp therapist

tells you that it is difficult to ventilate the patient and he is becoming hypotensive. A quick examination

of the patient’s neck reveals a deviated trachea to the left and distended neck veins. You diagnose a

tension pneumothorax. The diagnosis is made clinically and there is no need to delay while waiting for a

CXR. You perform a needle decompression (2nd intercostal space, midclavicular line) and a gush of air is

released and the patient becomes easier to ventilate and normotensive. A 36 Fr Chest Tube is placed on

the Right side (5th intercostal space, anterior or mid-axillary line), a gush of air is released and blood

pools in the pleurovac. A CXR is obtain for confirmatory placement. Make sure to check the left side as

there might be a penumo bilaterally. Restart the primary survey from the beginning again. Obtain 2

large pore IV cannulas, and start IV fluids (if not already done). Continue with the circulatory

assessment, disability assessment will be difficult as the patient is intubated and presumably sedated.

The exposure should take place and the log roll + DRE. Continue to monitor the vital signs. You order

investigations; ABG, Blood glucose, CBCD, INR, PTT, AST, ALT, ALP, albumin, Calcium, TSH, lipase, CK,

blood alcohol levels, urine drugs screen, urine analysis, ECG, c spine films, pelvic xray and CXR (if not

already done). You need to consider the other adjuncts to the primary survey (foley – if no blood at the

meatus, and NG tube). Not much history is available; EMS reported finding the patient unconscious

behind the wheel, after his car struck a tree. He was wearing the seat belt. You look for a medical

bracelet and don’t find any. You start your secondary survey and physical exam. The pelvis feels

unstable. The patient is becoming hypotensive and tachycardic. It is important at this point to think of

shock and the different types of shock. In most trauma patients, hemorrhagic hypovolemic shock is the

most common. The five places patients lost blood are: thorax, abdomen, pelvis and retroperitoneum,

fractured long bones and the floor (as in bled on the scene or from scalp lacerations etc.). The xray looks

like an open book pelvic fracture and you immediately should proceed to bind the pelvis to decrease the

volume and hence attempt to tamponade the bleed. You give 2 L crystalloid and assess if patient

responds. If he continues to be hypotensive give blood. You perform a FAST screen, and consult surgery.

You ask the nurse if there was any information in the patient wallet or any numbers that you can

contact, she tells you that his mother number is the last number dialed on his phone, you say that you

will call his mother.

Page 300: CANADIAN IMGS GUIDE TO OSCE AND PRACTICE · in an OSCE scenario You can use this study guide to prepare for the NAC OSCE and MCCQE2. The content was reviewed for the most recent updates;

300

2- Note; the case above can be made even more complicated by getting the patient to fake

a seizure. Do your ABCs, make sure the patient is safe, give 2 mg Valium or versed IV,

and a loading dose of phenytoin (15-20 mg/kg, max rate 15mg/h). The seizure could be

secondary to head injury or the effect of drugs or alcohol. Order a head CT in addition to

the above investigations and recheck the pupils, DTRs, tone and Babinski sign.

Page 301: CANADIAN IMGS GUIDE TO OSCE AND PRACTICE · in an OSCE scenario You can use this study guide to prepare for the NAC OSCE and MCCQE2. The content was reviewed for the most recent updates;

301

References;

1- Macleod, John. Macleod's Clinical Examination. 12th ed. Edinburgh: Churchill

Livingstone/Elsevier, 2009.

2- Hui, David. Approach to Internal Medicine a Resource Book for Clinical Practice. 3rd ed. New

York: Springer, 2011.

3- Sabatine, Marc S. Pocket Medicine. 4th ed. Philadelphia: Wolters Kluwer Health/Lippincott

Williams & Wilkins, 2011.

4- Stephanie H Abrams, MD, MS, Robert J Shulman, MD. Approach to neonatal cholestasis. In:

UpToDate, Topic 5941 Version 19.0, UpToDate, Waltham, MA. (Accessed on November 20,

2014.). http://www.uptodate.com/contents/approach-to-neonatal-

cholestasis?source=search_result&search=neonatal+jaundice&selectedTitle=2~68

5- "Guidelines for Detection, Management and Prevention of Hyperbilirubinemia in Term and Late

Preterm Newborn Infants." Guidelines for Detection, Management and Prevention of

Hyperbilirubinemia in Term and Late Preterm. Canadian Pediatric Society. Accessed November

19, 2014. http://www.cps.ca/documents/position/hyperbilirubinemia-newborn

6- Mimi Zieman, MD.Overview of Contraception. In: UpToDate, Topic 5459 Version 83.0,

UpToDate, Waltham, MA. (Accessed on November 25, 2014.).

http://www.uptodate.com/contents/overview-of-

contraception?source=search_result&search=contraception&selectedTitle=1~150

7- "Pocket Guide to COPD Diagnosis, Managemnet, and Prevention." Global Initiative for Chronic

Obstructive Lung Disease. Accessed July 30, 2014.

http://www.goldcopd.org/uploads/users/files/GOLD_Pocket_2015_Feb18.pdf

8- "The 5A's of Breif 3-5 Minutes Tobacco Intervention." Saskatchewan Dental Therapists

Association. Accessed August 24, 2014.

http://www.sdta.ca/mrws/filedriver/DentistTobaccoInterventionAlgorithmSept06.pdf

9- Phyllis August, MD, MPH, Baha M Sibai, MD. Preeclampsia: Clinical features and diagnosis In:

UpToDate, Topic 6814 Version 58.0 , UpToDate, Waltham, MA. (Accessed on September 20/

2014.) http://www.uptodate.com/contents/preeclampsia-clinical-features-and-

diagnosis?source=search_result&search=preeclampsia&selectedTitle=1~150

10- Errol R Norwitz, MD, PhD, John T Repke, MD, Preeclampsia: Management and prognosis. In:

UpToDate, Topic 6825 Version 58.0 , UpToDate, Waltham, MA. (Accessed on September 20/

2014.) http://www.uptodate.com/contents/preeclampsia-management-and-

prognosis?source=search_result&search=preeclampsia&selectedTitle=2~150

11- Annekathryn Goodman, MD , Postmenopausal uterine bleeding In: UpToDate, Topic 5421

Version 10.0 UpToDate, Waltham, MA. (Accessed on October 25, 2014.)

http://www.uptodate.com/contents/postmenopausal-uterine-

bleeding?source=search_result&search=post+menopausal+bleeding&selectedTitle=1~33

Page 302: CANADIAN IMGS GUIDE TO OSCE AND PRACTICE · in an OSCE scenario You can use this study guide to prepare for the NAC OSCE and MCCQE2. The content was reviewed for the most recent updates;

302

Chapter 6

Ethical and Legal

Principles Edited by; Dr. Fraulin Morales

Page 303: CANADIAN IMGS GUIDE TO OSCE AND PRACTICE · in an OSCE scenario You can use this study guide to prepare for the NAC OSCE and MCCQE2. The content was reviewed for the most recent updates;

303

Introduction;

The goal of this chapter is to touch on some basic ethical and legal issues. Culture plays an

important role in shaping what is ethically acceptable and what not. The ethical principles

reviewed here represent Canadian values. You need to understand your own patient’s values

and work towards his/her best interest. For example; while the principle of autonomy dictates

your patient makes her own decision, you may treat a patient who is willingly giving this right to

another person, like an elderly depending on his nurse daughter to make the best decision for

him. That is ok as long as the patient is not being abused or coerced. Nevertheless, you should

try your best to involve your patient.

Ethical principles are guidelines, and some cases can get complicated. The good news is there is

an ethics committee in most hospitals. And if a committee is not available, there is always a

more experienced consultant. When uncertain, ask for an ethics consult.

I strongly encourage you to read Doing Right, 3rd edition by Philip C. Hebert. It’s a very useful

book. It covers basic and controversial ethical issues in a case based approach that motivates

your own curiosity. Follow the following link for the Canadian Medical Association code of

ethics

http://policybase.cma.ca/dbtw-wpd/PolicyPDF/PD04-06.pdf

The College of Physicians and Surgeons of Alberta standards of practice can be found by clicking

on the link below;

http://www.cpsa.ab.ca/AboutUs/FAQs_Main/FAQs_For_Physicians/Standards_of_Practice_FA

Qs.aspx

As you read through this chapter, I advise you to reflect on your own values and experiences

with patients. I personally revert to my patient’s welfare whenever there is a conflict. It makes

it easier to go back to basics when faced with a dilemma.

All Canadian trainees and independently practicing doctors are required to register with the

Canadian Medical Protective Association (CMPA). CMPA provides legal advice to its members

and helps them out when a complaint is filed against them. CMPA website contains useful

information for your reference; www.cmpa-acpm.ca

This chapter covers basic ethical expectations and concepts;

- Confidentiality

Page 304: CANADIAN IMGS GUIDE TO OSCE AND PRACTICE · in an OSCE scenario You can use this study guide to prepare for the NAC OSCE and MCCQE2. The content was reviewed for the most recent updates;

304

- Autonomy, informed consent, capacity, substitute decision maker and power of

attorney

- Beneficence and non-maleficence

- Justice

- Dealing with ones’ own and colleagues’ errors

Confidentiality;

Patients share their most personal information with their doctors trusting that their privacy is in

good hands. Doctors must strive to protect their patient’s information. The following situations

illustrate how to practically apply this principle;

1- During a patient encounter, always ask the patient if it is ok for family members to stay

in the room. Repeat your question when you are about to examine your patient. Look

for non-verbal cues, and if you need to ask very personal questions; politely ask family

members to leave the room.

2- Cultural differences may arise. In some cultures males are more dominant. One of my

patient’s husbands answered all the questions for her and she was ok with it. If the

patient agrees, this is acceptable. However, in certain situations you need to make sure

you talk to your patient in private, like when treating teenagers or when spousal or

elderly abuse is suspected; or when you feel your patient is shy to ask family members

to leave.

3- You will encounter caring family members asking about your patient’s health. Always

take your patient’s permission before disclosing any information. In most circumstances,

it’s better to talk in the presence of the patient.

4- Don’t talk about patients in elevators or restaurants or other public spaces; if you

absolutely have to discuss a case, don’t mention names, and keep your voice low.

5- Discard all papers with patient information appropriately by putting them in a shredding

box not the regular garbage.

6- Protect your patient’s information when using electronic health records; always log off

and protect your password. If you exchange emails with or about patients make sure to

use appropriate email protection. Avoid saving patient’s information on your lap top,

and if you have to, use a security password, and delete information you no longer need.

7- If a patient asks for a letter for his/her work, don’t include any personal information,

simply say; medical reasons. An example of a letter to employer is found in chapter 4:

writing and counseling tips.

Page 305: CANADIAN IMGS GUIDE TO OSCE AND PRACTICE · in an OSCE scenario You can use this study guide to prepare for the NAC OSCE and MCCQE2. The content was reviewed for the most recent updates;

305

8- There are some circumstances that require you to break confidentiality and share

patient information with others; this is when your patient or someone else is at risk.

Examples include: a homicidal or suicidal patient, certain infectious diseases like sexually

transmitted infections or infections you have to report to public health. Inform your

patient that you have a duty to disclose this information to protect him/her or others.

9- A final word about medical records; the original record belongs to the treating physician

office or hospital, but the patient has the right to have a copy of his/her own chart. The

physician can provide the copy for free or charge a fee.

Autonomy, capacity, informed consent, substitute decision maker

and power of attorney;

I grouped all these concepts together because they all come back to the same point: the

patient’s right to make his/her own health decisions. The following illustrate these principles:

1- The patient has the right to make his/her own health decisions as long as he/she is

competent to do so.

2- To be able to exercise this right the patient must be well informed. It is the physician’s

duty to fully inform the patient about his/her condition, available treatments, side

effects and benefits of treatments, outcome with and without treatment and treatment

alternatives. Treatment can include drugs, surgeries, life style modifications or a

combination of these. In some situations like a patient with cancer considering

chemotherapy, the amount of information can be overwhelming and it’s a good idea to

give the patient handouts to read through. Patients may need some time to consider

their options. In cases where imminent intervention is needed, explain to the patient

that not much time is left, but don’t pressure or force your patient to make a decision.

3- To be able to make a decision, the patient must have decision making capacity. Capacity

means the patient understands his/her options, and the consequence of taking an

action or not doing anything. The patient must be mentally clear; not delirious nor

demented nor suffering from a psychiatric condition that deters his/her ability to make

sound decisions like depression, psychosis or being actively suicidal. A competent

patient understands his/her choices and their consequences and is able to appreciate

the seriousness of his/her condition.

4- A competent patient has the right to make his/her own health choices even if they seem

irrational to the health care provider, and doctors must respect them. For example, a

patient with localized rectal cancer that can be cured by surgery refuses surgery and

uses homeopathic remedies instead. In these situations make sure the patient is capable

of making the decision, and explore his/her concerns in a non-judgmental way.

Page 306: CANADIAN IMGS GUIDE TO OSCE AND PRACTICE · in an OSCE scenario You can use this study guide to prepare for the NAC OSCE and MCCQE2. The content was reviewed for the most recent updates;

306

5- Only the patient can decide if family members can be involved in decision making. This

may be different among cultures. I had a Jehovah witness patient who had a third

degree burn. His hemoglobin was 6 and his platelet count was low. His wife strongly

refused blood transfusion. When I talked to him in privacy he said he wasn’t that

religious and wanted to have the blood but was afraid to upset his wife. He finally

decided to take the blood and asked us not to inform his wife. Always speak to your

patient privately and make sure they are not being pressured or coerced. It’s ultimately

up to the patient to decide how much family and friends know about his/her health and

the extent of their involvement.

6- I want to expand more on cases where a patient with a life threatening condition

refuses a potentially lifesaving treatment. In addition to explaining to the patient his/her

options, alternatives, benefits, side effects and outcomes must also be fully discussed.

Explore his/her concerns. Patients often have just reasons but may be willing to change

their mind if encouraged to open up (as in case 15, chapter 5). It is important as well to

discuss with the patient the goals of care. Things that may come up include; would you

treat an infection? What about if the patient needs resuscitation? In a patient who

refuses a lifesaving treatment would you offer ICU care? For example; is it ethical to

resuscitate a patient with renal failure who refuses dialysis? The patient will die without

dialysis and there is no point of trying to bring him/her back temporarily, this will only

increase suffering. The doctor has the right to refuse to resuscitate in this case, and the

patient must be fully informed upfront. A lot of patients change their minds when goals

of care are discussed. Perhaps they realize the seriousness of their situation. No matter

what your patient decides, be always supportive and compassionate.

7- Situations arise where a previously competent patient loses the capacity to make

decisions. Follow the patient’s previously expressed wish if known. If unknown look for a

substitute decision maker. The substitute decision maker may know the patient’s wish,

or act in the patient’s best interest if this wish is unknown.

8- Who can act as a substitute decision maker? It is wise to ask the patient about his

wishes and who would he want to act on his behalf should he become incompetent in

advance. In most cases the spouse or another close family member is the substitute

decision maker. If no one is found, the court may appoint a representative.

9- In Canada, a lot of patients have an advance directive or another legal document like a

power of attorney or living will that states their wishes and who they would like to act

on their behalf. Always ask if your incompetent patient has one and encourage your

seriously ill competent patient to obtain one.

10- Note that the patient autonomy is limited in cases where there might be harm to

him/her or others. The best example is the involuntary admission of suicidal psychiatric

patients to hospital. Patients are also treated without consent in emergency situations.

Page 307: CANADIAN IMGS GUIDE TO OSCE AND PRACTICE · in an OSCE scenario You can use this study guide to prepare for the NAC OSCE and MCCQE2. The content was reviewed for the most recent updates;

307

11- What about children? In most cases the guardian(s) (usually the parents) makes health

care decisions on the behalf of the child. Situations arise where teenagers ask you to not

involve their parents. It is expected that you protect their confidentiality in certain

situations like treating sexually transmitted infections or prescribing birth control. The

situation becomes, however, more complicated when it comes to a child refusing life

sustaining treatments and refusing parents involvement. As with all cases; explore your

patient concerns in privacy and try to understand their reasoning. Some children are

mature enough to be labeled competent to make their own health decisions. Always ask

for ethics consult in such cases, you may as well seek a legal advice.

Beneficence and non-maleficence;

Acting in the patient’s welfare and doing no harm may seem like straightforward concepts.

Nevertheless, there are situations where these ethical principles are challenged.

1- With the advancement and expansion of medical knowledge and therapies, one must be

thoughtful about treatment offered to patients. The physician has a duty to inform

his/her patient about his/her options, their side effects and the consequence of each.

Treatments have side effects that must be weighed against benefits. In general,

treatment shouldn’t put the patient in a worse condition. This may be hard to predict

sometimes, and it’s the physician’s duty to fully inform the patient and recommend the

best option available. And as mentioned earlier, respect the patient’s decision.

2- Side effects of therapy can be physical, mental, emotional and social. We tend to focus

on physical side effects while others may be more significant to the patient.

3- Explore your patient’s values and concerns to avoid unsuspected emotional or social

harm. There are resources that can be utilized to help the patient out. I admitted a

patient with heart failure, and while I was focused on his physical wellbeing, he was

extremely worried. When asked, he said he paid parking for only 2 hours and was very

concerned his car may get towed. He was going through financial difficulties and

couldn’t afford additional expenses. I got the social worker involved, his car was moved,

and he didn’t have to pay for one week parking, or towing expenses. This was a big relief

for him, and his emotional wellbeing helped him get better. In this case patient

admission to hospital caused a stress that was easily identified and removed.

4- Harm does ensue sometimes as in the case of a patient with arrhythmia who was put on

Amiodarone that caused lung fibrosis. It is hard for us as physicians to predict all

possible side effects. However they do happen and it helps to remember that drugs are

prescribed with the best intensions. Make sure the risks and side effects are less than

that of benefit. The patient must be fully informed about his options which in this case

Page 308: CANADIAN IMGS GUIDE TO OSCE AND PRACTICE · in an OSCE scenario You can use this study guide to prepare for the NAC OSCE and MCCQE2. The content was reviewed for the most recent updates;

308

include more sophisticated electrocardiac interventions, or an alternative medication. If

you know the percentage of patients who get the side effect, mention it. The patient

can then choose what feels best for him or her.

5- Physicians should take care of themselves so they don’t get harmed when treating

patients. An example includes using appropriate infection control precautions.

6- What if parents refuse a lifesaving treatment for their children? Like when a Jehovah

witness refuses blood transfusion of a bleeding kid. The physician can override the

parents in these cases and order blood. If the child was mature enough, involve him or

her in making the decision. If the child refuses blood and is deemed competent, then

respect his/her wish and try your best to treat him/her with alternatives. If in doubt ask

for help.

Justice;

Justice in medicine can be viewed from different perspectives;

1- Physicians should use their time wisely, so that patients are given good care, yet at the

same time the physician is not spending a lot of time unnecessarily with a single chatty

patient while others are waiting to be seen.

2- Physicians should be mindful of resources. It is imperative that patients are provided

with the best care and needed investigations are ordered even if expensive. However,

physicians should try their best to avoid ordering unnecessary tests as this exhausts the

system and may delay other, sicker patient’s access to them. An example is ordering

daily CBCD in a stable non bleeding patient admitted with COPD exacerbation. Health

care in Canada is publically funded, and patients have to wait sometimes. One can

appreciate the importance of wise utilization of time and resources.

3- Physicians must treat all patients with respect and grant them all access to high quality

care without discrimination based on age, sex, color, ethnicity, religion, sexual

orientation or social class.

4- Physicians should not abuse the system. An example would be a doctor using his

connections in the emergency room to get his relative seen sooner. This means longer

waiting times to other patients and can create a delay in seeing critically ill patients

(especially if a lot of health care providers do it).

5- Physicians should keep their relationship with the industry professional and prescribe

drugs and services based on scientific evidence. It is unprofessional to accept expensive

gifts from pharmaceutical companies like a prepaid family vacation. It is ok to listen to

marketing and weigh pharmacist’s evidence as long as the focus is the patient best

interest and no secondary gain is involved.

Page 309: CANADIAN IMGS GUIDE TO OSCE AND PRACTICE · in an OSCE scenario You can use this study guide to prepare for the NAC OSCE and MCCQE2. The content was reviewed for the most recent updates;

309

Dealing with ones’ own and others errors;

It is said that if you don’t make mistakes, you do nothing! Even the most skilled and careful

doctors make mistakes. But our job in Medicine involves caring for sick people. Mistakes can be

small and non-significant or cause direct harm to the patient. The following points illustrate

these concepts;

1- A duty of care to patients principle covers; “doing no harm” under its umbrella.

2- Negligence is a breach to the duty of care that results in harm to the patient.

3- The following example illustrates the concept of negligence; Doctor A fails to review the

blood work for one of his patients. Luckily, his patient results didn’t require an

intervention and no harm was incurred. A case of negligence can’t be made. Doctor B

fails to review his patient’s blood work. His patient had hyperkalemia that was missed.

He had a cardiac arrest few days later as his potassium levels went even higher. Medical

therapy was not enough, and in addition to the pain of resuscitation, he needed dialysis.

Doctors B scenario is a strong case of negligence.

4- Note that adverse events of medications and procedures should be well explained in

advance when obtaining consent. They are not considered negligence if they occur.

5- It is a difficult situation when a doctor makes a mistake. Yet it happens. A lot of negative

feelings and blame may occur, and the doctor may feel embarrassed and try to cover

up. One may wonder what to do in these situations? It all comes back to patient

welfare. One should be professional and do what is right;

Be honest; Patients deserve to know the truth.

The more serious the harm is, the more important it becomes to act quickly.

When you inform your patient of the mistake, be straightforward, apologize but

avoid blaming yourself or others, allow the patient to ventilate his/her anger or

frustration, focus on what needs to be done to help your patient and to minimize

the harm that resulted from the mistake.

6- Revise the system you work within and see what can be done to prevent this mistake

from happening in the future. Inform your patient that you will do your best so it

doesn’t happen again to him/her or other patients.

7- If you work with a group of physicians, and the mistake could have been prevented by

better communication and coordination, talk to your colleagues about it, so that you all

work together on a solution and preventative measures.

8- Honesty and acting quickly to help patients are a big relief for doctors, as it is not easy to

see patients suffer as a result of an error.

9- Physicians may worry about law suits. However, patients are less likely to sue if

informed in a timely and respectful matter. Waiting to disclose can impose more harm

and make penalties worse.

Page 310: CANADIAN IMGS GUIDE TO OSCE AND PRACTICE · in an OSCE scenario You can use this study guide to prepare for the NAC OSCE and MCCQE2. The content was reviewed for the most recent updates;

310

10- Doctors should contact CMPA in cases of law suits or to ask for advice in difficult

situations.

11- When a team of doctors treat one patient, there is usually one most responsible

physician. If the patient had a significant problem or lab result, you must do something

about it. If it is not within your scope of practice, you should inform the most

responsible physician, who will then take the appropriate action. It is a good idea to

write a letter to all treating physicians. In all cases, you should inform the patient.

12- What if your patient complains to you about another doctor?

Stay professional. Don’t take sides or blame anyone, you don’t know the other side of

the story. You may say something like: “I’m sorry this happened to you, what canI do to

help you?” You may offer to talk to the other doctor or the hospital committee to try to

prevent this error from happening in the future. Allow the patient to vent his/her

feelings, and focus on what needs to be done for your patient’s health and wellbeing.

Page 311: CANADIAN IMGS GUIDE TO OSCE AND PRACTICE · in an OSCE scenario You can use this study guide to prepare for the NAC OSCE and MCCQE2. The content was reviewed for the most recent updates;

311

References;

1- Hebert, Philip C. Doing Right: A Practical Guide to Ethics for Medical Trainees and

Physicians. 2nd ed. Don Mills, Ont.: Oxford University Press, 2009.

2- CMA code of ethics, CMA website, accessed December 25/2014,

http://policybase.cma.ca/dbtw-wpd/PolicyPDF/PD04-06.pdf

Page 312: CANADIAN IMGS GUIDE TO OSCE AND PRACTICE · in an OSCE scenario You can use this study guide to prepare for the NAC OSCE and MCCQE2. The content was reviewed for the most recent updates;

312

Appendex A;

Abbreviations;

AKI Acute Kidney Injury

CBCD Complete Blood count and Differential

COPD Chronic Obstructive Pulmonary Disease

CK Creatine Kinase

CKD Chronic Kidney Disease

CABG Coronary Artery Bypass Grafting

CHF Congestive Heart Failure

CVS Cardiovascular System

CMPA Canadian Medical Protective Association

DM Diabetes Mellitus

DKA Diabetic Ketoacidosis

DIC Disseminated Intavascular Coagulation

DVT Deep Vein Thrombosis

Etoh Alcohol

GI Gastrointestinal

IMG International Medical Graduate

INR International Normalized Ratio

LMN Lower Motor Neuron

LMP Last Menstrual Period

LDH Lactate dehydrogenase

MSK Musculoskeletal

MI Myocardial Infarction

MVA Motor Vehicle Accident

OSCE Objective Structured Clinical Examination

PE Physical Examination/ Pulmonary Embolism

PVD Peripheral Vascular Disease

PTT Partial Throbmoplastin Time

RA Rheumatoid Arthritis

SLE Systemic Lupus Erythematosus

UMN Upper Motor Neuron

U/A Urine Analysis

US Ultrasound

Page 313: CANADIAN IMGS GUIDE TO OSCE AND PRACTICE · in an OSCE scenario You can use this study guide to prepare for the NAC OSCE and MCCQE2. The content was reviewed for the most recent updates;

313

WBC White Blood Cells

XR X-Ray

Index

A

Abdominal pain, 32, 72, 208, 215, 301

Adolescent (teenager) history, 44

Alcohol, 34, 45, 124, 132, 140, 169, 210, 217, 227, 231,

254, 317

Allen test, 62

Ankle Brachial Index, 62

Ankle edema, 31

Ankles, 105

Arterial, 59

Ascites, 58, 70

Autonomy, 14, 309, 310

B

Back, 13, 95, 96, 163, 164, 171

Beneficence, 14, 309, 312

Biceps, 79, 80, 94

breast, 37, 38, 40, 107, 108, 142, 230, 238, 240, 242, 244,

253, 254, 255, 265, 269, 270, 272, 273, 280, 281, 283,

284

C

CAGE, 34, 35

Cardiovascular, 8, 12, 31, 50, 72, 156, 317

Carpal Tunnel Syndrome, 87

Cerebellar, 82

Chest pain, 13, 31, 32, 131, 144, 146, 155, 156, 186

claudication, 31, 33, 173

Confidentiality, 14, 308, 309

Constipation, 32, 158, 229

contraception, 36, 37, 38, 186, 236, 237, 238, 239, 242,

244, 247, 248, 253, 257, 264, 306

Cough, 13, 32, 130, 131

Counsel, 117, 124, 128, 235, 237, 259

Courvoisier’s sign, 72, 73

Cranial nerves, 74

D

Depression, 43, 137, 139, 143

dermatomal distribution, 61, 81, 173

Developmental milestones, 41

diabetes, 29, 37, 39, 120, 132, 139, 148, 158, 168, 183,

185, 194, 195, 197, 198, 200, 201, 202, 203, 204, 216,

240, 253, 280

Diarrhea, 13, 32, 33, 166, 197, 205, 206, 212, 213

differential diagnoses, 18, 26, 27, 28, 31

Dizziness, 31, 207

drawer, 104, 180

Dysuria, 32, 229

E

Elbows, 78, 89

Endocrine, 33, 263

ethical, 7, 18, 19, 20, 22, 129, 134, 141, 153, 161, 171, 180,

193, 204, 211, 224, 234, 248, 256, 266, 277, 278, 304,

308, 311, 312

Page 314: CANADIAN IMGS GUIDE TO OSCE AND PRACTICE · in an OSCE scenario You can use this study guide to prepare for the NAC OSCE and MCCQE2. The content was reviewed for the most recent updates;

314

F

Fatigue, 13, 31, 33, 137, 208

Fever, 33, 165, 175, 176, 186, 208, 280

Fundoscopy, 13, 75, 76, 109, 234

G

Gait, 83

Gastrointestinal, 8, 32, 317

Genitourinary, 32

Glasgow, 73, 84

H

Headache, 32, 33

HEEADSS, 44

Hematemesis, 32

Hematochezia, 32

Hemoptysis, 32, 131

Hepatojugular reflux, 55

Hips, 79, 100

History, 6, 12, 17, 25, 26, 28, 29, 30, 34, 35, 36, 37, 38, 40,

42, 130, 131, 134, 135, 138, 140, 142, 143, 147, 154,

156, 161, 162, 171, 172, 175, 180, 183, 193, 194, 206,

212, 213, 217, 227, 235, 240, 256, 257, 267, 279, 283,

284, 293, 294, 301

hypertension, 29, 37, 39, 52, 154, 232, 235

I

IMGs, 1, 2, 5, 6, 17, 20, 26, 48

injury, 33, 81, 104, 120, 174, 180, 287, 292, 293, 296, 297,

301, 302, 305

J

Jaundice, 32, 208, 229, 283

Joint pain, 33

Justice, 14, 309, 313

JVP, 50, 54, 55, 56, 57, 111, 133, 141, 142, 151, 160

K

Knees, 79, 102

L

legal, 7, 22, 35, 114, 134, 141, 153, 161, 171, 180, 193,

204, 211, 224, 234, 248, 256, 266, 268, 277, 304, 308,

311, 312

LMN, 79, 80, 317

M

M SIGE CAPS, 43

mammogram, 38, 142, 253, 257

MCC, 7, 17, 20, 22, 107, 128

MCCQE2, 7, 12, 17, 18, 19, 20, 22, 23, 114

McMurray test, 104, 105

medications, 26, 30, 31, 37, 39, 50, 115, 116, 118, 119,

132, 140, 144, 149, 159, 162, 177, 182, 186, 194, 195,

197, 199, 202, 207, 209, 211, 216, 227, 230, 235, 241,

242, 254, 262, 264, 280, 282, 291, 296, 301, 314

menopause,, 37

MMSE, 73

MOCA, 73

Morning stiffness, 33, 175

motor, 41, 61, 73, 77, 79, 80, 84, 96, 98, 170, 171, 221,

235, 296, 298

murmur, 48, 52, 53, 54, 154

Murphy’s sign, 72, 73, 188

Muscle pain, 33

Musculoskeletal, 9, 13, 33, 85, 317

N

NAC, 7, 12, 17, 20, 22, 23, 130, 137, 164, 174, 194, 205,

214, 226, 237, 249, 259, 270, 279, 286, 296

Natal, 40, 280

Nausea, 32, 208, 212, 227

non-maleficence, 14, 309, 312

O

Obstetrics and Gynecology history, 36

OSCE, 1, 2, 3, 6, 7, 12, 17, 18, 20, 21, 23, 26, 48, 81, 124,

318

P

Palpitations, 31, 33

Pap test, 38, 123, 231, 264

Pediatrics history, 39

Perinatal, 39, 289, 294

Physical Exam, 17, 160, 298

Post natal, 40, 280

Precordium, 50, 51

prenatal, 37, 39, 239

Prenatal, 39, 279, 280

Proprioception, 82

Psychiatry history, 42

Page 315: CANADIAN IMGS GUIDE TO OSCE AND PRACTICE · in an OSCE scenario You can use this study guide to prepare for the NAC OSCE and MCCQE2. The content was reviewed for the most recent updates;

315

R

Recreational drug use, 34

Respiratory, 8, 12, 31, 65, 135, 156, 215

Romberg, 82

Rotator cuff, 90

S

Scoring, 12, 20, 22

SEADS, 86, 89, 91, 96, 100, 102, 105

Seizures, 33

sensory, 61, 73, 77, 81, 82, 96, 98, 166, 170, 298

Sexual history, 35, 38

Shortness of breath, 31, 32

Shoulder, 78, 80, 90, 92

Skin rash, 33, 34, 186, 208, 215, 280

Smoking, 13, 34, 45, 124, 130, 132, 140, 169, 196, 210,

213, 217, 219, 231, 254, 282, 301

Sputum, 32, 131

Straight leg raise test, 98

substance abuse, 35, 43, 45, 129

Syncope, 31

T

Tenesmus, 32, 206

U

UMN, 75, 78, 79, 80, 318

V

Vaccination, 40, 282, 290

Vascular, 50, 58, 318

Vibration, 81

Vomiting, 32, 166, 197, 208

W

Weight loss, 33, 165, 176, 208

Wheeze, 32

Y

Yergason, 94, 95

Page 316: CANADIAN IMGS GUIDE TO OSCE AND PRACTICE · in an OSCE scenario You can use this study guide to prepare for the NAC OSCE and MCCQE2. The content was reviewed for the most recent updates;

316

Page 317: CANADIAN IMGS GUIDE TO OSCE AND PRACTICE · in an OSCE scenario You can use this study guide to prepare for the NAC OSCE and MCCQE2. The content was reviewed for the most recent updates;

317


Recommended